Praxis Test 3, praxis test 4, PRAXIS SLP, ETS Praxis Review 2, Praxis Review test 1 & 2

Pataasin ang iyong marka sa homework at exams ngayon gamit ang Quizwiz!

Parallel talk

comments what the child is doing that cow you have is brown and white

Submucous or occult cleft palate

condition in which the surface tissues of the soft or hard palate fuse but the underlying muscle/bone tissue do not

Projection fibers

connect the cortex and the subcortical structures

What muscle lengthen and tenses vocal folds?

cricothyroid muscle

You need to view Allison's vocal folds. You know that the otolaryngologist will do this, also, but you are fortunate to have instrumentation available to you. You decide to use a procedure that uses a pulsing light to permit the optical illusion of slow-motion viewing of the vocal folds. This is called

stroboscopy.

What is the condition called in which the surface tissues of the soft or hard palate fuse but the underlying muscle/bone tissue do not

submucous or occult cleft palate

What technique includes having the patient hold the breath, swallow, cough on exhalation, swallow again before breathing and breathe again

supraglottic swallow

Spanish speaking normal articulations

t/th subsitutions (ting/thing) a/ae substitutions (block/black) devoicing of final consonants (luff/love)

Auditory training in a client with hearing impairment focuses on

teaching a client to discriminate speech sounds.

Construct Validity are when

test scores are consistent with theoretical concepts or expectations

variables such as attitudes/opnions may change simply because they are measured more than once, this is a problem of

testing, reducing internal validity

External validity of a study may be threatened by

the Hawthorne effect.

Public 94-142 was retitled as

the Individuals With Disabilities Education Act.

What does the pyramidal system control?

Pyramidal system controls voluntary and fine motor movements

What test assess cognition, bheavior at levels of no response, generalized response, localzated response

Ranchos Los Amigos Scales

Perturbation is

a disturbance in the quality of the laryngeal tone, or fundamental frequency F0, of the voice.

Pitch

auditory sensation

What is the ability to identify the temporal order in which auditory stimuli occur

Auditory sequencing

African American dialects

/f/ for /θ/ in postvocalic position

semi vowel that is voiced bilabial glide, anterior and continuant

/w/

Normal pharyngeal swallow is how long

1 second

In apraxia of speech, where are the lesions often associated with

-Broca's area -speech-motor programming areas in dominant hemisphere

Asian speakers articulation

-Confusion of /r/ and /l/ -Subsitution of a/ae (shock/shack) -t/k substitution (tin/kin)

Limitation of standardized speech-language tests are

-Inadequate participant and response and sampling -contrived tests situations not representative of natural -limited participation of families -inappropriate for CLD

present progessive -ing

19-28

On avg most 1-month old infants consume about

2-6 ounces of liquid per feeding

Sensitivity to sound of the normal ear of a young adult is limited to

20 hz to 20,000hz

coustic immittance is measured with A. tympanometry. B. immittanceometry. C. impedanometry. D. acoustic reflex.

A. tympanometry.

What is another word for tongue tie?

Ankyloglossia

What CN innvervates the larynx, the levator veli palatini, palatoglossus, and palatopharyngeus muscles

CN X, vagus

What is a specific loss at 2,000 Hz indicated by bone-conduction testing?

Carhart's notch

What malocclusion states: arches themselves are generally aligned properly, but some teeth are misaligned

Class I malocclusion

What malocclusion states: the upper jaw or maxilla is protruded and the lower jaw or mandible is receded (overbite)

Class II malocclusion

What malocclusion states: maxilla is receded and mandible is protruded?

Class III malocclusion

What injury involve no open wound in the head no penetration into the brain

Closed head injury

extremely rapid speech, compression of syllables, deletion of syllables in words, transposition of sounds in words, and a high rate of disfluencies but normal articulation of sounds at a slower speech rate.

Cluttering

What involves deviations in the positioning of individual teeth

Dental Malocclusion

Test-teach-retest, ability to learn when provided instruction is when assessment

Dynamic assessment

What is a non invasive prodcedure of electroglottal yields an indirect measure of vocal fold closure patterns

Electroglottalgraphy EGG

What controls the postural support for fine motor movement

Extrapyramidal system

What stutter method reinforce fluent speech in natural environement arranges pleasant relax setting positiviely reinforces fluent utterance

Fluency Reinforcement

You are asked to design a treatment program for young children who stutter, including preschoolers. Among the following choices, which would you most likely select?

Fluency reinforcement

What teaches airflow management easy soft onset of phonation reduced rate of speech

Fluency shaping method

The concept of period is related to which of the following?

Frequency of sound vibrations

What dementia uninbited and inapporpriate social behavior excessive eating depression impaired judgement dominant language problems with better preserved memory and orientation

Frontotemporal dementia assoicated with picks disease

Lowest Frequency of a periodic wave is

Fundamental Frequency

Theory that states 1. phonetic descriptions are dependent on information from other linguistic levels 2. phonological rules map underlying representation onto surface pronounctions

Generative Phonology Theory

What theory sound structure of human language 1. phonetic descriptions are dependent on information from other linguistic levels 2. phonological rules map underlying representation onto surface pronunciations

Generative Phonology Theory

What test is used for identifying motor, verbal and eye opening?

Glascow Coma Scale

What aphasia 1. most severe of nonfluent aphasia 2. caused by extensive lesion affected perisylvain region, all language areas 3.profound impaired language skills 4. greatly reduced fluency 5. impaired repetition 6. perservation 7. impaired reading and writing

Global aphasia

Critical diagnostic features of the right hemisphere syndrome include which of the following?

Impaired narrative skills

During the study in the radiology department, the clinician mentioned to her student intern that the following technique would result in widening of the vallecula space:

Head down (chin tuck)

What kind of speech uses one word to communicate a variety of meanings?

Holophrastic speech

The criteria for diagnosing autism in children include which of the following?

Impaired social interaction, stereotypic behaviors and interests, and disturbed communication

Autosomal recessive deficiency of X-L iduronidase

Hurler's syndrome

A U-shaped bone that suspends the larynx is the

Hyoid

What results when the velopharyngeal port does not close the opening to nasal passage during production on non nasal sounds

Hypernasality

African American patients. You will remember that:

Hypertension is prevalent among African American adults, and it can result in hemorrhagic strokes

What is the lack of appropriate nasal resonance on nasal sounds ?

Hyponasality

Patient is taught to inhale rapidly while keeping esophagus open and relaxed

Inhalation method

What is it called when the patient keeps the esophagus open and relaxed while inhaling rapidly

Inhalation method

Patient impounds air in the oral cavity, pushes it back to the esophagus and vibrates the cricopharyngeus muscle

Injection method

What is also known as the vocalis muscle?

Internal thyroarytenoids

Paying attention to the same object or activity as you when directed to do so

Joint reference

The cricothyroid does what to vocal folds?

Lengthens and tenses vocal folds

What theory phonological properties are linear strings of segments FLAW: did not account for stress or prosodic variables

Linear phonology theory

If a mother whose mother has a history of alcohol abuse while carry her son, in speech/lang problems, you may see...?

Low birth weight and length, behavior problems, and possible swallowing difficulties

What uses incidental teaching through typical adult-child interactions Clinician: tell me what you want Clincian tell me the whole sentence

Mand-model

What the TM is ruptured and there needs to be a repair, what is it called?

Myringoplasty

Is AAE a substandard form of standard american english?

NO!!! IT IS NOT A SUBSTANDARD FORM OF STANDARD AMERICAN ENGLISH

What is a frequnecy with which a source of sound normally vibrates?

Natural Frequency

What theory says natutal phono processes are innate, acquired early in life children suppress prcoess that do not occur in language BY STAMPE does not account for nonnatural simplification in speech

Natural phonology therory

Type of AAC that uses bioelectrical signals such as muscle-action potentials to activate and display messages limited hand mobility

Neuro-assisted AAC

Is injury to brocas area essential to have broca's aphasia?

No

What are geometric, abstract and arbitratry and are specifically taught?

Non iconic symbols

When there is a class II malocclusion and the upper teeth from the molar forward are position excessively anterior to the lower teeth

Overjet

muscles is primarily responsible for vocal fold abduction?

Posterior cricoarytenoid

What is a hearing loss due to aging?

Prescbycusis

What system controls the voluntary and fine motor movements?

Pyramidal System

Electromyography

Studies pattern of electrical activity and muscle activity pattern

Predicatable articulation for Mandarin Speakers

Subsitutions for t/th (tin/thin) Epenthesis in words with consonant blends Confusions of /r/ and /l/

Normal Korean Articulation patterns

Substitution of a/ae (bock/back) Final consonant deletion (be-/bed) Confusion of r/l (glow/grow)

What contains an independent clause and one or more subordinate clause

T unit

What muscle exerts the pull that allows eustachian tube to open during yawning and swallowing?

Tensor Palatini

Federal law specifically mandates which of the following treatment guidelines?

Testing and evaluation materials must be provided and adminstered in the language or other mode of communication in which the child is most proficeient

What is it called when frequency of stuttering may decrease from the first to subsequent reading called?

The adaptation effect

A clinician is a member of a cleft palate and craniofacial team that asks her to conduct an objective assessment of a 6-year-old child's velopharyngeal mechanism. The clinician decides to do nasopharyngoscopy, in which the nasopharyngoscope is passed through the middle meatus and back to the area of velopharyngeal closure. What will this procedure enable the clinician to observe?

The child's posterior and lateral pharyngeal walls, as well as the nasal aspect of the velum and the adenoid pad as the child produces sentences

You are planning treatment for a child who needs to learn morphological features of language. You have selected four grammatical morphemes to teach at the word level. You will then teach each word to a training criterion. You show a stimulus card, ask a question, wait for the response, reinforce the correct response, score the response, and wait for a few seconds before presenting the next opportunity for the child to produce the target morpheme. What is this procedure called?

The discrete trial method

What is Ex post facto research?

The independent variable have occurred in the past and the investigator tries to find potential causes of the dependent variable

What do social interactionists believe?

The structure of human language may have arisen from language's social communicative function in human relations

child has Moebius syndrome.

This child has delayed language and an articulation disorder, as well as bilabial paresis and weak tongue control for lateralization, elevation, depression, and protrusion, a mask-like face, a history of feeding problems in infancy, and unilateral or bilateral paralysis of the abductors of the eye.

What aphasia 1. fluent speech with normal phrase length, good articulation, good prosody 2. GOOD REPETITION but poor comprehension of repeated words

Transcortical Sensory Aphasia

What aphasia has nonfluent speech, paraphasia and agrammatic speech?

Transcortical motor aphasia

What aphasia has good fluent speech, normal prosody, and good articulation?

Transcortical sensory aphasia

What muscle adducts the vocal folds?

Transverse arytenoids

Acoustic immittance is measured with

Tympanometry

If you use phonetic placement to help a child produce /r/, what will you do?

Use verbal instructions, modeling, physical guidance, and visual feedback to show the child how /r/ is produced.

Frequency range, optimal pitch and habitual pitch are measured in

Visi pitch

What is the volume of air that a person can exhale after maximal inhalation

Vital capacity

Is excessive or even stress on syllables a part of ataxic dysarthria?

YES, excessive or even stress is a part of ataxic dysarthria

Phasic bite

a rhythmical opening and closing of the jaw infant gently nibbles

Carhart's notch

a specific loss at 2,000 Hz indicated by bone-conduction testing

overpressure due to prolonged adduction/tight closure of vocal folds

abductor spasmodic dyphonia

intermittent invontary fleeting vocal fold abduction

abductor spasmodic dysphonia

A patient was having difficulty swallowing solids and liquids and was regurgitating his food hours after eating. During a barium swallow the physician noted that there was a complete loss of peristalsis. She also noted that the nonrelaxing lower esophageal sphincter (LES) was preventing the downward passage of the bolus into the stomach. This condition is called

achalasia.

Tympanometry measures

acoustic immittance

Gliding Consonant cluster reduction Stopping Depalatization Vocalization Should persist after

age 3

Reduplication weak syllable deletion consonant assimilation prevocalic voicing fronting FCD Diminutization should all disapear by

age 3

Backing

alveolar sounds (t/d) subsituted with velar sounds (k/g) gog for dog

Tidal capacity

amount of air in inhalation and exhalation in normal breathing cycle

disorder in the oral phase includes

anterior as opposed to a posterior tongue movement

damage to this artery can cause a person have paralysis of the feet and legs

anterior cerebral

Development of phonation through coughing or throat clearing is an effective technique in the treatment of

aphonia

Response to intervention

at risk students who are struggling in classrooms are given increasing amounts of targeted invidual and small group support within the classroom setting before a special education referral is made form of dynamic assessment that uses scientifically based instruction in the regular education classroom with students who struggle

Excessive and even stress is in what dysarthria

ataxic dysarthria

An electromyographic assessment can be conducted by

attaching electrodes to structures (oral, laryngeal, pharyngeal muscles)

Projection

attribute their own emotions to someone else

Surgeon closes a cleft of the soft palate first and cleft of hard palate later is called

delayed hard palate closure

In Dysarthria, how is speech interrupted?

by forced inspirations and expirations that interrupt speech

Stuttering may be an operant behavior changed by

changed by its consequences

Regulatory functions in pragmatics are

child attemps to control the behavior of others (do as i tell you do)

Heuiristic functions are when

children attempt to have environment explained to them tell me why, whats that, why doggy bark

Rationalization

client provies a logical but untrue response a client who stutters may say no one will talk to me because I stutter

Displacement

client takes his or her feelings of hostility about a situation and transfers to a safe person/thing

Self-talk

clinician describes her own activity look, im putting the dress on the doll

Rinne test

compares air and bone conduction hearing unilateral hearing loss screens for conductive hearing loss

The radiographic imaging procedure that allows X-ray beams to circle through segments of the brain and pass through tissue while a camera takes pictures of sections of the structures being scanned is known as

computerized axial tomography (CAT) scan.

What releases and arrests syllables

consonants

each item on the test to make sure that all items were relevant to children's language skills is called

content validity

Extrapyramidal system controls what?

controls the postural support for fine motor movement

In AAC, small number of words that constitute the majorite of what is said

core vocabulary

select 70 vocabulary words that are functional in Aubrey's environment and these words are used for training. You are using which approach?

core vocabulary approach

What muscle lengthens and tenses vocal folds to increase pitch

cricothryroid

Which muscle has the greatest control over the fundamental frequency of the laryngeal tone by lengthening or tensing the vocal folds.

cricothyroid

Action + agent

daddy kiss

What model shows when a child is unable to cope with expectations of fluent speech productions may begin to stutter

demands and capacities

Webster test

determines whether an issue is conductive OR sensorineural hearing loss

Articulation characterized by groping, inconsistency, and errors of sound and syllable sequencing strongly suggests

developmental apraxia of speech.

Laryngeal Cancer

difficulty swallowing ear pain lump in neck ongoing sore throat

When this muscle contracts, it elevates the hyoid bone.

digastric.

Ataxic cerebral palsy distinguishing features are

disturbed balance, awkward gait, and uncoordinated movements, some dysarthria due to cerebellar damage

Construction of a palatal-lift appliance is appropriate for a patient with flaccid paralysis characterized by an intact palate that does not function.

flaccid paralysis of soft palate

Naturalistic teaching

focuses on the successful production of utterances that are useful in context for communicating.

Stuttering in preschool children occur where

function words than on content words

Toddlers and preschoolers respond better WHAT and than to structured exercises.

games and play activities

type of vision loss, half side of vision

hemianopsia

Thermal-tactile stimulation is indirect or direct of dysphagia

indirect

What is it called when a child is unable to close velopharyngeal port and as a result has difficulty producing non-nasal sounds

hypernasality

infant presents with bilateral choanal atresia with respiratory distress, recommendation is to

immediate oral airway placement gavage feedings

acoustical, mechanical or electrical resistance to motion or sound transmission is called

impedance

Spanish speakers may use multiple negation

improperly

What is otitis media?

infection in the middle ear

external otitis

infection of the outer ear canal swimmer's ear

In browns morphemes, what is in stage V

irregular third person contractible auxillary contractible copula

Manual depression of the larynx will serve to

lengthen the vocal folds, allowing them to vibrate at a lower frequency.

Difficulty using cohesion devices (therefore, for example) is evidence of a

language impairment

What is called when the structure at the inferior portion of the tongue that connects with the mandible

lingual frenum

Granuloma

localized inflammatory vascular lesion -caused by intubation, vocal abuse, GERD -breathy hoarse voice

Granuloma

localized, inflammatory, vascular lesion that is usually composed of granular tissue in a FIRM ROUNDED SAC caused by intubation, GERD, vocal abuse

Fundamental Frequency is the

lowest frequency of a periodic wave first harmonic

Hearing level

lowest intensity of a sound that will stimulate the auditory system

agent + object

man hat

In a voice evaluation, air pressure can be measured with a

manometer.

Hearing loss the occurs when the middle ear and inner ear are not functioning properly is known as

mixed hearing loss

Ethnographic studies are

mostly descriptive

What is it called when a single subject design avoids the disadvantage of treatment withdrawal which the effects of treatment are demonstrated by showing that untreated skills did not change and only the treated skills did

multiple baseline designs

Electroglottalgraphy

non invasive electroglottal yields indirect measure of vocal fold closure patterns

Where there is no relationship between two variables being studied

null hypothesis

Ordinal Scale

numerical scale that can be arranged according to rank

factor that may postively influence pediatric feeding for infants is

nutritive sucking

cross-sectional studies?

observations are made of differences between subjects of different ages to generalize about developmental changes that would occur within subjects as they mature.

Spanish speaker normal productions are

omission of /h/ in word-inital position (-elp/help) t/th subsitutions in word-intial positions (tin/thin) Devoicing of final consonants (beece for bees)

Anterior as opposed to a posterior tongue movement is a swallow disorder in what area?

oral phase

What is done to close the cleft or clefts of the palate (9-24 months)

palatal surgery

Velopharyngeal closure is largely produced by soft palate elevation, the opposing muscle is the

palatoglossus

Impaited facial recognition is more common in patients with

posterior right hemisphere damage

over and underextends words in piagets stage is

preoperational 2-7 years

A manometric assessment assess

prep phase of the swallow using posterior and lateral plane examinations

Name brown morpheme order

present progressive -ing prepositions on, in regular plurals -s irregular past tense 's posessive

What are generalized productions

probes

The 11 paired internal intercoastal

pull ribs down to decrease diameter of the rhoracic cavity for exhalation

What do the 11 paired INTERNAL intercostal do?

pull ribs downward to decease the diameter of the rhoracic cavity for exhalation

Alveolar

raising the tip of the tongue to make contact with the alveolar ridge

sound waves travel back after hitting an obstacle with no change in speed progration

reflection

Hyperkeratosis

rough lesion, often benign due to tissue irritation, smoking, gerd, vocal abuse

What helps determine whether a client needs a more complete assessment

screening procedure

Brown's morphemes are acquired by children in an order that is determined by

semantic and syntactic complexity, with the simplest forms acquired first

excessive drooling is also known as

sialorrea

What is the back and forth movement of symmertrical and periodic

simple harmonic motion

What is a wave with horizontal and vertical symmetry

sinusoidal wae

Auditory Discrimation

skills that enable children to identifu differences between sound stimuli

Corniculates

small and cone-shaped and sit on the apex of the arytenoids

myringotomy

small incision into ear drum to relieve pressure or drain fluid

The softest level of hearing at which a person can understand 50% of the words presented is known as

speech reception threshold

Intensity

sound pressure

refraction

sound waves move from one medium to another, motion causes bending of the sound wave due to change n it speed of propagation

the laryngeally produce sound is modified by resonating cavities

source-filter theory of speech

Dysarthria is

speech disorder associated WITH muscle weakness or paralysis

a person who deciphers speech by looking at the face of the speaker and uses visual cues to understand what the speaker is saying is known as

speech reading

Speech samples of persons who clutter may contain such productions as "many thinkle peep so." This phenomenon is a

spoonerism.

What cleft is called when surface tissue and hard palate are fused together but not underlying muscle and bone not fused

submucous cleft

What is external otitis also known as

swimmers ear

Linguavelars are produced by

the back of the tongue rising to contact to the velum

The overshooting or undershooting of an intended target is typical of individuals with deficits related to a lesion of

the cerebellum.

standardized, norm-referenced instruments to assess speech-language functioning allow

the clinician to understand and make informed statements about how a client's performance compares with the performance of other people.

What is a lingual frenum?

the structure at the inferior portion of the tongue that connects the tongue with the mandible

What is a silent prolongation?

the same as an articulatory posture without voicing

total lung capacity is

the total volume of air in the lungs

Whispered speech is composed largely of APERIODIC sounds because

the vocal folds do not vibrate while whispering is taking place.

Vital capacity

the volume of air that patient exhale after maximal inhalation

A retrocochlear disorder is when

there is damage to nerve fibers along the ascending auditory pathways from the internal auditory meatus to the cortex

What are divided into two muscle masses

thyroartenoids

The amount of air inhaled and exhaled during normal breathing cycle

tidal volume

Cuneiforms are

tiny cone-shaped cartialges under mucous membrane that cover aryepiglottic folds

total volume of air in the lungs

total lung capacity

-major dysfunction in his gastrointestinal tract -short bowl syndome, removeal of intestense -gastroenterologist would recommend

total patenteral nutrition (TPN)

Which aphasia has 1.nonfluent, paraphrasic, agrammactic telegraphic speech 2. INTACT repetition 3. echolalia/perservation

transcortical motor aphasia

The thryoarytenoids are divided into

two muscles masses

Myringoplasty

tympanic membrance is permenently ruptured, repairing the tympanic membrane

Holophrastic speech

uses one word to communicate a variety of meanings

Reminiscence therapy is

using past experiences and events of the patient's life photographs of patients family music they like

cranial nerve VIII, which has two branches: the branch and the branch, which carries electrical sound impulses from the cochlea to the brain.

vestibular, auditory-acoustic

advantages of standarized speech-language tests

yield quantitative scores converted in measures like percentile rank schools use normatively based quantitative measures for placement

Fronting

when velar sounds (k/g) are replaced with alveolr sounds (t/d) tat for cat

Fiber-optic nasopharyngoscopy or laryngoscopy evaluated what?

-Phonation -vocal fold anatomy and physiology for voice production

When a sound from a nasal consonant carries over to adjacent vowel is called

Assimilative nasality

What are skills enable children to identify differences between between sound stimuli?

Auditory Discrimination

What refers to the ability to process acoustic stimuli that are presented at different rates or speed?

Auditory Rate

What is the ability to ignore irrelevant acoustic stimuli and focus on important information?

Auditory attention

Right hemisphere specializes in

visual and spatial information

Juncture

vocal punctuation, intonation and pausing that mark special distinctions or grammatical divisons in speech

Internal thyroarytenoid is also known as the

vocalis muscle

When a vowel is subsituted for a syllabic consonant (bado instead of bottle)

vocalization

what is performed to repair the cleft of the palate by raising two bipedicles bringing them together to close the cleft

von Langenbeck surigcal method

Vocalization

vowel is subsituted for syllabic consonant

What forms the nucleus of syllables

vowels

Stopping

when a fricative like (f/s) is subsituted with a stop (p/d) pan for fan

Single-subject designs components

-useful in establishing treatment efficacy -multiple-baseline design avoids the disadvantage of treatment withdrawal -disadvantage of single-subject designs is that they cannot efficiently predict the behavior of groups of individuals

Hemangioma

-soft,pliable, fillefd with blood -caused by intunation or GERD

Spastic CP

-stiff, arupt, jerky slow movements -increased spasticity -increase muscle tone -increase ridgidity damage in motor cortex or direct motor pathway

Normal Spanish articulation productions

-Insertion of schwa before word intial /s/ clusters -b/v substitutions -devoicing of final consonants

Hyponasality components

-Lack appropriate nasal resonance on nasal sounds -substutite oral sounds for nasal (baby for maybe)

Right Hemisphere disorders consist of

-attentional and perceptual deficits -affective deficits -communicative deficits

Leukoplakia

-benign growths of white thick patches -due to tissue irritatin, caused by alcohol, smoke, vocal abuse -considered precancerous

What would u see in orofacial myofunctional disorder (tongue thrust)

-deviant swallows -tongue exert force agianst front teeth -articu errors with s,z

allophones

-may vary from production to production -do not change word meaning -variation of phonemes -percevied as the same

You are evaluating a basketball coach, Susan, who tells you, "I am having problems with my voice." You notice intermittent, involuntary, fleeting vocal fold abduction when she tries to phonate. This is known as A. abductor spasmodic dysphonia. B. adductor spasmodic dysphonia. C. bilateral paralysis. D. unilateral paralysis.

A. abductor spasmodic dysphonia.

Establishment of which of the following is most important in ensuring that the results of any diagnostic test of speech or language are replicable? A. Content validity B. Interjudge reliability C. Split-half reliability D. Face validity

B A test that has interjudge reliability is one whose results are replicable, even if different people administer the test.

An SLP receives a referral regarding a 4-year-old boy who uses two words spontaneously and functionally, who began walking at 3 years of age, and who responds to his name inconsistently. On the basis of the information alone, the SLP can legitimately conclude that the child's communication profile reflects A.a developmental delay B.autism spectrum disorder C.a chromosomal anomaly D.a metabolic disorder

A A 4-year old typically developing child would have 4 word utterances & would respond to his name consistently. Most children start to walk around age 1. Therefore, the delay in walking, along with the delay in language, indicates a general developmental delay.

Which of the following muscles is primarily responsible for vocal fold abduction? A. Posterior cricoarytenoid B. Cricothyroid C. Interarytenoid D. Lateral cricoarytenoid

A Posterior cricothyroid muscle has its origin on the posterior wall of the cricoid lamina, and courses anterolaterally and then anteriorly to insert onto the lateral aspect of the arytenoid cartilage. Given the anatomy of the cricoarytenoid joint, posterior cricoarytenoid contraction (shortening) can only produce rotation of the arytenoid cartilages such that the vocal processes are displaced laterally, abducting the true vocal folds.

The most serious limitation of employing imitation as an intervention strategy for children with a language impairment is that imitation A. lacks communicative intention B. relies on semantic knowledge C. is clinician controlled D. is contextualized speech

A The most serious shortcoming of imitation as an intervention strategy is that imitation is not an intentionally communicative act.

regular plural inflection -s

24-33

irregular past test verbs

25-46

past tense regule -ed is typically mastered by

26-48 months

prepositions in, on

27-30

When should a child be able to understand agent-action relationships?

3-4 years

Symptoms of Parkinson's, Alzheimer's, or Pick's disease, evidence of damage to the basal ganglia, mask-like face, slowness of movement, micrographic writing, monopitch and imprecise articulation among other symptoms

hypokinetic dysarthria

A person who deciphers speech by looking at the face of the speaker and using visual cues to understand what the speaker is saying is using a technique known as A. cued speech. B. visual speech. C. speech reading. D. deaf speech.

C. speech reading.

Linguistic approaches to the treatment of sound-production errors in children are based on the notion that the errors are systematic and rule-based and that the goal of treatment is to modify a child's rule system to approximate the rule system used by adults. Which of the following is a treatment objective that reflects a linguistic approach to treatment? A. The child will contrast alveolar stops with velar stops in meaningful word pairs. B. The child will produce voiceless alveolar sibilants correctly in unstructured conversation. C. The child will coarticulate stop plus liquid clusters as easily as other children of the same age. D. The child will recognize and identify phonetic distortions of his or her error sound.

A The objective refers to speech sounds in terms of general distinctive-feature classes, rather than in terms of isolated phonemes or overly specific classifications.

Which of the following is a typical symptom of cerebellar involvement? A. Overshooting or undershooting an intended target B. Rigidity during voluntary motions C. Spasticity during involuntary action D. Word-finding difficulty

A The overshooting or undershooting of an intended target is typical of individuals with deficits related to a lesion of the cerebellum.

An adult client exhibits visuospatial disorganization, an inability to initiate interactions, left-side neglect, and lack of facial expression. This combination of symptoms is most likely associated with which of the following? A. Right-hemisphere traumatic brain injury B. Left-hemisphere cerebrovascular accident C. Bilateral traumatic brain injury D. Alzheimer's dementia

A The symptoms listed are indicative of deficits that generally are associated with RHD.

What is 1. most debilitating and pervasive word finding difficulty 2. fluent speech

Anomic Aphasia

Which of the following cognitive rehabilitation practices is most appropriate for the SLP to use to help increase Alan's success in learning basic information about his external memory aid? A.Errorless learning B.Attention process training C.Method of vanishing cues D.Expanded rehearsal

Correct Answer: A Option (A) is correct. The most appropriate practice for the clinician to use in this instance is errorless learning.

When would you diagnose a disorder of fluency?

5% of the words spoken are disfluent

The veterbral column consists of

7 cervical, 12 thoracic, 5 lumbar, 5 sacral and 3-4 coccygeal (fused) vetebrae

utero development of hard palate fuse at

8-9 weeks

Which of the following types of cerebral palsy is characterized by slow, arrhythmic writhing and involuntary movements of the extremities? A. Athetosis B. Spasticity C. Hypotonia D. Bulbar palsy

A Athetosis is a type of CP that is characterized by slow, arrhythmic writhing & involuntary movements of the extremities.

Questions 64-67 refer to the following. Michael is a 32-month-old boy who has been receiving early intervention services over the past ten months for delayed speech and expressive-language development. Although his birth was reportedly unremarkable, Michael does have a history of recurrent otitis media with effusion. His parents described him as having been a "well-behaved and quiet baby." When Michael began receiving services, he communicated mainly through gestures and crude vocalizations. An open resting mouth position with slight tongue protrusion was sometimes noted. However, his receptive-language skills were found to be age appropriate and he showed no oral motor deficits during feeding. Michael's expressive-language skills have shown some progress since he began working with the speech-language pathologist, but he remains poorly intelligible. Michael's imitation of tongue, lip, and jaw movements is characterized by inconsistent groping and errors of sequencing not observed in his spontaneous oral movements. Michael has an age-appropriate vocabulary and produces utterances of up to five words. Articulation errors, especially metathesis of phones and syllables, increase as his utterance length increases. Michael's intelligibility is greatest at the single-word level. Automatic speech and highly familiar utterances are much more intelligible than his imitated productions. QUESTION: Given Michael's age, which of the following is likely to be the most effective strategy for speech and language intervention? A. Using structured play to address short-term goals B. Facilitating Michael's ability to self-monitor his speech C. Instructing Michael's parents in structured home exercises for their son D. Introducing augmentative and alternative communication devices

A Toddlers & preschoolers respond better to games and play activities than to structured exercises

Which of the following, if observed in the speech of an African American child, is most likely to represent a dialectical variation rather than an articulation error? A./f/ for /θ/ in postvocalic position B./θ/ for /s/ in all positions C.Affricates for fricatives in word-final position D.Dentals for velars in word-initial position

A Use of voiceless labiodental fricatives for voiceless interdental fricatives is a feature of African American Vernacular English (AAVE).

Which of the following would be most likely to help a client who has aphonia? A. Development of phonation through coughing or throat clearing B. Pairing the production of /s/ and /z/ C. Respiratory exercises D. Easy initiation of phonation

A Development of phonation through coughing or throat clearing is an effective treatment in the treatment of aphonia.

The speech reception threshold (SRT) is a basic component of an evaluation of hearing function. Which of the following statements about the SRT is most accurate? A.It is measured in decibels and corresponds to the intensity level at which spondaic words can be recognized approximately 50% of the time. B.It makes use of test materials that are limited to monosyllabic words. C.It provides information on how well speech is understood at conversational levels. D.It is useful in validating acoustic intermittence measures.

A This is an accurate definition of speech reception threshold.

Which of the following is the ratio of reinforcement that will most quickly cause a newly acquired behavior to be habituated? A .A random ratio of tokens to correct responses B. A ratio of 1 token to 1 correct response C. A ratio of 1 token to 4 correct responses only D. A ratio of 1 token to 10 correct responses only

A When the goal is to reinforce a behavior that has already been acquired, a random ratio of tokens to correct responses creates an intermittent reinforcement schedule and is the most effective. Such a reinforcement schedule decreases the client's dependence on the token reward.

Which of the following factors contributes to UES opening? Select all that apply. A. Partial relaxation of the cricopharyngeal portion of the inferior constrictor muscle B. Superior and anterior hyolaryngeal excursion C. Velopharyngeal closure D. Posterior and inferior hyolaryngeal excursion

A & B Partial relaxation of the cricopharyngeal portion of the inferior constrictor muscle is part of the neurological sequence. Superior & anterior hyolaryngeal excursion provides a mechanical opening.

Fela is a third-grade student in a public school. She is a speaker of African American Vernacular English (AAVE) who has difficulty with the Standard American English (SAE) dialect used in her classroom. Her teacher believes that Fela's language skills are affecting her academic performance and has referred her to the school's speech-language pathologist. Which of the following is an appropriate rationale for providing language intervention for Fela? Select all that apply. A.It will likely foster better communication with Fela's linguistically and culturally diverse peers. B.It will likely improve Fela's code-switching ability with her teacher and other adult speakers of SAE. C.It may expand Fela's later academic and vocational opportunities. D.It will likely lead Fela to adopt SAE as her primary dialect.

A, B, & C The intervention will foster better communication because Fela and her peers will have a common dialect. Also, the intervention will provide Fela with the ability to switch easily between dialects using SAE structures. Furthermore, being able to switch codes will enable Fela to participate in more educational and vocational opportunities.

An SLP who is treating an adolescent who stutters designs a treatment plan that includes three fluency management strategies: prolonged speech, cancellation, and pullout. Which of the following is true about the use of these treatment strategies? Select all that apply. A.Use of prolonged speech is likely to reduce the frequency of part-word repetitions and sound prolongations significantly. B.Each of the three strategies entails deliberate regulation of speech motor movements. C.The client will seek to apply cancellation whenever he feels anxious about the possibility of stuttering overtly. D.The client will seek to apply pullout during the course of part-word repetition or sound prolongation.

A,B, & D Prolonged speech, deliberate regulation of speech motor movements, and pullout could all be appropriately applied

A researcher is interested in completing a single-subject design experiment to study the additive and subtractive effects of individual components of treatment rather than the comparison of two treatments. He decides to use an interaction design to study the interactive effects of two or more variables. The researcher is also interested in examining the effects of both variables alone and in combination. He is interested in isolating the components that are effective to any extent from those that are not at all effective and controls for phase lengths during the treatment. He also counterbalances the order of treatment in the experiment. The most appropriate design for this experiment would be

A-B-BC-B-BC.

A clinician has been asked to evaluate a patient, Sydney T., who is 65 years old. He was rushed to the emergency room because of trouble with speaking, numbness and loss of movement in his face, vision problems, and confusion understanding simple statements. After completing a medical test, it was determined that Sydney had suffered a cerebrovascular accident (CVA). According to the medical chart, he "has trouble swallowing and suffers from speech and language problems secondary to the CVA." The dietitian is concerned that Sydney has lost weight and is not eating all his meals. The report indicates that "the patient seems depressed and uninterested in eating." The nursing staff indicates that he eats minimally during breakfast, lunch, and dinner. Reportedly, the patient seems to be unable to produce intelligible speech, and the certified nursing assistants are unable to understand him. Sydney's wife indicated that prior to his hospitalization he was an outgoing person and coached high school football. Question As previously stated, Sydney has trouble swallowing. During the modified barium swallowing study, the clinician also noted that he has a delay in the pharyngeal stage of swallowing. The clinician observed that he had a 5-second delay before the pharyngeal swallow was triggered. The clinician noted in the chart that the average duration of a normal pharyngeal swallow is A. 1 second. B. 2 seconds. C. 3 seconds. D. 4 seconds.

A. 1 second.

rocedures in which stuttering is directly reduced without teaching fluency skills (e.g., slow speech) are known as A. direct stuttering reduction methods. B. fluent stuttering. C. fluency shaping. D. desensitization to stuttering.

A. direct stuttering reduction methods.

Single-subject design

A: A phase during which the skills are measured without intervention B: Skills are taught A: Treatment is withdrawn B: Same treatment is reinstated

establishing the baselines of target phoneme productions, then offered treatment to all children who were base rated, withdrew treatment for a period of time, and finally offered the treatment again. This is an example of

ABAB Design

Auditory Rate

Ability to process acoustic stimuli that are presented at different rates or speed

typical signs with TBI

Aggression confuses lang dysarthria hallucinations lack of turn taking

The client has an advanced stage of amyotrophic lateral sclerosis with consequent progressive deterioration of communication abilities.

An augmentative-communication system is the best option for improving or maintaining communication for this client.

In the mand-model method of teaching language skills to children, the clinician does which of the following?

Asks questions like "Tell me what you want" and models or prompts the correct response if necessary

Federal laws regarding freedom of access to information stipulate that client records kept or written by health care professionals can be A.reviewed only by other health-care professionals B.reviewed only by the clients themselves unless the client provides written permission to share with others C.reviewed by anyone who submits a formal written request D.released only by subpoena

B Clients have the right to review their own records.

Successful use of an alternative and augmentative communication system is based on such factors as selecting appropriate vocabulary, seating and positioning, and having a reliable method of controlling the system. To facilitate the most effective use of the system, clinicians most often advocate which of the following approaches? A.Unimodal B.Multimodal C.Bimodal D.Gestural

B Multimodal approach offers more communicative options than any other approach listed

Lesion in the third convolution left cerebral hemisphere damage is where

Broca's area

A child who is 4 years and 4 months old is referred for assessment of speech sound development, with the following results. Sounds in error Error frequency Stopping ​ only on ð 85% Fronting /k, g, ng/ → [t, d, n] 90, 100, & 20% Gliding /r/ → [w] 100% FCD Obstruents- null 80% (correctly - /n, m/) Epenthesis Insert ə in stop /s/ clusters 10% QUESTION: Which of the following phonological error patterns is most appropriate to address in a treatment program? Select all that apply. A. Stopping B. Fronting C. Gliding D. Final consonant deletion E. Epenthesis

B & D Fronting and final consonant deletion should be suppressed earlier, as they make the biggest difference in intelligibility.

Six months ago, an SLP evaluated 4-year-old Molly's speech fluency during conversation. At that time, she displayed physically relaxed repetitions of words and phrases (occurring at a frequency of 2 per 100 words), and interjections such as "um" (occurring at a frequency of 1 per 100 words). She did not display any sound prolongations or facial grimaces; she did not produce any pitch rises or phonatory breaks; and she did not appear to avoid any sounds or words. Results from several formal tests suggested that her articulation and language development were within normal limits. Molly reportedly began producing repetitions and interjections at age 24 months, and the frequency of these disfluency types reportedly has remained stable since then. The SLP did not recommend speech-language intervention following the previous evaluation; however, she did provide the parents with information about fluency development, symptoms of stuttering, and general suggestions for how to facilitate children's fluency. A reevaluation is scheduled for next week. Which of the following is most appropriate for the SLP to do if Molly's speech fluency has remained the same since the previous evaluation? A. Recommend that Molly immediately begin fluency therapy, in which the focus is on reducing the frequency of repetitions and interjections in her conversational speech B. Recommend that Molly be released from the SLP's active caseload C. Recommend that Molly be referred for psychological counseling, with a focus on on helping Molly improve speech fluency by learning how to manage anxiety more effectively D. Recommend monthly evaluations of Molly's speech fluency until she is five years old

B Molly's fluency development was within normal limits at the previous evaluation, and, based on the parents' report, it also seemed to be within normal limits at age 2. Further, no other concerns about Molly's communication development were mentioned in the scenario. Thus, it appears that Molly has never stuttered and her communication skills have been and continue to be within normal limits. Therefore, it is unnecessary to reevaluate her speech or enroll her in fluency therapy.

You are about to begin intervention with a 7-year-old child with developmental delays who reportedly has the language skills of a typically developing 4-year-old child. Which of the following skills would not be appropriate to work in treatment? A. Use of future tense B. Production of gerunds C. Definition of common words D. Topic maintenance over successive utterances

B. Production of gerunds

A two-year-old child produces the following utterances during a play-based conversation with her mother. Utterance: Gloss/Child's Intended Meaning: No kitty It's not a kitty. My doggie This toy dog is mine. No goat It's not a goat. Kitty house The cat is in the house. Big kitty It's a big cat. Doggie ball The dog pushed the ball. QUESTION: Which of the following semantic-syntactic constructions does the child show evidence of using? Select all that apply. A. Disappearance B. Denial C. Possessor + possession D. Attribute + entity E. Agent + action

B, C, D, & E Denial is shown through the utterance of "no kitty," possessor + possession is shown through the utterance of "my doggie," attribute + entity is shown through the utterance of "big kitty," and agent + action is shown through the utterance of "doggie ball."

Which of the following views make up a standard videofluoroscopic swallow study? Select all that apply. A.Frontal B.Lateral C.Transverse D.Anterior-posterior

B, D A lateral view is best for seeing all stages of a swallow, and an anterior-posterior view shows the symmetry (or asymmetry) of the swallow.

You have completed your assessment of a 6-year-old boy's language skills. Reports from such specialists as an audiologist, a pediatrician, and a psychologist are negative, but your assessment results show that the boy's language skills are limited, with marked deficiencies in producing grammatical morphemes. The correct diagnosis you convey to the parents of this child is

B. "Your child has a specific language impairment."

What theory believes that language can be taught by targeting any observable behavior and manipulating the elements of a stimuls, response and reinforcement?

Behavioral theory

What theory is based on conditioning and learning observable and overt behaviors

Behavioral theroay

Weakness, atrophy, fasciculations, and the other described symptoms are all consistent what damage

Brainstem with a lower motor neuron locus

A child exhibits the following production errors. w/r θ/s t/ʃ t/tʃ z/dʒ t/k d/g If a target sound for initial intervention is to be selected on the basis of established developmental norms, then that sound will be A./s/ B./ʃ/ C./k/ D./tʃ/

C According to developmental norms, /k/ is the target phoneme that should be selected for intervention.

Which of the following communication disorders is most frequently associated with significant dysphagia? A. Aphasia B. Ataxic dysarthria C. Flaccid dysarthria D. Organic voice tremor

C Flaccid dysarthria & dysphagia are both disorders likely to be characterized by flaccidity or weakness of the oromotor and laryngeal mechanisms that results from cranial nerve damage. These 2 disorders frequently coexist.

A physician told the spouse of a client that melodic intonation therapy (MIT) would improve the client's speech considerably. The most appropriate next action by the SLP would be to A. provide MIT, as recommended B. tell the physician that it is inappropriate for the physician to make recommendations for a speech treatment C. consider the potential value of incorporating MIT into the client's treatment D. explain MIT to the client's spouse to assist in the decision-making process about the type of therapy to use

C MIT is a procedure appropriately used by SLPs to promote certain results. The SLP has responsibility for determining the value of this procedure in relation to the objectives of the SLP treatment program.

Treatment for apraxia of speech most appropriately emphasizes A. coordination of respiration with phonation and articulation B. auditory discrimination, resonance, and respiration C. auditory-visual stimulation, oral-motor repetition, and phonetic placement D. rate of speech, range of movement, strength, and coordination of the oral mechanism

C Treatment for AOS appropriately emphasizes auditory-visual stimulation, oral-motor repetition, and phonetic placement.

You have been asked to give an in-service to a group of students who wish to eventually specialize in service delivery to children with cleft palates and their families. The students want to know detailed information about in utero development of the hard and soft palates (among other things). You can accurately tell them that in utero the hard palate fuses between which developmental ages? A. 1-2 weeks B. 4-6 weeks C. 8-9 weeks D. 10-12 weeks

C. 8-9 weeks

You are treating a 4-year-old boy with specific language impairment (SLI) for intervention. You notice that he omits all grammatical morphemes in his speech. Which one of the following morphemes would you target first in therapy with him? A. Articles a, an, the B. Regular plural -s C. Present progressive -ing D. Regular past tense -ed

C. Present progressive -ing

A teacher has referred Jeremy to you for an evaluation. Jeremy is an African American first grader who is reportedly doing well in class academically. When you observe him on the playground with his peers, you see that he has many friends and does not appear to have problems interacting appropriately with other children. His friends do not appear to have any difficulty understanding what he says. However, the teacher is concerned. She says, "I think Jeremy pronounces some of his sounds wrong. I think he needs speech therapy." When you conduct a speech screening with Jeremy, you will remember that which one of the following patterns indicates a possible disorder, not a difference, based on Jeremy's use of African American English (AAE)? A. f/th substitution in word-final position B. Production of [ks] instead of [sk] C. th/s substitution in all word positions D. Differing stress on some words (e.g., police instead of police)

C. th/s substitution in all word positions

A professor is teaching a class anatomy and physiology and mentions that closure of the airway occurs at anatomically different locations and functionally separate levels in the larynx. A student in the class answers that the appropriate choice would be A. vallecula. B. pyriform sinus. C. the supraglottic portion of the laryngeal vestibule. D. cricopharyngeus muscle.

C. the supraglottic portion of the laryngeal vestibule.

In your private practice, you receive a referral of a sixth-grade girl who is not speaking; the pediatrician thinks that she might have functional aphonia. A diagnosis of functional aphonia means that A. there is evidence of neurological damage. B. the condition is not psychogenic. C. there is no voice. D. the larynx is damaged.

C. there is no voice.

what assess the prep phase of the swallow using the posterior and lateral plane examinations

manometric asessment

Damage to cranial nerve VII will cause what?

mask like appearance

A problem you might expect with children who come from homes containing neglect, abuse, or both

mothers reluctance to engage in reciprocal interactions with their infants

Trigeminal nerve

motor component: involved in mastication sensory: sensory info from entire face

Facial nerve

motor: innervates muscles in lower face especially orbicularis oris and buccinator

Which of the following sets of minimal pair words best targets the phonological patterns of stopping of fricatives? A.Sun and ton B.Sip and ship C.Star and tar D.Shoe and shoot

Correct Answer: A Option (A) is correct. "Sun" and "ton" are minimal pair words, and substituting /t/forward slash t forward slash for /s/forward slash s forward slash is an example of the phonological process of stopping of fricatives.

Which of the following best distinguishes a dialect from an accent? A.Differences in language as well as in pronunciation B.Differences in pronunciation only C.Differences due to the influence of a second language D.Differences that are unique to a particular speaker

Correct Answer: A Option (A) is correct. A dialect involves differences in language and pronunciation

Cognitive therapy for stuttering focuses on

Changing distorted beliefs about self-efficacy and the need to speak with complete fluency

Which of the following techniques is most effective when treating phonation in a patient with spastic dysarthria? A.Completing head and neck relaxation exercises B.Engaging in lip-stretching exercises C.Providing instruction in phonetic placement D.Working on pitch-range exercises

Correct Answer: A Option (A) is correct. Head and neck relaxation exercises assist in reducing hyperadduction of the vocal folds.

Oropharyngeal dysphagia in a child with Down syndrome is most likely caused by which of the following factors? A.Hypotonia B.Digestive problems C.Pneumonia D.Aversive feeding behaviors

Correct Answer: A Option (A) is correct. Hypotonia is the leading cause of swallowing disorders in children with Down syndrome.

Inhalation during respiration is primarily carried out through the movement of which of the following muscles or muscle groups? A.Diaphragm B.Internal intercostals C.External intercostals D.Scalenes

Correct Answer: A Option (A) is correct. The diaphragm is the muscle that creates the most volume in the lungs.

As you evaluate the language of an eighth-grade boy, Derek, and listen to him talk about his hobbies and interests, he says things such as "I like to play football, and I also like Mario Cart Wii" and "My football team won the championship last Saturday; later, we celebrated at a pizza place." What has Derek just used?

Compound sentences containing two independent clauses

What is 1. variable fluency , generally less fluent than wernicke's aphasia 2. marked word finding problems 3. good syntax, prosody, articulation 4. lesion sites controversial

Conduction aphasia

What effect is called when stuttering recurs on certain words on repeated oral readings?

Consistency effect

Hard driving personality glottal fly uses voice extensively in daily life Sores, crater

Contact ulcers

A 92-year-old patient with dementia is experiencing severe oral-and-pharyngeal-stage dysphagia including aspiration. The clinician would like to teach the patient a swallowing therapy technique, as the family is concerned that he is losing weight. The patient is currently being tube fed and is monitored closely by the hospital staff and dietitian. Ethically, which is the only appropriate technique that the clinician should consider?

Continue with the tube feeding.

What are small and cone-shaped and sit at the apex of the arytenoids?

Corniculates

Which of the following conditions is primarily characterized by premature closure of the sutures of the skull? A.Craniosynostosis B.Craniopharyngioma C.Deformational plagiocephaly D.Positional plagiocephaly

Correct Answer: A Option (A) is correct. Craniosynostosis is a premature closure of one or more of the cranial sutures of the skull and can be associated with a cleft palate.

Which of the following muscles is responsible for changing vocal pitch? A.Posterior cricoarytenoid B.Thyroarytenoid C.Lateral cricoarytenoid D.Cricothyroid

Correct Answer: D Option (D) is correct. The cricothyroid is responsible for changes in vocal fold pitch as it tenses the vocal folds.

What syndome? Young boy whose symptoms include low birth weight, microcephaly, low set ears, micrognathia, wide eyes, excessive drooling agression CAT LIKE MEOWING

Cri du chat

What cartilages are cone shapes and located under the mucous membrane that cover the aryepiglottic folds

Cuneiforms

To justify providing individual treatment for a 2½ year old with apraxia of speech, which of the following would be LEAST important for the SLP to include in the evaluation report? A.A description of the child's typical interaction with peers B.Relevant prognostic data C.Information about apraxia of speech D.A description of the language development of the child's older siblings

D A description of the child's older siblings' language development does not provide the objective, documented evidence required to justify provision of treatment of a child of an age at which some unintelligibility would be typical.

In terms of communication impairment, an adult with Alzheimer's-type dementia tends to differ from an adult with aphasia associated with a CVA in that A. repetition abilities are typically more seriously impaired in the adult with dementia B. the adult with dementia generally experiences greater dysfluency C. the adult with dementia is generally more aware of any disruptions in communication D. the capacity of the adult with aphasia to make appropriate comments about recent events is generally better

D Adults with dementia typically have impaired short-term memory abilities. Adults with aphasia are more likely to recall recent events and consequently would be more likely to make appropriate comments about recent events when compared to those with dementia.

Laborious, halting, telegraphic utterances are typical of clients with which of the following types of aphasia? A. Conduction B. Anomic C. Wernicke D. Transcortical motor

D Transcortical motor aphasia is characterized by dysfluent, telegraphic utterances

Ms. Lopez's articulation errors consist of the following: f/v, ʃ/ʒ, and s/z. On the basis of these errors, the SLP should begin remediation that focuses on A. manner B. place C. fricatives D. voicing

D Ms. Lopez's phonological errors are errors of voicing

For a patient with communication and swallowing disorders secondary to acquired immunodeficiency syndrome (AIDS), a speech-language pathologist will most appropriately A. serve only as a consultant to others who are directly involved in the patient's care B. provide treatment on a monthly basis C. provide treatment only as prescribed by the patient's physician D. provide treatment in consultation with the patient's primary-care physician or medical team

D Treatment in consultation with the patient's primary-care physician or medical team would provide the information needed to determine the best management of the patient.

A child repeatedly inserts an inappropriate sound in certain environments; for example, [fpɪʃ] for [fɪʃ] . Which of the following would likely be most helpful for the child as a target for treatment? A. Bisyllabic words for which a minimal-contrast pair can be easily identified B. Words containing phonemes that have distinctive features in common with the sound the child inserts inappropriately C. Repeated practice with the combinations of phonemes that the child finds particularly easy to produce D. Words that contrast the child's error pattern with the target pattern in the word

D Words that contrast the child's error pattern with the target pattern would be most helpful for this child.

Ms. Helene, a 60-year-old woman with a suspected neurological disorder, is referred for speech-language evaluation. She achieves a score of 35/50 on a measure of confrontation object-naming ability. This score is below norms established for individuals of her age and educational level. Based on these results alone, which of the following statements can most reliably be made about Ms. Helene's disorder? A. She has anomic aphasia, which might or might not be associated with other language deficits. B. She has aphasia, but the type of aphasia cannot be specified on the basis of this test score alone. C. She does not have aphasia but probably does have a memory disturbance. D. She has difficulty with naming, but the precise nature of the deficit cannot be determined on the basis of this test score alone.

D Ms. Helene has a suspected neurological disorder and she achieves a score that is below the norms for individuals her age. Given this information alone, the most that can be said is that she has difficulty with naming; the precise nature of her deficit cannot be determined

What is a pink or white wart-like growth that can be found anywhere in the airway and make a person's voice sound hoarse, breathy, and low pitched? A. Hyperkeratosis B. Leukoplakia C. Hemangioma D. Papilloma

D. Papilloma

A Cambodian child is referred to you because the teacher "can't understand a word he says." In the course of your speech-language screening, you record the following utterances. Which might be typical of an articulation disorder, not a difference? A. "The funny circus crown had red nose." B. "I won't ste on your toes." C. "Please hand me the block [black] one." D. "I am derry [very] appy [happy] to meet your tids [kids]."

D. "I am derry [very] appy [happy] to meet your tids [kids]."

What is the most researched procedure and is useful establishing skills but may not promote generalization to natural settings?

Discrete Trial Procedures

motor speech disorder characterized by slowness, weakness, incoordination, or altered tone of the speech production mechanism

Dysarthria

Where do hearing aids amplify sounds to?

Ear canal

What stuttering method states stutter more fluency teaches identification desensitizes client modify stutter cancellation, pullouts, prep sets stablize and counsel not normal fluency more more fluent stuttering Van riper

Fluency stuttering method

In a trach, an inflated tube may do what to laryngeal elevation

May restrict laryngeal elevation

What disease causes fluctutiating senurineural hearing loss includes vertigo sense of fullness in the ear tinnitus

Menieres disease

What theory alternative to account for influence of stress and tone features in levels of representation of segmental or linear representatiion

Non linear Theory

According to ASHA and the Joint Committee on Infant Hearing (JCIHJ C I H) recommendations of 2007, for babies who fail the newborn hearing screening, the follow-up diagnostic audiologic evaluation should be completed no later than A.3 months of age B.6 months of age C.9 months of age D.12 months of age

Option (A) is correct. The JCIHJ C I H (2007) recommends a time line that includes a hearing screening to be completed no later than 1 month of age. For those newborns who fail the newborn hearing screening, the diagnostic audiologic evaluation should be completed as soon as possible, but no later than when the child is 3 months of age.

In preparing for feeding and swallowing with a patient with a tracheostomy tube, which of the following is recommended while conducting therapy?

Occlude the patient's tracheostomy during and immediately after the swallow, as the exhalatory airflow after the swallow may contribute to clearance of residual food from the top of the airway, reducing the chance of aspiration after the swallow.

When is stuttering more likely to occur?

On words that begin with consonants

Production of phoneme in isolation is used to teach

PHONETIC PLACEMENT

Which of the following factors contributes to UES opening?

Partial relaxation of the cricopharyngeal portion of the inferior constrictor muscle Superior and anterior hyolaryngeal excursion

What muscle produces opposing action that producees velopharyngeal closure?

Palatoglossus

Which muscles help with velopharyngeal closure?

Palatoglossus, tensor veli palatini, levator veli palatini

What is a shaping technique that focuses on orofacial and articulatory postures with specific instructions about how to change current speech and non-speech movements to achieve target sounds

Phonetic derivation

The parents of a 10-year-old child consult a clinician. The child is short and has intellectual delay, cryptorchidism, and sleeps excessively. He also has a speech delay, hyperphagia, and is obese. The clinician notices that the child has a prominent nasal bridge, high, narrow forehead, thin upper lip, downturned mouth, almond-shaped eyes, and small hands and feet. The parents report that he frequently picks his skin. They report that the genetic counselor informed them that his condition is caused by autosomal dominant inheritance and deletion in the long arm of chromosome 15 (15q11015q13) in some cases. The child has

Prader-Willi syndrome.

What connects the cortex and subcortical structures?

Projection fibers

The chewing technique is used to reduce

muscular tension in the laryngeal area.

possessor + possession is shown through the utterance of "my doggie

my doggie

What is it called when there is a small incision into the ear drum to relieve pressure or drain fluid?

myringotomy

Tilting your head back and swallowing bolus will result in

narrowing or closure of the vallecula space

A chin tuck will result in

narrowing the airway entrance, pushes epiglottis posteriorly

difficulty adducting vocal folds, damage is to the

Recurrent Laryngeal nerve

What a child repeats a patten wawa for water

Reduplication

What is a therapy that discussing -past experiences and events of life -photographs of patient's family -music that she likes

Reminiscence therapy

single subject design

Repeated measures of behaviors before a treatment is administered Intensive study of a few individuals Absence of a control group

When at risk students are struggling in classroom and given increasing amounts of targeted small group support, it is called

Response to Intervention

What hemisphere specializes in visual and spatial information?

Right hemisphere

40-year-old woman who talks to herself, uses confused language (although the speech is mostly grammatical), and describes events and experiences that may be unreal.

Schizophrenia

What method should be considered to represent language in AAC

Semantic compaction

What is phonetic derivation?

Shaping technique that focuses on orofacial and articulatory postures with specific instructions about how to change current speech and non-speech movements to achieve target sounds

What are ideographic or pictographic symbols based on ASL and often used in conjunction with ASL?

Sig symbols

What is the same as an articulatory posture without voicing?

Silent prolongation

Denial is shown through the utterance of "no kitty,"

no kitty

Vocal Nodules

nodes on vocal folds protrude from surrounding cells, reddish or pinkish in the beginning and over time appear white or gray because of fiber

simple harmonic motion is

back and forth movement of symmetrical and periodic

What are pretreatment measures

baselines

Single-subject design using ABAB or ABA design is the

best method to determine whether intervention has succeeded.

attribute + entity is shown through the utterance of "big kitty

big kitty

place of articulation examples

bilabial, labiodental, interdental, alveolar, palatal,velar

paralysis of two vocal folds, may cause aphonia and wide open glottis

bilateral paralysis

You are working with a mother who wishes to reduce her son's whining behavior. You have determined that because of his limited oral language skills, he whines to get what he wants. In the clinic, you have established a few mands the child can reliably produce (e.g., "I want [juice, water, candy, cookie, car, etc.]"). Now you are asking the mother not to respond to whining but instead to prompt the verbal mands and immediately give what the child mands. The mother reports success with the procedure. This procedure is known as

differential reinforcement of alternative behaviors (DRA).

Recent research has highlighted findings regarding the linguistic and cognitive development of internationally adopted children. You are summarizing this research for a group of parents to present to them in a workshop. Which one of the following statements is the MOST accurate?

for many children, after they arrive in the United States, receptive English language skills will increase more rapidly than expressive English language skills

As a clinician, you are concerned with using the most appropriate assessment approach that suits your clients, avoids false positive or false negative diagnoses, helps generate treatment goals, and is fair to clients of all ethnocultural backgrounds, including mainstream clients. To accomplish this assessment goal, you would select which of the following approaches? A. The portfolio approach B. The authentic approach C. A criterion-referenced approach D. An integrated approach

d. An integrated approach

Cochlear implants deliver electrical impulses

directly to auditory nerve

Neurogenic stuttering distinguishing features

disfluencies on function words and in imitated speech, lack of adaptation, minimal or no effect of masking noise

A weak cricopharyngeus causing difficulties in passing the bolus through cricopharyngeus muscle and past the 7th cervical vertebra

disorder in the esophageal phase of the swallow

Entity + locative

dolly bed juice glass

You find that a child you are evaluating uses a number of phonological processes. One of those processes is stopping. You know this when you hear the child make what substitutions?

du/zoo

otolaryngologist is a

ear, nose, throat doctor

A researcher places surface electrodes on both sides of a patient's thyroid cartilage and passes a high-frequency electric current between the electrodes.

electroglottography.

What assessment can be conducted by attaching electrodes to structures?

electromyographic

What INVASIVE procedure that directly measures laryngeal function to study the pattern of electrical activity of the vocal folds and view muscle activity pattern

electromyography

if and then statements in piagets stage

formal operations

in speech language sampling

frequnetly repeat what the child says

A physician refers a 50-year-old male patient with dementia to you for assessment and treatment. The referring physician suggests the strong possibility of dense intracellular formation in the neuronal cytoplasm and ballooned and inflated neurons. Your assessment reveals that the patient has had a progressive loss of vocabulary, paraphasia, circumlocution, and dominant language problems, with somewhat better preserved memory and orientation. The most likely diagnosis you would make on this patient is

frontotemporal dementia associated with Pick's disease.

Patient experiences heartburn, acid indigestion, gastric contents are emptying into esophagus

gastroesphageal reflux

When a person produces voiced and voiceless /th/, muscle most involved is

genioglossus

Conduction aphasia is characterized by

good syntax, prosody, and articulation

What design are 1. effective in establishing internal validity 2.cause-effect relationships 3.randomization may be difficult to meet

group design

What non-nutritive sucking?

infant has difficulty brining her hands to her mouth to inititate sucking during breast feeding, infant has bursts and pauses about two up and down cycles of the jaw per second Rhythmic interval sucking for comfort resulting in no fluid release, typically on a finger or pacifie

Hypoglossal CN XII

innervates tongue muslces

what practice uses simultaneous practice to bring consciousness the look and sound of target speech?

integral practice

Laryngeal Web

membrane that grown across anteior portion of the glottis congentital or acquired

statistical method involving the integration of data across studies to form general conclusions.

meta analysis

A speech-language pathology graduate student received her first adult patient diagnosed with apraxia of speech. The student consulted with the supervisor, who recommended that they try to maintain the natural prosody of the patient's utterances. She suggested using computer-generated pacing tones each time the patient produced an utterance, so that the patient could keep the natural rhythm of the utterance without regard to the rate of speech. The student clinician conducted further research and during therapy coupled hand-tapping and choral reading along with the suggestions made by the supervisor. This type of therapy is called

metrical pacing.

clinician instructed the patient to produce speech at one syllable per beat. The beat was set at a slower rate than the patient's actual speaking rate. Hand-tapping was also used simultaneously with this treatment technique.

metronomic pacing.

Children with specific language impairments typically have difficulty producing utterances that are

morphologically and syntactically well formed.

Discrete trial procedures

most research procedure and useful establishing skills but may not promote generalization to natural settings

Spongy growth that starts on the footplate of the stapes and causes it to become rigid is known as calcium buildup

otosclerosis

Ultrasound examination measures what

oral tongue movement and hyoid movement

Cohesion is

ordering and organizing utterances in a message so that they build logically on one another

when patient says that true vocal folds adduct instead of abduct during inhalation vocal folds remain close throughout the respiratory cycle

paradoxical vocal fold

when there is inappropriate closure or adduction of true vocal folds during inhlation or exhalation is called

paradoxical vocal fold

Patient receives irradiation to oral and pharyngeal, experienced xerostomia, weight loss and an increase in dental caries

patient should be given synthetic salvia prior to eating

Direct laryngoscopy

performed by a surgeon when the patient is under anethesia -patient cannot phonate

Waves that repeat themselves at regular intervals

periodic waves

What is a secondary palatal surgery surgical procedure in which a muscular flap is cut and attached to the velum flap closes velo pharynegeal port and reduces hypernasality

pharyngeal flap surgery

Rhyming, phoneme isolation, and sound blending are all a part of

phonological awareness treatment

What therapy technique of phonetic placement is used to teach or establish

production of phoneme in isolation

The Mendelsohn maneuver

prolong the duration and diameter of upper esophageal sphincter opening.

Tensor palatini does what during yawning and swallowing

pulls that eustachain tube to open

Action + object

push truck

In using AAC, considering response efficiency involves

quality, rate, immediacy of reinforcement as well as response effort

For reliable measure of child's language skills through language sampling, what should u do?

repeat the language sample

Acoustic measurements of voice are becoming extremely popular, especially as a means of evaluating the effectiveness of voice therapy. Sound spectography, the graphic representation of a sound wave's intensity and frequency as a function of time, yields a spectrogram or picture that reflects:

resonant characteristics of the vocal tract and the harmonic nature of the glottal sound source

You are asked to prepare a clinical management plan for family members of a patient with dementia. Among other skills, you would teach the family members to

restate important information

What is it called when there is damage to nerve fibers along the ascending auditory pathways from the internal auditory meatus to the cortex

retrocochlear disorder

Functional aphona means

there is no voice

Repression

thoughts and feelings are under strict control, not conscious of them

Supression

thoughts and feelings under controlled and ARE conscious of them

Reaction Formation

thoughts that are shocking/unacceptable are reacted with opposite emotions

What is a stationary blood clot that blocks the flow of blood

thrombus

Hypernasality components

velopharyngeal port does not close opening to the nasal passage during production of non nasal sounds trouble with non nasal sounds decreased/insuffiecnt intraoral breath pressure affects fricatives/afficates

Ankyloglossia is another word for

tongue tie

A 45-year-old woman was complaining of difficulty masticating. She had tonic spasms after irradiation. The physician explained to the speech-language pathologist that this condition is believed to exist secondary to fibrosis of the muscles involved in mastication. This condition is called

trismus.

oral tongue movement and hyoid movement is measured by

ultrasound examination

one vocal fold is paralyze and assumes static position

unilateral paralysis

Cover-body theory of phonation states

-The epithelium, the superficial layer of the lamina propria and much of the intermediate layer of the lamina propria vibrate as a "cover" that is made up of the remainder of the intermediate layer, the deep layer, and the TA muscle

Medically fragile babies have feeding difficulties. Name some components they would have

-problems with oral-motor development -often need to be fed through nasogastric tubes -to be fed orally, they must be at least 35 weeks old

Autism behaviors

-stereotypic body movements -insistence on routine -disinterest in interaction with others -echolalia -hypersensitivity to touch

Cricoid cartilage

-uppermost tracheal ring -second largest -laryngeal cartilage

A patient with end-stage cancer was experiencing loss of weight, fatigue, weakness, and a significant reduction in appetite. The oncologist mentioned to the speech-language pathologist that even though the patient was not actively trying to lose weight, his loss of body mass would be difficult to reverse nutritionally because he had aggressive cancer. This condition of weight loss that is a common complication associated with cancer is called

. cachexia.

Components of veterbral column consist of?

7 cervicial, 12 thoracic, 5 lumbar, 5 sacral and 3-4 coccygeal (fused) vetebrae

Brutten and Shoemaker hypothesized that stuttering, as they defined it

A. is caused by classically conditioned negative emotion.

limitation of employing imitation as an intervention strategy

A.lacks communicative intention

What is it called when Responding to both the content and the affect of the client's remarks?

Active listening

What is it called when you have the ability to mentally sort speech stimuli or remember what one has heard

Auditory memory

Which of the following errors is likely to persist the longest in the speech of children who are learning Standard American English (SAE) as a first language and are following the normal developmental course for speech and language acquisition? A. Assimilation B. Consonant cluster reduction C. Final-consonant deletion D. Velar fronting

B Consonant cluster reduction is the most persistent of the normal developmental processes listed.

A child being assessed for a possible language impairment says to you, "My dad put my shoes on my feet before he drove me to school." This sentence has A. 14 words, 14 morphemes B. 14 words, 15 morphemes C. 14 words, 16 morphemes D. 14 words, 17 morphemes

B. 14 words, 15 morphemes

A child who calls all tall and brown-haired men "Daddy" is exhibiting the phenomenon of A. underextension. B. overextension. C. joint reference. D. denial.

B. overextension.

Control over the fundamental frequency of the laryngeal tone is most closely related to the activity of which of the following muscles? A. Posterior cricoarytenoid B. Lateral cricoarytenoid C. Cricothyroid D. Sternocleidomastoid

C The cricothyroid muscle has the greatest control over the fundamental frequency of the laryngeal tone by lengthening or tensing the vocal folds.

A 6-year-old child produces [t] for /s/, [d] for /z/, [p] for /f/, and [b] for /v/. Intervention for this problem would target language at the level of A. morphology B. syntax C. phonology D. semantics

C The speech patterns described in the question stem all pertain to speech sound production. Phonology is the aspect of language that deals with speech sound production.

Which of the following muscles produces the opposing action to those that produce velopharyngeal closure? A. Musculus uvulae B. Levator veli palatini C. Palatoglossus D. Stylopharyngeus

C Velopharyngeal closure is largely produced by soft palate elevation, and the only muscle in the list that produces soft palate depression (opposite of soft palate elevation)

What stimulation targets general improvement in social situations? relaxing or listening to music theme based activity, small group FOR DEMENTIA

Cognitive stiumulation therapy

What test assesses auditory, visual, motor, and oral coomunication and arousal

Coma recovery sacle

Which of the following instrumental assessment tools provides the most direct dynamic view of velopharyngeal movement during speech? A.Nasopharyngoscopy B.Lateral-view x-ray C.Nasometry measurements D.Aerodynamics

Correct Answer: A Option (A) is correct. Nasopharyngoscopy is an example of a direct instrumental assessment tool that provides a dynamic (moving) view of the velopharyngeal movements

In Bloom and Lahey's model, morphology is considered A.form B.content C.use D.semantics

Correct Answer: A Option (A) is correct. The Bloom and Lahey model classifies morphology as form.

Dysarthria in a patient with lower motor neuron damage generally manifests as A.spastic dysarthria B.flaccid dysarthria C.ataxic dysarthria D.hyperkinetic dysarthria

Correct Answer: B Option (B) is correct. Flaccid dysarthria is the only choice that correlates to lower motor neuron damage.

Which of the following approaches is the SLP's most appropriate response to the resident psychiatrist's recommendation? A.Discharging the order for the new evaluation because therapy has already started B.Working collaboratively to further address the patient's aphonia C.Sending treatment recommendations to the psychiatrist D.Telling the patient that further speech therapy is inappropriate because of his psychological diagnosis

Correct Answer: B Option (B) is correct. Involving the professional who is treating the psychological piece of the disorder will hopefully lead to success for the patient.

A 6-year-old child has difficulty producing both regular and irregular plural forms. Intervention for this problem would best target language at the level of A.syntax B.phonology C.morphology D.semantics

Correct Answer: C Option (C) is correct. Regular and irregular plural forms are morphological markers.

A 12-year-old native speaker of Spanish who has been studying English as a second language for three years is most likely to do which of the following when speaking English in casual conversation with teachers at school? A.Use the auxiliary "have" in place of "be" in progressive tenses B.Use incorrect word order within prepositional phrases C.Use conjunctions in place of prepositions D.Use multiple negation improperly

D Multiple negation is a grammatical feature of Spanish but not of Standard English. The contrast between the 2 languages can cause multiple negation to persist as a speaker attempts to learn Standard English.

What is it when the clinican repeatedly models a target structure to stimulate the child to use it wants the child to use plural s there are two pigs here, i see two pigs

Focused stimulation

displays ability to think and speak in the abstact in piaget stage occurs in

Formal operations more than 11 years

Action + locative

IN bed

What is it called when the patient impounds the air in the mouth. Then impounded air is pushed back into the esophagus and expelled belch into speech

Injection method

A clinician is measuring communicative behaviors in a child with a cleft palate. The clinician measures the time intervals during which the speech behaviors selected for observation occurred. What are the methods of measurement called?

Time sampling measures

Rarefaction

when vibrating objects return to equilibrium, air molecules become thinner

Components of AAE

1) Influenced by languages of West Africa 2) If children speak AAE, they can become bidialectal through learning to use both AAE and Standard American English 3) If children speak AAE, it is best to assess their language using alternate forms of assessment such as language sampling

A clinician who employs active listening is doing which of the following? A.Responding to both the content and the affect of the client's remarks B.Listening very carefully and taking extensive notes C.Conducting a clinician-directed interview D.Directing the client to specific answers to questions

A A clinician who employs active listening responds to both the content (the denotative message) and the affect (the emotional content) of the client's remarks.

A single exposure of several hours duration to continuous music with an overall level of 100 dB SPL will most likely produce A.tinnitus and a temporary threshold shift in high frequencies B.tinnitus and a distortion of speech perception C.a temporary threshold shift in the low frequencies D.a permanent threshold shift

A A single exposure of several hours duration to continuous music @ level of about 100 dB SPL will most likely produce tinnitus & a temporary threshold shift in high frequencies.

Questions 64-67 refer to the following. Michael is a 32-month-old boy who has been receiving early intervention services over the past ten months for delayed speech and expressive-language development. Although his birth was reportedly unremarkable, Michael does have a history of recurrent otitis media with effusion. His parents described him as having been a "well-behaved and quiet baby." When Michael began receiving services, he communicated mainly through gestures and crude vocalizations. An open resting mouth position with slight tongue protrusion was sometimes noted. However, his receptive-language skills were found to be age appropriate and he showed no oral motor deficits during feeding. Michael's expressive-language skills have shown some progress since he began working with the speech-language pathologist, but he remains poorly intelligible. Michael's imitation of tongue, lip, and jaw movements is characterized by inconsistent groping and errors of sequencing not observed in his spontaneous oral movements. Michael has an age-appropriate vocabulary and produces utterances of up to five words. Articulation errors, especially metathesis of phones and syllables, increase as his utterance length increases. Michael's intelligibility is greatest at the single-word level. Automatic speech and highly familiar utterances are much more intelligible than his imitated productions. QUESTION: Michael demonstrates speech behaviors that are typical of children diagnosed with which of the following? A. Childhood apraxia of speech B. Conductive hearing loss C. Hyperkinetic dysarthria D. Autism spectrum disorder

A Articulation characterized by groping, inconsistency, and errors of sound and syllable sequencing strongly suggests developmental apraxia of speech.

For which of the following reasons would the therapy for a client whose language loss is due to brain injury differ from the therapy for a client whose language loss is due to a progressive disorder? A. There is a much higher chance of spontaneous recovery for the client with brain injury. B. There is a much higher chance of spontaneous recovery for the client with a progressive disorder. C. The client with brain injury typically has problems with fatigue and mental confusion, whereas the client with a progressive disorder does not. D. The client with a progressive disorder typically has problems with attention and memory, whereas the client with brain injury typically does not.

A Brain injury is typically characterized by some degree of spontaneous improvement over a period of several days to weeks, which may be evidenced through gradual improvement in language abilities, whereas a progressive disorder is characterized by gradual loss/reduction in language skills.

Children diagnosed as having specific language impairments are likely to exhibit the greatest deficits in which of the following? A.Production of sentences with appropriate inflectional morphology and syntax B.Acquisition of word meanings C.Comprehension of short sentences D.Motoric aspects of written expression

A Children with SLI typically have difficulty producing utterances that are morphologically and syntactically well formed.

A correct-response rate of 51 percent on a two-choice picture-pointing task would most likely indicate which of the following? A. A random pointing response B. Successful intervention C. Development of crucial discrimination skills by the client D. Readiness to progress to a three-picture point task

A If an individual is responding randomly on a two-choice task, then there should be no significant preponderance of correct responses over incorrect responses. A result of 51 percent represents such an expected pattern of random responses.

A 9-month-old child was observed during a speech-language evaluation. To express herself, the child occasionally touched her mother, gained eye contact, and then gestured toward an object. If the child's development is normal, within the next month or so the child will begin to A. use consistent sound and intonation patterns as signals for specific intentions B. reach for objects that she desires C. establish joint reference with her caretaker D. use recognizable words and phrases to express her intentions

A In normal development, a child begins to use vocalizations to express specific intentions at around 10 months of age.

Before an SLP initiates voice intervention, it is most important that the SLP gather information about the A.cause of the client's voice problem B.client's breathing patterns C.client's oral motor skills D.fundamental frequency of the client's laryngeal tone

A It is essential to know the cause of a voice problem prior to commencing voice intervention, because some voice problems are symptoms of diseases/disorders (e.g., laryngeal cancer) that do not respond to the types of voice interventions that an SLP typically provides, & can only be treated through medical intervention.

A public-school-based speech-language pathologist is employed in a state that sets the maximum caseload at 65. However, the clinician's caseload is currently at 64 with a waiting list of 10 additional students. The school principal insists that the speech-language pathologist enroll the 10 students immediately, because the district cannot locate another clinician to assist with the caseload. Which of the following is the most appropriate way for the speech-language pathologist to address the situation? A.Enroll 1 of the 10 students and provide the principal with a written statement of caseload needs, mentioning the amount, type, and frequency of treatment B.Refer the 10 students to a speech-language pathologist working in a private setting C.Suggest that the principal ask a school speech-language pathologist from another school district to take the 10 students D.Maintain current caseload until an additional speech-language pathologist is hired

A It provides the principal with a statement detailing the needs of the students awaiting treatment but also observes the state's caseload limit.

Naturalistic teaching chiefly involves which of the following? A. Establishing successful and useful communication B. Using multiple trials and training techniques C. Using more adult-initiated interactions than child-initiated interactions D. Using differential reinforcement, fading, and modeling

A Naturalistic teaching focuses on the successful production of utterances that are useful in context for communicating.

Which of the following treatment goals related to pragmatics best addresses a young child's use of language? A. In a play situation, the child will request a turn, either verbally or through gestures, 90 percent of the time. B. In a treatment session, the child will produce a two-word combination 90 percent of the time. C. In conversation, the child will produce correct velars 90 percent of the time. D. When telling a story, the child will use the standard dialect's irregular past tense forms of five specific verbs 90 percent of the time.

A Pragmatics is the study of language in realistic contexts, and the treatment goal describes a real-life, naturalistic use of communication skills.

Which of the following indicates the goal of the Health Insurance Portability and Accountability Act (HIPAA) ? A. To ensure a patient's privacy and confidentiality of health-care information B. To maximize health insurance coverage for speech-language pathology services C. To ensure a free and appropriate education D. To ensure a patient's knowledge of health insurance

A Protecting patient privacy & confidentially of health-care info is main purpose of HIPAA

Immediately following removal of a benign tumor from the base of the brain, a 76-year-old client exhibits severe nasalization and a weak, breathy voice. A four-month postsurgical assessment reveals no improvement. At this time, the remediation strategy for this client should focus on A.evaluation for prosthetic or surgical intervention B.strengthening exercises for the oral articulators C.a trial period using the yawn-sign technique D.complete vocal rest

A The client has a resonance and phonation disorder indicative of velopharyngeal & laryngeal problems. The VP problem could be assisted by prosthetic or surgical intervention.

A client with anomic aphasia is a native speaker of Spanish with fair proficiency in English. Production of the word "shoes" as [tʃuz] on a repetition task is most likely due to which of the following factors? A.Phonological interference from the speaker's native language B.Semantic interference from the speaker's native language C.Semantic paraphasia due to the aphasia D.Phonemic paraphasia due to the aphasia

A The spanish language does not have /ʃ/ in its phonemic inventory and native speakers of Spanish typically substitute the affricate /tʃ/ when producing English words with that phoneme. A native speaker of Spanish would most likely have made the substitution before acquiring an anomic aphasia.

After sustaining a CVA, Ms. Williams, age 75, was referred to an SLP for a speech and language evaluation. While Ms. Williams was describing the cookie-theft picture, the SLP observed that her grammatical structure appeared to be intact and her prosody was normal but that many of her sentences were meaningless, did not fit the context, and included nonsensical paraphasic errors. Additional testing also revealed that Ms. Williams exhibited poor repetition and naming skills, did not respond appropriately to many simple commands, and had difficulty reading. Ms. Williams appeared happy and talked excessively. She did not appear to be aware of her communication deficits. What is the most likely location of the lesion? A.Left posterior superior temporal gyrus B.Left inferior frontal gyrus C.Left superior frontal gyrus D.Left inferior parietal gyrus

A The symptoms that are described in this scenario are consistent with damage to the left posterior superior temporal gyrus

Native speakers of a language possess several capacities or abilities that provide insight into their language competence. Sentences such as "Visiting friends can be a nuisance" are especially useful to test a person's ability to A. recognize syntactic ambiguity B. interpret metaphoric language C. distinguish homophones by means of syntactic cues D. recognize a semantically anomalous sentence

A There are two ways to parse or interpret the structure of the sentence "Visiting friends can be a nuisance." One interpretation is that it is bothersome to visit friends; the other is that friends who have come for a visit are bothersome. Therefore, this sentence would be appropriate as part of a test of a person's ability to recognize syntactic ambiguity.

Language impairment in a child with Down syndrome is often determined by comparing performance on one or more standardized language tests with the child's mental age, rather than with the child's chronological age. Although mental age should not be used to specify the need for treatment, mental age can legitimately be used as a performance criterion because A. using chronological age would overidentify language disorders B. using chronological age would underidentify language disorders C. mental age always correlates with verbal performance D. language performance is expected to exceed mental age

A Using chronological age as a performance criterion for a child with DS would make the child's language disorders seem more pronounced than they are.

A client exhibits weakness, atrophy, and fasciculations of the right side of the tongue and lower face. The client also has right vocal-fold weakness and nasal regurgitation of fluid when swallowing. These problems are the result of damage to which part of the nervous system? A.Brain stem B.Cerebellum C.Left cerebral cortex D.Right cerebral cortex

A Weakness, atrophy, fasciculations, and the other describes symptoms are all consistent with a lower motor neuron locus and suspected cranial nerve abnormalities (primarily CN X and CN XII). These cranial nerves emerge directly from the brain stem and help mediate the transfer of messages from the brain to the brain stem and to the structures of the head and neck.

In the treatment of acute Wernicke's aphasia, the initial focus should be on A. encouraging self-monitoring of the adequacy of verbal output B. increasing the rate of speech C. improving the client's ability to elaborate verbally D. increasing the complexity of sentence structures

A Wernicke's aphasia results in deficits related to the ability to recognize the adequacy of one's verbal production.

Which of the following is the most reasonable standard to apply when judging whether a client has achieved generalization of a targeted skill? A. The client uses the targeted skill under stimulus conditions that were not present during the training process and without reinforcement. B. The client maintains the correct production of the targeted skill when the reinforcement schedule is changed. C. The client correctly produces the targeted skill effortlessly and without hesitation. D. The client is able to monitor errors and correct them with only a minimal number of cues from the clinician.

A When judging whether a client has achieved generalization of a targeted skill it is a reasonable standard to apply. It shows that the client is exhibiting the skill independently in situations not covered during training.

What technique narrows the airway and pushes the epiglottis posteriorly?

A chin tuck

n assessing children with language disorders, clinicians often count the number of morphemes in a child's utterance. This counting helps assess the length of single utterances the child typically produces. Clinicians follow certain rules in counting morphemes. Of the following rules, which one is correct?

A compound word, such as birthday, is counted as one morpheme.

suggests the ways in which two variables are related to each other.

A correlation coefficient

A team of SLPs is evaluating whether a new language intervention is suitable for use with children who are on their caseload. The clinicians read an article documenting research on the effect of a novel treatment on the language performance of 30 children with specific language impairment. The researchers administered a standardized language test to the children before and after the treatment program. The children's post treatment scores on the test were significantly higher than their pretreatment scores. This led the researchers to conclude that the treatment was highly effective. Based on the preceding description, which of the following appears to be a significant limitation of the design of the study? Select all that apply. A. The use of a single-group pretest-posttest design B. The absence of a control group C. The inability to rule out the possibility that the children's language scores improved for reasons other than the treatment D. The use of random assignment to place the children into the treatment group E. The number of participants in the study

A,B, & C The use of a single-group pretest-posttest design is a limitation because there is inadequate control of internal & external validity without a control group. The absence of a control group is a limitation because control groups are a way of introducing control and can better isolate the effect of the treatment. Furthermore, since it is a longitudinal study, improvement based on language development cannot be ruled out.

The American Speech-Language-Hearing Association (ASHA) regulates professional practice in speech-language pathology and audiology by specifying the scope of practice. According to the existing scope-of-practice documents, which of the following statements is false? A. ASHA does not allow infant feeding by speech-language pathologists (SLPs). B. Memory and such other cognitive aspects of communication disorders are within the SLP's scope of practice. C. Counseling is not just the province of psychologists; SLPs, too, can counsel their clients. D. Sensory awareness training related to swallowing disorders is within the SLP's scope of practice.

A. ASHA does not allow infant feeding by speech-language pathologists (SLPs)

Which of the following is the cranial nerve that innervates the larynx and the levator veli palatini, palatoglossus, and palatopharyngeus muscles? A. Cranial nerve X, the vagus nerve B. Cranial nerve V, the trigeminal nerve C. Cranial nerve XI, the spinal accessory nerve D. Cranial nerve VII, the facial nerve

A. Cranial nerve X, the vagus nerve

You are working with Devin, a 9-year-old boy with an orofacial myofunctional disorder (OMD). Devin has been referred by his orthodontist because orthodontia is not effective due to Devin's problems accompanying the OMD. You have been asked to evaluate Devin and provide remediation, if needed. Which one of the following most accurately describes the situation you will probably encounter? A. During Devin's speech production, his tongue may exert some force against his front teeth, and he will probably have a deviant swallow, as well as articulation errors involving /s/, /z/, and others. B. Devin will probably have a posterior open bite, no difficulties with swallowing, and problems producing /b/ and /v/. C. Devin will probably have an anterior open bite and a deviant swallow but normal posture of his tongue at rest. D. Devin will probably have a deviant swallow, articulation errors of /s/ and /z/, and an anterior open bite, but it is not within the purview of the speech-language pathologist to provide intervention; you can only provide an evaluation.

A. During Devin's speech production, his tongue may exert some force against his front teeth, and he will probably have a deviant swallow, as well as articulation errors involving /s/, /z/, and others.

A child with a language delay is in Brown's Stage V of morpheme mastery. Which of the following would not be appropriate to target in intervention? A. Regular plural inflection -s B. Irregular third person C. Contractible auxiliary D. Contractible copula

A. Regular plural inflection -s

childhood apraxia of speech (CAS). As you plan intervention for these children, what can you anticipate they will display?

A. Slow, effortful speech, inconsistent and multiple articulation errors, and more difficulty with consonant clusters than fricatives

You have just completed an evaluation of Tanveer, a 6-year old who speaks Urdu at home (Urdu is his primary language) and English at school. You have discovered that he has a language impairment, and you are creating an intervention plan for him. It has been found that Urdu is his stronger language, and that he is still in the process of learning English. Which one of the following intervention principles should most strongly guide your treatment plan? A. carrying out treatment in Urdu will be more effective and efficient than carrying out therapy in English B. to not confuse Tanveer, therapy should be conducted in English only C. Tanveer's parents should be told to speak only English at home, as being in a bilingual atmosphere will confuse him D. Tanveer needs to be placed in an all-English special education classroom where he can get intensive English input and extra support

A. carrying out treatment in Urdu will be more effective and efficient than carrying out therapy in English

You are evaluating Tatyana, a Russian-speaking child with a suspected language impairment. To accurately estimate her language skills, you engage a Russian-speaking interpreter and count Tatyana's responses to test questions in both Russian and English. You are employing: A. conceptual scoring B. dynamic assessment C. criterion-referenced testing D. assessment of working memory

A. conceptual scoring

A school-based clinician is assessing the velopharyngeal adequacy of Tanveer, a 13-year-old immigrant high school student from Pakistan. Tanveer was born with a cleft of the palate and lip; there was no repair until his family came to the United States when he was 11 years old. In Pakistan, Tanveer and his family lived in a rural area where surgery was unavailable. Though the repair surgery a year ago was successful and Tanveer now has a more aesthetically pleasing appearance and better speech, there is still audible nasal emission and hypernasality when he speaks. The clinician plans to refer him to a local craniofacial team, but she first wants to conduct as thorough an examination as she can. Despite the lack of instrumentation available at her school site, she does have access to an oral manometer. She used this to provide a beginning point from which to refer Tanveer to the craniofacial team. After obtaining a ratio by comparing pressures achieved in the nostrils-occluded and the nostrils-open conditions, she concluded that he especially needs to be referred to the craniofacial team for possible further surgery or a pharyngeal flap. When she did oral manometry, she probably found that Tanveer A. had a ratio of .87. B. had a ratio of .91. C. had a ratio of 1.4. D. had a ratio of 1.0.

A. had a ratio of .87.

the theory that asserts that each child is born with an innate language acquisition device (LAD) is the A. nativist theory of Chomsky. B. cognitive theory of Piaget. C. behavioral theory of Skinner. D. social interactionist theory of Vygotsky

A. nativist theory of Chomsky.

A treatment efficacy study on phonological disorders proposed that the treatment to be investigated will not make a difference. This means that the investigator A. proposed a null hypothesis. B. proposed an alternative hypothesis. C. proposed a neutral hypothesis. D. proposed an acceptable hypothesis.

A. proposed a null hypothesis.

When sound waves move from one medium (e.g., air) to another (e.g., water), the motion causes a bending of the sound wave due to change in its speed of propagation. This phenomenon is known as A. refraction. B. reflection. C. compression. D. rarefaction.

A. refraction.

Certain variables (e.g., attitudes and opinions) may change simply because they are measured more than once. This is a problem of A. testing, reducing internal validity. B. reactive effect of pretesting, affecting external validity. C. instrumentation, affecting internal validity. D. the Rosenthal effect.

A. testing, reducing internal validity.

spontaneous autosomal dominant mutations. The gene and the locus of this syndrome is FGR2 at 10q25-26. The child has midfacial hypoplasia, an arched and grooved hard palate, and mild mental retardation

Apert syndrome

CNS damage but not weakness or paralysis of facial muscle, difficult to program movements for speech Motor programmer disorder is

Apraxia

absence of any deficits in strength, tone, or coordination and rather is characterized by deficits in motor planning and/or programming of speech

Apraxia

Which of the following provides the most important diagnostic information to an SLP making a differential diagnosis between childhood apraxia of speech and flaccid dysarthria in a child? A. The child's articulation performance at the sentence level B. A history of the child's development of chewing, eating, and swallowing C. A history of the child's language development D. The child's willingness to function in sociocommunicative events

B A child with CAS does not have difficulties with chewing, eating, and swallowing, whereas a child with flaccid dysarthria is likely to have such difficulties.

Doing which of the following would likely yield the most useful information about the effectiveness of an intervention strategy? A. Reviewing the results of standardized assessment instruments B. Utilizing a single-subject design (ABAB) or a reversal procedure (ABA) C. Utilizing the subjective, objective, assessment, and plan (SOAP) format D. Requesting that a colleague reevaluate the client

B An ABAB or ABA design shows changes when intervention is applied. Single-subject design using ABAB or ABA is the best method to determine whether intervention has succeeded.

Which of the following is the best action to take initially with a client who presents with poor oral control of liquids and solids, coughing and choking while eating and drinking, and a history of hospitalizations associated with pneumonia? A.Thickening liquids so that the client will be better able to control oral movements for swallowing B.Obtaining a modified barium-swallow study to determine appropriate interventions C.Evaluating the client's ability to eat a variety of foods in order to determine which foods are safest D.Prescribing that the client be NPO, since aspiration is present

B An assessment of the problem must be undertaken before treatment is provided, and the best way to do this is to obtain a modified barium-swallow study.

Questions 9-11 refer to the following. This investigation was motivated by observations that when persons with dysarthria increase loudness, their speech improves. Some studies have indicated that this improvement may be related to an increase of prosodic variation. Studies have reported an increase of fundamental frequency (F0) variation with increased loudness, but there has been no examination of the relation of loudness manipulation to specific prosodic variables that are known to aid a listener in parsing out meaningful information. This study examined the relation of vocal loudness production to selected acoustic variables known to inform listeners of phrase and sentence boundaries: specifically, F0 declination and final-word lengthening. Ten young, healthy women were audio-recorded while they read aloud a paragraph at what each considered normal loudness, twice-normal loudness, and half-normal loudness. Results showed that there was a statistically significant increase of F0 declination, brought about by a higher resetting of F0 at the beginning of a sentence and an increase of final-word lengthening from the half-normal loudness condition to the twice-normal loudness condition. These results suggest that when some persons with dysarthria increase loudness, variables related to prosody may change, which in turn contributes to improvement in communicative effectiveness. However, until this procedure is tested with individuals who have dysarthria, it is uncertain whether a similar effect would be observed. Watson and Hughes are cautious when suggesting that the speech of some persons with dysarthria improves because of the prosodic changes that result from increasing vocal loudness. Of the following, which is the most likely reason for this caution? A.Only women were studied. B.The prosody of persons with dysarthria may not show similar loudness effects. C.Several acoustic variables related to speech prosody were not included. D.Reciting a paragraph aloud is unlike spontaneous speech.

B At the end of the abstract, the researchers Hughes suggest a relationship between increased vocal loudness and "improvement in communicative effectiveness" in some persons with dysarthria. This relationship is not directly supported by their study, as individuals with dysarthria were not tested.

Cognitive therapy for stuttering focuses on which of the following? A.Extinguishing the overt, dysfluent speech behavior by withholding positive reinforcement B.Changing distorted beliefs about self-efficacy and the need to speak with complete fluency C.Providing positive reinforcement during periods of fluent speech D.Reducing dysfluent speech behavior by using visual imaging

B Changing distorted beliefs about self-efficacy and the need to speak with complete fluency represents the only answer choice that is cognitive in nature.

When counseling the parents of a child who has an articulation disorder, the SLP can cite developmental norms to show which of the following? A.The child's misarticulation will interfere with reading skills. B.A certain percentage of children of a certain age can correctly articulate the misarticulated sound. C.The misarticulation is caused by faulty learning. D.The misarticulation is not physically based.

B Developmental norms demonstrate that a certain percentage of children of a given age can correctly articulate certain sounds

Individuals diagnosed as having hemifacial microsomia are also most likely to have A. laryngeal dysfunction B. ear malformation C. webbed fingers and toes D. widely spaced eyes

B Ear malformation is the only symptom of those listed that is typical of hemfacial microsomia, a genetic diagnosis with the oculo-auricular-vertebral (OAV) spectrum.

Which of the following should be the primary focus of early language intervention for at-risk infants? A.Establishing object permanence through play activities B.Training primary caregivers to facilitate language learning C.Creating readiness activities in the context of play D.Enhancing social communication through play activities

B Early language stimulation in at-risk infants is best provided by primary caregivers who have been trained in practices that promote learning.

Fiber-optic instrumentation is appropriate for evaluation of which of the following types of disorders? A.Articulation B.Phonation C.Proprioception D.Respiration

B Fiber-optic nasopharyngoscopy or laryngoscopy is a commonly used approach to evaluate vocal fold anatomy and physiology for voice production (phonation).

An SLP sees a college-educated 22-year-old man who has sustained a brain injury as a result of a motor vehicle accident eighteen months earlier. The man was unconscious for five days and had posttraumatic amnesia for three months. In the last year he has held three unskilled jobs, from which he was released for unspecified reasons. Based on the information given, the man's most pervasive condition is most likely impaired A. visual construction B. attention and memory C. speech D. language

B Impaired attention & memory are consistent with brain injury sustained by the client. There is no evidence for any other areas of potential deficit listed in the other choices.

A client has been determined to have poor upper esophageal sphincter opening secondary to decreased hyolaryngeal excursion, following a lateral medullary stroke. Cognitive functions are within normal limits. Which of the following interventions would be the most appropriate recommendation for this client as an initial course of treatment for the underlying disorder? A. Thermotactile stimulation B. Mendelsohn maneuver C. Thickened liquids D. Chin-down posture (head/neck flexion)

B Mendelsohn Maneuver is the only one of the listed interventions designed & shown in research to prolong the duration & diameter of UES opening.

Which of the following is most important for an SLP to do when assessing a child who has an acquired brain injury? A.Evaluate pragmatics through a structured language test B.Compare premorbid performance with present performance C.Ensure administration of an intelligence test D.Compare nonverbal performance with verbal performance

B SLP needs to know what skills the child attained prior to the brain injury in order to select appropriate treatment goals

A speech-and-hearing clinic has recently opened, but referrals are slow in coming. Which of the following would be most appropriate and effective for the clinic's director to do first? A. Ask local hospitals to provide names of recent clients likely to need speech-language services B. Identify and define the major consumer groups and referral sources, and develop a plan to reach them C. Identify the weaknesses in the competition, and inform consumer groups and referral sources of the weaknesses D. Wait for demand to increase on its own, because marketing speech-language services is against the ASHA Code of Ethics

B The clinical director of a new SLP clinic would most appropriately prospect for referrals by identifying major consumer groups & creating a plan to contact them.

Language intervention for a child at the one-word stage should be most strongly influenced by a consideration of the child's A.motor skills B.cognitive skills C.syntactic skills D.articulation skills

B The cognitive skills of a child at the 1-word stage will most strongly influence the child's speech-language responses, so language intervention for the child should take into account the child's cognitive skills.

Of the following sentences, which represents the greatest degree of syntactic complexity? A. Is John helping Bill? B. Why isn't John helping Bill? C. John isn't helping Bill. D. Why is John helping Bill?

B The sentence involves inversion, adding negation, and adding a question word, so with three transformations, this option has the greatest degree of syntactic complexity.

The sensorimotor integration of the muscles of the lower face depends on which two of the cranial nerves? A. The accessory and hypoglossal B. The trigeminal and facial C. The vagus and glossopharyngeal D. The phrenic and facial

B The trigeminal nerve has a motor component which is involved in mastication (and thus the lower face) and a sensory component, which provides sensory information from the entire face (including the lower face). The facial nerve provides motor innervation to several muscles that are found in the lower face, including obicularis oris & the buccinator.

"Book read me." "Me TV see." The utterances above, spoken by a 3-year-old monolingual English-speaking child, indicate that the child most likely has a problem with which of the following? A. Semantic redundancy B. Syntactic relationships C. Morphological relationships D. Pragmatics

B The utterances are not in the correct order, which is characteristic of a syntactic error.

For which individual would a recommendation for an augmentative and alternative communication (AAC) intervention be least appropriate? A. A preschool child with a language-learning disorder and highly unintelligible speech B. A teenager with a repaired cleft palate who continues to experience hypernasality C. A young adult with severe cerebral palsy precluding functional oral communication D. A 55-year-old man who has had a laryngectomy

B This is correct because someone with hypernasality would still be intelligible, and an AAC device would not be needed.

A 70-year-old female has dysphagia characterized by poor posterior oral containment of the bolus during the oral preparatory stage, causing aspiration before the swallow. Cognition and the pharyngeal stage of the swallow are intact. Which of the following is the most appropriate treatment approach for the client? A.Providing a puree diet with thickened liquids B.Having the client flex her head forward (perform the chin-down posture) during oral preparation and transit stages of the swallow C.Having the client turn (rotate) her head to the right when swallowing D.Providing a diet that consists of thin liquids

B This treatment alters oral configuration to place the base of the tongue superior to the bolus inside the oral cavity. Since liquid cannot flow uphill against gravity, this intervention compensates for impaired palatoglossus contraction, which is causing impaired posterior oral containment.

Which of the following is an accurate statement about whispered speech? A. It is produced by approximating the arytenoid cartilages so that their medial surfaces are in direct contact. B. It is composed largely of aperiodic sounds. C. Spectrographic analysis of it reveals no discernible formants for the vowels. D. Most people can produce longer utterances per breath using it than they can using conventional phonation.

B Whispered speech is composed largely of aperiodic sounds, as the vocal folds do not vibrate while whispering is taking place.

A clinician at a trauma center receives a referral of a 19-year-old college student, Peter, who was involved in a motorcycle accident. He recently received his motorcycle license. His parents stated, "He is a carefree young man who has always believed he was invincible." The medical chart indicates that Peter was riding his bike at high speeds and was not wearing a helmet when he crashed into a stationary delivery truck. The paramedic report indicates that he was thrown from his bike and went head first into the delivery truck. He was unconscious when he entered the hospital and was rushed to the emergency room. He suffered several fractures. His right leg is in a cast, and he has injuries to several parts of his body. The medical chart also indicates that he has a traumatic brain injury (TBI) and suffers many symptoms of TBI. Emergency surgery was performed to repair hematomas that were a result of the accident. Question When the clinician initially assesses Peter, she observes that he manifests a classic sign of TBI. When the clinician begins assessing him at the bedside, he would be most likely to display which typical sign of TBI? A. Euphoria B. Aggression C. Bed wetting D. Mutational falsetto

B. Aggression

While the clinician was conducting the examination in the radiology department to determine whether there was a delay in the pharyngeal swallow reflex, she most likely would have selected the following treatment technique with the patient: A. Head back B. Head down (chin tuck) C. Lying on one side D. Cervical auscultation

B. Head down (chin tuck)

Measures of jitter and shimmer are becoming more common in use with voice patients because they can be useful in early detection of vocal pathology. Although you suspect that the otolaryngologist will find obvious vocal pathology, given Allison's history of prolonged hoarseness, you still want to obtain measures of jitter and shimmer because these can serve as an excellent baseline, especially if Allison has phonosurgery. When you take these measures, you might expect to see

B. large amounts of both jitter and shimmer, with more than 1 dB of variation across vibratory cycles when shimmer is measured.

A clinician has been asked to evaluate a patient, Sydney T., who is 65 years old. He was rushed to the emergency room because of trouble with speaking, numbness and loss of movement in his face, vision problems, and confusion understanding simple statements. After completing a medical test, it was determined that Sydney had suffered a cerebrovascular accident (CVA). According to the medical chart, he "has trouble swallowing and suffers from speech and language problems secondary to the CVA." The dietitian is concerned that Sydney has lost weight and is not eating all his meals. The report indicates that "the patient seems depressed and uninterested in eating." The nursing staff indicates that he eats minimally during breakfast, lunch, and dinner. Reportedly, the patient seems to be unable to produce intelligible speech, and the certified nursing assistants are unable to understand him. Sydney's wife indicated that prior to his hospitalization he was an outgoing person and coached high school football. Question The clinician decides to conduct a thorough swallowing assessment. She conducts a modified barium swallow study and notes during the evaluation that Sydney has left hemiplegia. Consequently, the results of the study reveal that during administration of a mechanical soft bolus, Sydney has food stuck in the left side of the buccal cavity. This is an example of A. pooling. B. pocketing. C. Passavant's pad weakness. D. uvular paralysis.

B. pocketing.

A thrombus is a A. traveling blood clot that blocks the flow of blood. B. stationary blood clot that blocks the flow of blood. C. cause of aphasia in women only. D. bleeding vessel.

B. stationary blood clot that blocks the flow of blood.

A study designed to evaluate a treatment procedure for hoarseness of voice recruited all subjects who had witnessed a football game the previous day. The treatment, conducted over the following 2 weeks and offered to all subjects, resulted in improved voice quality. A critic said that the study lacked internal validity because of A. median statistics. B. statistical regression to the mean. C. maturational variables. D. genetic influences.

B. statistical regression to the mean.

A clinician is treating a 7-year old girl, Ariela, with a w/r substitution. The clinician says "Ariela, watch me, listen carefully, and say what I say--/r/." The clinician then evaluates Ariela's ability to imitate his production of /r/. He has just assessed Ariela's: A. auditory discrimination skills B. stimulability C. phonological awareness skills D. evoked speech ability

B. stimulability

The consistency effect has shown that A. stuttering always occurs on the sample words on repeated readings. B. stuttering recurs on certain words on repeated oral readings. C. adaptation is not real. D. adaptation and consistency cancel each other.

B. stuttering recurs on certain words on repeated oral readings.

Which aphasia has 1. Non fluent, effortful, slow halting uneven speech 2. limited word output 3. impaired repetition 4. agrammatism 5. involves brodmanns areas 44 & 45 6. posterior inferior frontal gyrus of the LEFT hemisphere 7. supplied by upper middle cerebral artety

Broca's aphasia

You are assessing a 60-year-old woman for a possible diagnosis of aphasia, and you are interested in the specific type of aphasia this woman might have. You notice that she has marked buccofacial apraxia. This condition alone might suggest which of the following?

Broca's aphasia, transcortical motor aphasia, or conduction aphasia

A 4-year-old child presents with general speech patterns that include liquid gliding, stridency deletion, final-consonant deletion, and consonant-cluster reduction. Which of the following is the speech-language pathologist's most appropriate recommendation for the child? A. Treatment is not needed, because the child's speech will improve during the next year. B. Treatment should focus on the production of /p/, because /p/ is one of the earliest acquired phonemes. C. Treatment should use a phonological approach and focus initially on the production of final consonants. D. Formal treatment is not needed not, but the parents should be counseled to read aloud to the child, correct the child's errors, elicit correct repetitions, and, when needed, interpret to other people what the child is trying to say.

C A 4-year old child should have already developed final consonants.

According to research on the development of Brown's morphemes in young children, which of the following is a determinant of acquisition order? A.Phonological ease of production B.Figurative-language ability C.Semantic and syntactic complexity D.Sequencing and segmentation strategies

C Brown's morphemes are acquired by children in an order that is determined by semantic & syntactic complexity, with the simplest forms acquired first. The order of acquisition is typically followed by all children.

Compared with children who do not have language disorders, children with language disorders tend to A. take more conversational turns B. initiate topics and direct the flow of conversation more C. ask fewer open-ended questions D. initiate more indirect requests

C Children with language disorders tend to ask fewer open-ended questions than do children who are developing normally

Computer software that has been developed to facilitate speech and language treatment can best be used A. in group sessions when the SLP's caseload precludes working individually with clients B. by clients in place of services that would otherwise be provided by an SLP C. by clients under the direction of SLPs D. when SLPs are unavailable

C Computer software and other aids are intended not to replace treatment sessions provided by an SLP but rather to enhance clients' opportunities to improve their skills in relation to goals established by the clinician.

For which of the following conditions is it most appropriate for the SLP to recommend that the patient's primary-care physician refer the patient to a prosthodontist for construction of a palatal-lift appliance? A. Submucous cleft palate B. Unrepaired cleft of the secondary palate C. Flaccid paralysis of the soft palate D. Congenitally short palate

C Construction of a palatal-lift appliance is appropriate for a patient with flaccid paralysis characterized by an intact palate that does not function.

Which of the following describes an important diagnostic distinction between apraxia of speech and dysarthria? A. Apraxia of speech is a result of lower motor neuron lesions, whereas dysarthria is the result of upper motor neuron lesions. B. Clients with apraxia of speech lack the ability to monitor reactive speech, whereas clients with dysarthria lack the ability to sequence volitional speech movements. C. Strength and coordination of the speech musculature are intact in clients with apraxia of speech, whereas slowness, weakness, incoordination, or altered tone of the speech musculature are associated with dysarthria. D. Apraxia of speech is characterized by distortions, omissions, and substitutions, whereas dysarthria is characterized by inconsistent, highly variable misarticulations.

C Dysarthria is a motor speech disorder characterized by slowness, weakness, incoordination, or altered tone of the speech production mechanism. The definition of apraxia includes the absence of any deficits in strength, tone, or coordination and rather is characterized by deficits in motor planning &/or programming of speech.

A 55-year-old woman, recently hospitalized for probable cerebrovascular accident (CVA), is referred for evaluation of stuttering speech. The initial conversation with the client indicates that speech is characterized by frequent initial-phoneme repetitions and prolongations as well as associated mildly effortful eye blinking. Which of the following pieces of information is crucial to accurate speech diagnosis and decisions regarding management of the speech problem? A.The site and extent of the lesion associated with the suspected CVA B.Whether the client has any associated dysphagia or dysphonia C.Whether the dysfluencies began before or after the suspected CVA D.Whether the client feels frustrated by the dysfluencies

C Dysfluent speech may or may not be directly caused by the patient's suspected CVA.

Hearing loss in infants who are born with a cleft palate is usually related to which of the following? A.The infant's inability to create positive pressure in the oral cavity B.Malformation of the middle-ear ossicles associated with malformation of the palate C.Eustachian tube dysfunction D.Cochlear dysfunction

C Eustachian tube dysfunction, a major factor contributing to middle-ear disease & conductive hearing loss, is nearly universal in infants with cleft palate.

Which of the following constitutes the major component of an audiologic rehabilitation program for infants with a moderate sensorineural hearing loss? A.Development of cognitive skills B.Development of social skills C.Parent-mediated auditory stimulation D.Gestural stimulation

C For an infant known to have a HL, parents are the best position to provide consistent auditory stimulation, which the infant needs in order to develop an awareness of sound.

To provide greater independence for a client who has a brain injury and is in a late stage of speech-language treatment, which of the following techniques is most appropriate? A. Increasing memory-retention span B. Using word-repetition drills C. Teaching compensatory strategies D. Training visual-perceptual skills

C Functional independence is a main goal for a client shortly before dismissal from treatment. It can be achieved by teaching compensatory strategies to minimize any remaining deficits.

Which of the following procedures would be effective in remediating a falsetto voice for an adult male with a severe bilateral hearing loss? A.Development of phonation from coughing B.Pushing exercises C.Manual depression of the larynx D.Manual elevation of the larynx

C Manual depression of larynx will serve to lengthen the vocal folds, allowing them to vibrate at a lower frequency.

An otolaryngologist has referred a 45-year-old man for voice treatment following medialization thyroplasty for a paralyzed vocal fold. Which of the following is the most appropriate therapeutic strategy for the SLP to use? A. Assisting the patient to produce a soft glottal attack B. Digitally manipulation of the patient's neck to reduce strap-muscle tension C. Assisting the patient to produce a hard glottal attack D. Employing techniques aimed at increasing airflow

C Medialization thyroplasty moves the paralyzed vocal fold closer to the mid-glottis to allow better compensation by the unaffected fold. Only the production of a hard glottal attack addresses the compensatory behavior.

When treating a client who is using an electronic augmentative-communication device, the speech-language pathologist's primary goal should be to A. ensure that the client develops skill in using every technical aspect of the aid B. ensure that the client's caregivers learn how to modify the aid's hardware and any applicable software to meet the client's communication needs C. train the client to use the aid as independently and interactively as possible in a variety of settings D. help the client develop the skills necessary for moving on to a more sophisticated device

C Primary goal of any intervention is to effect the best functional outcome for the client. Generalization with respect to the environments in which the AAC devices is used effectively would help accomplish this goal.

A number of research reports have described poor auditory memory in children with language impairments. Which of the following can most appropriately be concluded from these studies? A.Poor auditory memory can be improved by language-intervention programs that focus on teaching vocabulary and word meanings. B.Poor auditory memory is a reflection of a language impairment, and clinicians and researchers cannot effect improvement. C.Poor auditory memory could be a reflection or a cause of a language impairment or could be related to some other factor, and further research is needed to determine which is the case. D.Children with language impairments normally do not have well-developed representational skills.

C Research reports are restricted to the variables examined in the studies on which they are based. A relationship between poor auditory memory and language impairment has been found in some research studies. Other factors could be involved, however, suggesting that further research is needed. Conclusions about treatment or the effects of the impairment must be made through additional research studies.

Excessive nasality is associated with inadequate velopharyngeal closure. An SLP is training a client to self-monitor nasality during speech. Which of the following tactics will best allow the speaker to determine whether there is excessive nasal airflow? A.Looking in a mirror while speaking B.Being aware of vowel-sound productions C.Speaking/phonating while alternately leaving the nostrils open and pinching them closed D.Monitoring production of consonant blends

C Speaking while alternately leaving the nostrils open & pinching them closed is an easy way for a speaker to determine whether inappropriate nasal airflow is occurring. This technique allows one to determine the difference in airflow pattern when speech is produced with the nostrils occluded as compared with speech produced when the nostrils are open. For a speaker with velopharyngeal incompetence, closure of the nostrils will eliminate the nasal airflow in production of /s/.

A 4 1/2-year-old boy has significant speech and language difficulties exemplified by poor oral-motor control, slight difficulty in swallowing, high palate, poor tongue mobility, and fasticulation on protrusion. During an evaluation, the speech-language pathologist notes very poor paper-and-pencil grasp, poor posture, and an inability to complete performance tasks requiring fine motor control. To which of the following should the child be referred in order to obtain additional diagnostic information? A. An orolaryngologist B. A physical therapist C. A neurologist D. A physiologist

C The boy's deficits as noted are most indicative of a neurological disorder, and a neurologist can best provide the additional diagnostic information needed.

A teacher asks the speech-language pathologist for advice regarding a child who talks excessively during class, rarely listens to instructions, and does work only intermittently. Attempts at alternative seating for the child have not been successful. Of the following, which is the most appropriate recommendation that the speech-language pathologist can provide to the teacher? A. Have the school counselor consider working with the child and the parents on self-control and discipline B. Put the child on a management system for classroom behavior C. Refer the child for evaluation by members of the child-study team D. Suggest remedial speech-language services for the child to improve the child's interactive communication

C The child has a behavioral problem that should be evaluated for the purposes of identification and management planning.

A 5-year-old girl with a repaired cleft palate has recently undergone a pharyngeal flap operation to correct velopharyngeal incompetence, but she continues to use glottal stops, pharyngeal fricatives, and mid-dorsum palatal stops. Which of the following is the most appropriate action for the SLP to take? A.Recommending that the child be examined by a cleft palate team to determine the cause of the persistent articular errors B.Referring the child back to the surgeon to determine the need for a revision of the pharyngeal flap C.Initiating articulation treatment to teach the correct placement for the stops and fricatives D.Initiating articulation treatment to teach correct production of nonpressure consonant sounds

C The child is using compensatory glottal stops, mid-dorsum palatal stops, and pharyngeal fricatives for standard American English stops and fricatives. She has had surgical management to correct velopharyngeal incompetence. She now needs to learn to produce the stops & fricatives for which she is making compensations. Therefore, articulation treatment is appropriate.

A 60-year-old man says that he has "trouble thinking of names and words" and that it is interfering with his job performance. The problem began three months ago after he had a minor cerebrovascular accident. He does not report any other problems. His conversation is characterized by some hesitancies, latencies, repetitions, interjections, and self-corrections. On the basis of an interview and the results of an aphasia battery, it is concluded that he has a mild aphasia. The most appropriate course of action is to A.advise the client to wait for three more months in order to allow spontaneous recovery to take place B.begin a treatment program designed to decrease dysfluencies in his speech C.offer a treatment program designed to improve word-retrieval skills D.encourage the client to increase his reading of information related to his profession in order to facilitate recall of professional terminology

C The client's language reflects the word-finding difficulties of an individual with aphasia. Treatment to remediate his difficulties is appropriate.

Which of the following areas needs to be evaluated first for a 5 year old who says [pun] for "spoon" and [top] for "soap"? A. Auditory discrimination B. Dialectal differences C. Phonological system D. Receptive language

C The error patterns described are phonological in nature (cluster reduction and stopping) and are not consistent with a deficit in language comprehension (receptive language), dialectal differences, or problems with auditory discrimination.

Which of the following most accurately represents the etiology of cleft palate? A.Genetic factors alone B.Environmental influences alone C.Genetic factors interacting with environmental influences D.Medications taken by the mother during pregnancy

C The interaction of genetic factors & environmental influences represents the etiology of cleft palate.

A 67-year-old male patient with no history of swallowing problems has undergone a cardiothoracic surgical procedure. Postoperatively, he is found to be aspirating while swallowing and is diagnosed with a left vocal-fold paralysis and left pharyngeal paresis. Which of the following is the most likely etiology? A. An intraoperative CVA in the right pons B. Damage to the right recurrent laryngeal nerve C. Damage to the left recurrent laryngeal nerve D. A left hemispheric stroke

C The left recurrent laryngeal nerve courses under the aortic arch in its course back to innervate the left larynx and the inferior pharynx. The nerve can be damaged in cardiothoracic operations including aortic arch or valve repairs. Only the left (& not right) recurrent laryngeal nerve has this course.

A disfluent 4-year-old child is referred to an SLP for assessment. Which of the following is most important for the SLP to consider in deciding whether the child is developmentally nonfluent or stuttering? A. The length of time the child has been disfluent B. The rate at which the child talks C. The nature and frequency of the child's disfluencies D. The child's comments to the clinician about the disfluencies

C The nature & frequency of the child's repetitions will help distinguish between normal disfluency and the type of stuttering for which remediation is indicated.

John is a 4 1/2 year old whose consonantal inventory includes word-initial [ w ], [ m ], [ n ], [ p ], [ b ], [ t ], [ d ], and [ f ]. He uses [ t ] for /k/, [ d ] for /ɡ/, [ b ] for /v/, and [ f ] for /θ/. He produces no consonant clusters. His word-final consonantal inventory consists of [m] and [n]. His word shape inventory includes V, CV, CVC, and CVCV. The information given most strongly indicates that the child has A.childhood apraxia of speech B.an oral motor impairment C.delayed phonological development D.a significant high-frequency hearing loss

C The pattern describes the phonological pattern of a younger child. The errors are consistent, unlike childhood AOS, which has an inconsistent error pattern. Children at this age typically have mastered consonant clusters & use word-final consonants.

Questions 9-11 refer to the following. This investigation was motivated by observations that when persons with dysarthria increase loudness, their speech improves. Some studies have indicated that this improvement may be related to an increase of prosodic variation. Studies have reported an increase of fundamental frequency (F0) variation with increased loudness, but there has been no examination of the relation of loudness manipulation to specific prosodic variables that are known to aid a listener in parsing out meaningful information. This study examined the relation of vocal loudness production to selected acoustic variables known to inform listeners of phrase and sentence boundaries: specifically, F0 declination and final-word lengthening. Ten young, healthy women were audio-recorded while they read aloud a paragraph at what each considered normal loudness, twice-normal loudness, and half-normal loudness. Results showed that there was a statistically significant increase of F0 declination, brought about by a higher resetting of F0 at the beginning of a sentence and an increase of final-word lengthening from the half-normal loudness condition to the twice-normal loudness condition. These results suggest that when some persons with dysarthria increase loudness, variables related to prosody may change, which in turn contributes to improvement in communicative effectiveness. However, until this procedure is tested with individuals who have dysarthria, it is uncertain whether a similar effect would be observed. 9. Which of the following represent(s) the independent variable or variables used in the Watson and Hughes study? A.Prosody of dysarthric speech B.F0 declination and final-word lengthening C.Vocal loudness D.Speech intelligibility and communicative effectiveness

C The researchers manipulated vocal loudness to determine its effect on prosodic F0 and durational variables.

Questions 9-11 refer to the following. This investigation was motivated by observations that when persons with dysarthria increase loudness, their speech improves. Some studies have indicated that this improvement may be related to an increase of prosodic variation. Studies have reported an increase of fundamental frequency (F0) variation with increased loudness, but there has been no examination of the relation of loudness manipulation to specific prosodic variables that are known to aid a listener in parsing out meaningful information. This study examined the relation of vocal loudness production to selected acoustic variables known to inform listeners of phrase and sentence boundaries: specifically, F0 declination and final-word lengthening. Ten young, healthy women were audio-recorded while they read aloud a paragraph at what each considered normal loudness, twice-normal loudness, and half-normal loudness. Results showed that there was a statistically significant increase of F0 declination, brought about by a higher resetting of F0 at the beginning of a sentence and an increase of final-word lengthening from the half-normal loudness condition to the twice-normal loudness condition. These results suggest that when some persons with dysarthria increase loudness, variables related to prosody may change, which in turn contributes to improvement in communicative effectiveness. However, until this procedure is tested with individuals who have dysarthria, it is uncertain whether a similar effect would be observed. Which of the following best describes the experimental design of the Watson and Hughes study? A.A multiple-baseline design B.A between-subjects design C.A within-subjects design D.A mixed between- and within-subjects design

C The researchers use the same group of subjects who recite a paragraph under three conditions: at normal, twice-normal, and half-normal loudness. This is an example of a within-subjects design, in which the DV (in this care, prosodic F0 and durational variables) are measured repeatedly in the same subjects under different task conditions (in this care, vocal loudness).

Ms. Brown, a 70-year-old retired female, was admitted to the hospital following a CVA. The SLP conducted a comprehensive evaluation that revealed the presence of left-side neglect, anosognosia (denial of impairment), and visuospatial problems, including prosopagnosia (difficulty recognizing familiar faces). Although her auditory comprehension and repetition skills were good, she experienced difficulty with topic maintenance and turn taking. Based on the clinical features described, Ms. Brown's diagnostic classification would most likely be A.Wernicke's aphasia B.Conduction aphasia C.Cognitive-communicative disorder consistent with right hemisphere damage D.Cognitive-communicative disorder consistent with left hemisphere damage

C The symptoms described in the scenario are consistent with damage to the right hemisphere.

Which of the following best describes the rationale for using standardized, norm-referenced instruments to assess speech-language functioning? A.They enable the clinician to generate weekly statements about a client's treatment progress. B.They provide the clinician with the information that is necessary to generate a specific and comprehensive treatment plan. C.They enable the clinician to understand and make informed statements about how a client's performance compares with the performance of other people. D.They provide the clinician with reliable information about how well a client is likely to respond to treatment.

C The use of standardized normative instruments enables a clinician to compare a client's language & speech function with the average normal function for a person in the client's age range.

A videofluoroscopic study of a client with dysphagia revealed post-swallow vallecular residue occupying more than 50 percent vallecular height. Which of the following is the most likely overt symptom that the client will experience? A. Watery eyes during swallowing B. Oral pocketing of foods C. Coughing after swallowing D. Esophageal reflux

C The valleculae are depressions that lie lateral to the median epiglottal folds. Pooling of liquids in the valleculae gives a person the feeling that there is material remaining in the respiratory pathway, so coughing after swallow would be a natural reaction to expect.

A third-party reimburser asks the speech-language pathologist to demonstrate in a diagnostic statement that a child's communication problems have a physiological cause. Which of the following observations, if included in the statement, would best satisfy the request? A. The child's dentition is not yet fully developed but is within normal limits for a child of that age. B. The child has developmental delay, exhibiting speech that is not appropriate for a child of that age. C. The child demonstrates a motor-speech disorder and is unable to perform voluntarily the oral movements required for speech production. D. The child exhibits stridency deletion, consonant cluster reduction, stopping, and fronting.

C The wording represents a well-written diagnostic statement for a child with communication problems that are physiological, or functional, in nature. None of the other answer choices demonstrate the presence of a physiological problem.

The mode is evident in which of the following sets of scores? A. 12, 10, 5, 15, 8, 4, 11 B. 10, 3, 9, 12, 25, 29, 43 C. 16, 23, 16, 8, 16, 16, 4, 16, 16 D. 58, 76, 96, 12, 5, 9

C. 16, 23, 16, 8, 16, 16, 4, 16, 16

You are working in a special needs school with children who are at risk for language disorders. A teacher referred to you a young boy whose symptoms include low birth weight, microcephaly, low-set ears, micrognathia, wide eyes, and excessive drooling. His behaviors include hyperactivity, tantrums, aggression, repetitive movements, feeding problems, catlike meowing, as well as severe cognitive, speech, and motor delays. The case history indicates that this syndrome is partially due to the deletion of the short arm of chromosome 5. Which syndrome does this child display? A. Bantis B. Cruveilhier-Baumgarten C. Cri du chat D. Mobius

C. Cri du chat

A clinician at a trauma center receives a referral of a 19-year-old college student, Peter, who was involved in a motorcycle accident. He recently received his motorcycle license. His parents stated, "He is a carefree young man who has always believed he was invincible." The medical chart indicates that Peter was riding his bike at high speeds and was not wearing a helmet when he crashed into a stationary delivery truck. The paramedic report indicates that he was thrown from his bike and went head first into the delivery truck. He was unconscious when he entered the hospital and was rushed to the emergency room. He suffered several fractures. His right leg is in a cast, and he has injuries to several parts of his body. The medical chart also indicates that he has a traumatic brain injury (TBI) and suffers many symptoms of TBI. Emergency surgery was performed to repair hematomas that were a result of the accident. Question While the speech-language pathologist is conducting a TBI assessment on Peter, the attending physician asks her student intern whether she knows which test is used to identify the motor, verbal, and eye opening response. The student intern promptly responds that it's the A. Traumatic Brain Injury Scale. B. TBI Checklist Scale. C. Glasgow Coma Scale. D. Ranchos Los Amigos Scale.

C. Glasgow Coma Scale.

A researcher wishes to assess the efficacy of the new Riverton method of training children to say /r/ accurately. He decides to use a single-subject design because he has a private practice where he serves a number of children who have w/r substitutions. The researcher needs to keep several concepts in mind as he begins his research. Which one of the following concepts is false? A. An example of a multiple-baseline-across-settings design would be teaching a behavior (e.g., correct /r/ production) sequentially in different settings to demonstrate that the behavior changes only in a treated setting and thus treatment is effective. B. In a multiple-baseline-across-subjects design, several subjects are taught a behavior sequentially to show that only treated subjects change, and thus the treatment is effective. C. In the ABAB withdrawal design, a target behavior is base rated (e.g., /r/ in the A phase), taught to the subject (e.g., /r/ is accurately produced in the B phase), reduced by teaching its counterpart or an incompatible behavior (e.g., teaching w/r in the A phase), and then taught again (e.g., /r/ is accurately produced in the B phase) to show that the treatment is effective. D. For this researcher, an advantage of using a single-subject design to evaluate the efficacy of the Riverton method in training children to say /r/ accurately is that he can integrate research and clinical service by using the clients he serves as subjects in an experiment that attempts to answer a significant clinical question.

C. In the ABAB withdrawal design, a target behavior is base rated (e.g., /r/ in the A phase), taught to the subject (e.g., /r/ is accurately produced in the B phase), reduced by teaching its counterpart or an incompatible behavior (e.g., teaching w/r in the A phase), and then taught again (e.g., /r/ is accurately produced in the B phase) to show that the treatment is effective.

While working in a school setting, you are planning to assess a child whose parents speak Spanish. The child's primary language is Spanish, although the child is in the process of learning English. You want to find out if any standardized test will be applicable for the child. In selecting a particular test, you should consider which of the following? A. You can have a family member translate English test items into Spanish. B. You can go ahead and administer an English test because the child is learning English anyway. C. Spanish-speaking children of the same background as the client have been adequately sampled in the standardization of a potentially useful test. D. Whether you can translate English test items into Spanish.

C. Spanish-speaking children of the same background as the client have been adequately sampled in the standardization of a potentially useful test.

You are a new clinician in a hospital that has a neonatal intensive care unit (NICU). Your job title is that of newborn development specialist (NDS). Which one of the following would be false regarding your role as an NDS? A. You would support the families and infants with issues involving infant development and behavior. B. You would support the families and infants with issues involving hearing conservation and aural habilitation. C. You would not serve as primary care coordinator or case manager for the infants and families because a medical doctor usually fills this role. D. You would create individualized assessment and intervention strategies to support infants and their caregivers.

C. You would not serve as primary care coordinator or case manager for the infants and families because a medical doctor usually fills this role.

A patient with ataxia complains of difficulties maintaining balance and posture. The physician explains to the patient that the structure that regulates equilibrium, body posture, and coordinated fine-motor movements may be damaged. Therefore, the damaged structure is the A. cerebrum. B. pons. C. cerebellum. D. temporal lobe.

C. cerebellum.

A clinician at a trauma center receives a referral of a 19-year-old college student, Peter, who was involved in a motorcycle accident. He recently received his motorcycle license. His parents stated, "He is a carefree young man who has always believed he was invincible." The medical chart indicates that Peter was riding his bike at high speeds and was not wearing a helmet when he crashed into a stationary delivery truck. The paramedic report indicates that he was thrown from his bike and went head first into the delivery truck. He was unconscious when he entered the hospital and was rushed to the emergency room. He suffered several fractures. His right leg is in a cast, and he has injuries to several parts of his body. The medical chart also indicates that he has a traumatic brain injury (TBI) and suffers many symptoms of TBI. Emergency surgery was performed to repair hematomas that were a result of the accident. Question The clinician conducts several tests on Peter and notices that he has lost half of his side of vision (right side). This type of vision loss is called A. hemiplegia. B. hemiparesis. C. hemianopsia. D. hemispherectomy.

C. hemianopsia.

A 62-year-old male presents to an outpatient SLP following an extended stay at a rehabilitation facility. He had a left-hemisphere stroke three months ago and currently has moderate Broca's aphasia and severe apraxia of speech. It is difficult for him to participate in conversation because of the combination of his word-finding deficits and apraxia of speech. The patient's normal speech pattern consists of one- to two-word phrases, at times including paraphasias. He is able to write but often misspells words. During evaluation, the SLP finds the patient is able to initiate communication, recognize and categorize picture symbols, and comprehend familiar words and phrases. In general, his communication is fragmented and inefficient, but he will try anything to get his message across. Which of the following AAC treatment strategies would be most appropriate for the patient to start with? A.Accessing stored messages in a speech-generating device B.Learning sign language C.Answering multiple-choice questions and writing answers D.Using a 20-item picture board to convey essential wants and needs

Correct Answer: A Option (A) is correct. Accessing stored messages with a device can ease the burden of communication and assist the patient in answering simple questions.

A school-based SLP wants to discharge a fifth-grade student. The student has mastered all his goals except /r/forward slash r forward slash in conversational speech. The SLP worked with the student for the entire school year, and the student is able to use /r/forward slash r forward slash accurately 75 percent of the time. However, the goal is to reach 90 percent accuracy. Because the student worked on the skill all year and still has not met the established goal, the SLP feels the student performs at his highest possible level. The SLP does not think it is in the student's best interest to continue pulling the student out of class to address the goal. The parents are upset and are questioning the SLP's decision. Which of the following statements from the American Speech-Language-Hearing Association (ASHA) Code of Ethics can be used to back up the SLP's decision? A.Individuals who hold the Certificate of Clinical Competence shall evaluate the effectiveness of services provided, technology employed, and products dispensed, and they shall provide services or dispense products only when benefit can reasonably be expected. B.Individuals shall honor their responsibility to hold paramount the welfare of persons they serve professionally or who are participants in research and scholarly activities, and they shall treat animals involved in research in a humane manner. C.Individuals shall use every resource, including referral and/or interprofessional collaboration when appropriate, to ensure that quality service is provided. D.Individuals shall provide all clinical services and scientific activities competently.

Correct Answer: A Option (A) is correct. According to ASHA, it is important that SLPs remember to provide services when a benefit can be expected. The SLP has exhausted all treatment options and a benefit for this student is no longer expected.

An SLP uses evidence-based practice by integrating the perspectives and values of the client, patient, or caregivers into the treatment plan. Which of the following best reflects simultaneous incorporation of the host culture's perspectives and values and maintenance of the native culture's perspectives and values? A.Acculturation B.Assimilation C.Enculturation D.Ethnocentrism

Correct Answer: A Option (A) is correct. Acculturation is the incorporation of the host culture's perspective and the native culture's perspective and values.

To compensate for the effects of normal aging on cognitive functions, older adults will naturally demonstrate A.greater amounts of bilateral activation of prefrontal brain regions B.significant reduction in the activation of the hippocampus C.increased activation in areas of the cortex responsible for visual-spatial processing D.an increase in semantic abilities that do not rely on long-term memory abilities

Correct Answer: A Option (A) is correct. Alterations in the prefrontal areas in particular are some of the changes seen in normal aging, in part to compensate for reduced frontal activation.

A 72-year-old male presents to a skilled nursing facility (SNF) following a complicated hospital stay. The patient is morbidly obese and has a history of COPD, depression, hypertension, diabetes, Bell's palsy, encephalopathy, and congestive heart failure. During his stay, he was not intubated but became very weak as a result of extended time in bed. In the hospital, he was placed on an NDD Level 2 diet with nectar-thick liquids due to aspiration of thin liquids and poor oral control of regular solids. The patient worked until he was 50 years old, but his health conditions led to his early retirement. Before his hospitalization, the patient did not leave his house very often, and his wife helped him with all ADLs. While evaluating the patient at the SNF, the SLP noticed that the patient was disoriented and confused, appeared to be hallucinating, grunted loudly when attempting to stand, and presented with aphonia. There was no indication of this condition in the patient's hospital chart, and the patient's wife stated that the patient had no trouble using his voice while in the hospital. An ENT consult stated there was no structural reason for the aphonia. Suspecting that the aphonia was of a psychogenic etiology, the SLP made a referral to the resident psychiatrist and began treatment for both the aphonia and for dysphagia. After a few sessions of speech therapy, the patient has made limited progress with his aphonia. The resident psychiatrist has continued to work with the patient and recommends cognitive behavioral therapy sessions and continued speech therapy. A primary factor causing the patient's psychogenic aphonia is the presence of A.depression B.Bell's palsy C.COPDC O P D D.hallucinations

Correct Answer: A Option (A) is correct. Depression can lead to psychogenic disorders and given what this patient has been through, it is very likely that his depression has deepened during his hospital stay.

Which of the following will most effectively decrease the fundamental frequency? A.Increasing the mass of the vocal folds B.Increasing the subglottal pressure C.Raising the position of the larynx within the neck D.Lengthening the vocal folds

Correct Answer: A Option (A) is correct. Frequency is inversely proportional to mass. As the mass of the vocal folds increases, fundamental frequency decreases.

A 65-year-old patient is transferred from another facility with a diagnosis of aphasia. The patient's symptoms, however, appear more consistent with apraxia. Which of the following tasks for the patient is most appropriate when assessing verbal apraxia of speech? A.Repeating words of increasing length B.Recalling three common items C.Performing voluntary oral movements D.Following two-step commands

Correct Answer: A Option (A) is correct. Inability to repeat two-syllable words is a clear indicator of apraxia of speech.

An SLP receives a referral from an audiologist for a 72-year-old woman who demonstrates a moderate to severe bilateral sensorineural hearing loss preventing her from participating in social activities. Which of the following best identifies the SLP's role? A.Training in speech reading B.Prescribing hearing aids C.Performing an auditory evaluation D.Using auditory integration training

Correct Answer: A Option (A) is correct. Providing speech-reading therapy is a primary role for SLPs as it enhances the overall communicative competency for the client.

Which of the following benchmarks best aligns with current research on typical communication development for 3-year-old children? A.Understanding approximately 1,000 words B.Using irregular third-person-singular verb forms C.Having a mean length of utterance (MLUM L U) in morphemes of six D.Producing approximately ten consonant phonemes accurately

Correct Answer: A Option (A) is correct. Receptive vocabulary is reported at approximately 1,000 words for 3-year-old children.

Which of the following statements about dysphagia screening and assessment/evaluation is true? A.Screening identifies the likelihood of dysphagia and the need for further assessment. B.Screening identifies the nature and severity of dysphagia and enables treatment planning. C.Assessment/evaluation is a pass-fail procedure that determines whether or not a patient is aspirating. D.Assessment/evaluation can be performed by non-SLP observers.

Correct Answer: A Option (A) is correct. Screening is a method that allows for the identification of dysphagia and determines the need to assess the condition further

Which of the following is generally considered most effective and appropriate for viewing the vocal folds during phonation? A.Stroboscopy B.Endoscopy C.Fluoroscopy D.Laryngeal mirror examination

Correct Answer: A Option (A) is correct. Stroboscopy is generally the most efficient and effective instrumentation for viewing the vocal folds during phonation.

Which of the following activities is a principal component of the Lidcombe Program for childhood stuttering? A.Teaching caregivers to provide feedback to their child about the child's fluent and stuttered speech B.Teaching caregivers to support their child's communication attempts but to avoid acknowledging the child's fluency performance C.Teaching children to describe their emotional state to their caregiver when stuttering is anticipated on an upcoming word D.Teaching children to present nonverbal cues to their caregivers when stuttering is anticipated on an upcoming word

Correct Answer: A Option (A) is correct. The Lidcombe Program is considered a direct therapy approach in which primary caregivers explicitly provide feedback on the child's level of fluency. All other options are part of indirect therapy approaches.

An SLP working with a 45-year-old patient with acquired expressive communication deficits is emphasizing reengagement by focusing on realistic short-term goals of the patient's choice. The SLP is primarily using which of the following approaches to therapy? A.Life Participation Approach to Aphasia (LPAAL P A A) B.Visual Action Therapy (VATV A T) C.Supported Communication Intervention (SCIS C I) D.Prompts for Reconstructing Oral Muscular Phonetic Targets (PROMPTP R O M P T)

Correct Answer: A Option (A) is correct. The Life Participation Approach to Aphasia (LPAAL P A A) has the overriding philosophy to maximize clients' re-engagement in life and bases all decision making on the life concerns identified by the clients and their families.

Mr. Aviz, a 62-year-old chemist, is recovering from hip-replacement surgery following a spontaneous fracture. The surgery was completed without complications. He is now in his second postoperative day, and his wife reports alterations in his memory and some confusion not previously noted. Which of the following is the most reliable screening instrument for the SLP to use in this case? A.The Mini-Mental State Examination (MMSE) B.The Test of Problem Solving (TOPS-3) C.The Cognitive Linguistic Quick Test (CLQT) D.The Global Deterioration Scale (GDS)

Correct Answer: A Option (A) is correct. The MMSE is the only screening tool in the list appropriate for the age and condition described.

An SLP works with a patient who has severe dysphagia. The patient is on a pureed diet and honey-thick liquids. During treatment, the patient becomes tearful and expresses frustration. The patient says, "I feel like I'm not making any progress and I'm never going to eat normal food again. I hate this diet, and I hate doing therapy. I just want to give up." The SLP responds by saying, "What you're feeling is normal, and I understand. You're doing everything you can right now to get better, and giving up isn't the answer." The SLP reaches across the table and places a comforting hand on the patient's arm and has a caring facial expression. Which of the following principles of counseling is the SLP primarily demonstrating? A.Showing congruence B.Having unconditional positive regard C.Offering the patient sympathy D.Providing emotional support

Correct Answer: A Option (A) is correct. The SLP is demonstrating congruence by using words and body language that match.

The acoustic reflex is triggered in a person with typical hearing when the listener is exposed to a sound above approximately A.85 dB HTL B.130 dB HTL C.1000 Hz D.6500 Hz

Correct Answer: A Option (A) is correct. The acoustic middle ear reflex in a person with typical hearing usually occurs at about 85 dB HL.

An SLP is working with a 30-month-old child who is at the one-word production level. The goal of treatment is to facilitate growth in play as a precursor to the acquisition of two-word semantic relationships. To achieve the goal, the SLP can best begin by facilitating A.the use of pretend play, such as brushing a doll's hair or feeding a doll with a bottle B.the purposeful exploration of toys C.exploration, by mouthing of toys D.mean-end behaviors, such as pulling a string to get a toy

Correct Answer: A Option (A) is correct. The child needs to be encouraged to proceed to the two-word production stage, and the use of pretend play will encourage action/object two-word phrases such as "Feed baby" and "Brush hair." All of the other answer choices involve behaviors that do not necessarily promote linguistic behaviors.

Clinician: "What do you have there?" Child: "I had a big bus." Clinician: "Is that right?" Child: "Uh, I have a big bus." Clinician: "Now, is that right?" Child: "Yeah." What language-stimulation technique is the clinician using in the above exchange? A.Self-evaluation B.Rephrasing C.Reauditorization D.Parallel talking

Correct Answer: A Option (A) is correct. The clinician is using prompts to have the child self-evaluate his or her own statements.

An 82-year-old female patient is admitted to a skilled nursing facility following an acute hospital stay. Her diagnoses include urinary tract infection, frequent falls with subsequent hip fracture, and chronic obstructive pulmonary disease (COPD). No surgery was required for the hip fracture. Before admission to the hospital, she was living independently and able to complete all activities of daily living (ADLs) and instrumental activities of daily living (IADLs) on her own. She is referred to the facility SLP for cognitive screening because she has difficulty carrying over new information, confusion regarding weight-bearing status, and difficulty processing directions. The SLP administers the Montreal Cognitive Assessment, and the patient scores 17/30. The patient's main deficits are in the areas of short-term memory, executive functioning, and planning. After the patient is appropriately treated both medically and therapeutically, cognitive deficits remain and the interdisciplinary team does not recommend that the patient return home independently. The patient's family is frustrated with this recommendation and seeks input from other sources. Which of the following treatment procedures is most appropriate for the patient? A.Engaging in spaced retrieval B.Naming divergent items C.Following specific directions D.Completing word searches

Correct Answer: A Option (A) is correct. The patient's goal is to return to prior level of function (PLOFP L O F). The patient was able to live independently before her admission, so spaced retrieval techniques will help her to return to her previous level of memory ability or at least teach her compensatory strategies to assist with her memory loss, since she does not yet have a diagnosis of dementia.

A large metropolitan school district wants to determine the prevalence of developmental stuttering among all enrolled students during the past year. Which of the following approaches is most appropriate for accomplishing the task? A.Dividing the total number of students who currently stutter by the total number of students who were enrolled during the past year B.Subtracting the total number of students who received treatment for stuttering during the past year from the total number of students who currently stutter C.Multiplying the total number of students who currently stutter by the total number of students enrolled during the past year D.Dividing the total number of newly identified students who stutter by the total number of students enrolled during the past year

Correct Answer: A Option (A) is correct. The prevalence of stuttering is the percentage found by dividing the number of students who currently stutter by the number of students enrolled in school.

An SLP provides home practice for a patient with aphasia as part of discharge plans. The patient is approximately six-months post left-hemisphere stroke that resulted in aphasia. Verbal output has improved significantly since the stroke; however, the patient is still very concerned that written expression is moderately impaired at the single-word level. The homework assignment is to implement copy-and-recall treatment to improve written expression, which involves practicing writing target words (from photographs) and then copying those words multiple times. However, during a follow-up phone call, the SLP determines that the patient is struggling to write the assigned target words because the patient "can't think of the correct letters." Although a model is provided on the back of each target photograph, the patient wants to find a way to practice without looking at the answer. The SLP's best course of action is to suggest that the patient A.try anagramming the words using letter tiles to form the target word B.practice writing the alphabet in order multiple times before beginning the homework again C.use a cell phone voice-to-text application as a compensatory strategy D.use drawing to convey messages instead of using written expression

Correct Answer: A Option (A) is correct. This approach would give the client the best access to completing the homework. The letter tiles would provide a support for the client to complete the homework and are a regular part of this treatment approach

According to empirical research with people who stutter, which TWO of the following are true regarding the age of symptom onset for most cases? A.It is usually earlier for girls than for boys. B.It is similar for girls and boys. C.It most often occurs in the range of 2 to 5 years old. D.It most often occurs in the range of 6 to 9 years old

Correct Answer: A, C Options (A) and (C) are correct. Research findings indicate that the age of onset for stuttering symptoms is somewhat earlier for girls than for boys, and in most cases, symptom onset occurs in the preschool years.

Place the phonemes below in the order of typical acquisition. /p/ /θ/ /k/ /ʒ/

Correct Answer: A, C, B, D /p/ is first because this phoneme is developed between ages 1 and 3. /k/ slash is second because this is developed by age 3 1/2. /θ/ is third because this is developed between ages 3 and 8. /ʒ/ is last because this is developed between ages 6 and 8.

Which THREE of the following are included in a pragmatic language assessment during conversation? A.New topic initiation B.Speech intelligibility C.Discourse cohesion D.Repair strategies E.Type-token ratio

Correct Answer: A, C, D Options (A),(C) and (D) are correct. Initiating new topics is an important piece when conversing with another person. A pragmatic assessment would analyze this area of conversation. Discourse cohesion shows a child's understanding of not interrupting and responding when appropriate. Using repair strategies will provide the assessor with data related to the child's ability to respond based on what the conversational partner said.

Which of the following sounds are typically mastered by the time a child turns 3? A./k/ and /g/ B./p/ and /b/ C./l/ and /s/ D./ch/ and /sh/

Correct Answer: B Option (B) is correct. /p/ and /b/forward slash p forward slash and forward slash b forward slash should be mastered by age 3 in a typically developing child.

Which dental condition is most likely to have a negative effect on articulation? A.Class II malocclusion B.Class III malocclusion C.Missing maxillary lateral incisors D.Missing mandibular central incisors

Correct Answer: B Option (B) is correct. A class III malocclusion has the most negative effect on articulation, as it can affect all anterior sounds.

An elderly patient with multiple diagnoses including Parkinson's disease (PD) presents to a home health SLP for treatment. The SLP recommends that the patient follow a mechanical soft diet with thin liquids, alternate bites and sips, take small bites, and have supervision when eating. Throughout the course of treatment, the patient exhibits a 12 percent weight loss with variable intake levels. To which of the following professionals is it most appropriate for the SLP to refer the patient? A.A gastroenterologist, for possible feeding tube placement B.A dietitian, for nutrition management C.An ENT specialist, for possible esophageal deficits D.A neurologist, for assessment of progression of PD

Correct Answer: B Option (B) is correct. A dietitian will be able to assess caloric intake and recommend supplements and other ways for the patient to maintain a healthy weight.

After the patient completes treatment, the most important referral for the SLP to make is to a A.psychiatrist B.neurologist C.gastroenterologist D.gerontologist

Correct Answer: B Option (B) is correct. A neurologist can evaluate and diagnose for dementia and other neurodegenerative diseases that this patient may have. It is accepted practice that SLPs will refer to neurology when a dementia diagnosis is suspected, as dementia is a neurological disorder.

A patient is diagnosed with a cerebrovascular accident of the temporal lobe. The patient exhibits deficits in repetition, written expression, and auditory comprehension. In addition, literal paraphasias are noted. These findings are consistent with what type of aphasia? A.Broca's B.Conduction C.Anomic D.Globa

Correct Answer: B Option (B) is correct. A temporal lobe stroke and the symptoms described are typical of a patient with conduction aphasia.

Mr. Lewis, a fourth-grade teacher, has referred a 9-year-old student to the school speech-language pathologist (SLP) because he is concerned that the student's speech is interfering with spelling performance. The SLP observes that the student substitutes the voiceless labiodental fricative for the voiceless interdental fricative in the word-final position and that the student reverses /s/ and /k/forward slash s forward slash and forward slash k forward slash in words such as "ask." Which of the following areas needs to be considered first? A.Auditory discrimination B.Dialectal differences C.Disordered phonological system D.Dyslexia

Correct Answer: B Option (B) is correct. An f/"th"f forward slash t h substitution as well as the reversal of "sk"s k in "ask" are common characteristics of a dialectal difference.

A transfer student is referred for a language evaluation. The student has spoken English for less than a year, and English is not the primary language used at home. To provide the student with culturally and linguistically appropriate service delivery, the student should be evaluated A.using norms for standard English speakers B.in the student's native language C.with the student's parents present D.based on formal assessments

Correct Answer: B Option (B) is correct. Assessment should be administered in the student's native language for accurate results and appropriate service delivery.

Which of the following is a distinguishing characteristic of articulation therapy for a 2-year-old patient with a repaired cleft palate and compensatory errors? A.Cycling through multiple speech goals in a given session B.Having a play-based learning approach to address goals C.Focusing on carryover of speech productions D.Addressing speech targets with high frequency and intensity

Correct Answer: B Option (B) is correct. Based on the age of the child, the only appropriate treatment listed is a play-based approach. All other treatments listed are inappropriate for the age of the child.

During an examination of his oral mechanism, an adult male is asked to pucker his lips and then to spread them into a wide smile. This ability depends on the bilateral integrity of which of the following cranial nerves? A.CN V B.CN VII C.CN IX D.CN XII

Correct Answer: B Option (B) is correct. CN VIIroman numeral 7 (the facial nerve) is the major nerve innervating the described physiological action of the lips.

As part of a study, the speech of several subjects with Wilson's disease is evaluated using an acoustic analysis system. The same subjects are assessed again, but with a different system from another manufacturer. Which of the following can be assessed by comparing the two data sets? A.Content validity B.Concurrent validity C.Test-retest reliability D.Interjudge reliability

Correct Answer: B Option (B) is correct. Concurrent validity refers to the extent to which the results of a particular test, or measurement, correspond to those of a previously established measurement for the same construct.

Children with semantic-pragmatic language disorders often have difficulty learning deictic words because such terms A.require an understanding of indirect directives B.take the perspective of the communication partner C.refer to object permanence D.have many alternate forms

Correct Answer: B Option (B) is correct. Deictic terms are words whose meaning shifts depending on the point of view of the speaker; difficulties with such terms is a core feature of semantic-pragmatic language disorders.

A patient is referred to speech therapy after a total laryngectomy. The SLP teaches the patient to speak by taking air in through the mouth, trapping it in the throat, and then releasing it. Which of the following treatments is the SLP primarily implementing with the patient? A.Tracheoesophageal puncture B.Esophageal speech C.Neck electrolarynx D.Intraoral artificial larynx

Correct Answer: B Option (B) is correct. Esophageal speech focuses on using the esophagus to provide speech, and no devices are used.

A 6-year-old child who is a bilingual English-Spanish speaker produced the following statements during a speech-language assessment. I cutted the finger. I played with her yesterday. She eated too much candy. You like ice cream? Maria is going? Father is happy. Buy a new car. Based on the sample, the speech-language pathologist should begin remediation by focusing on A.adjectives and imperatives B.irregular verbs C.prepositional phrases D.conjunctions and embedding

Correct Answer: B Option (B) is correct. Given the language sample, the child incorrectly recognizes the past tense forms of irregular verbs

It is most appropriate for a SLP to treat hyperadduction of the vocal folds by having the client A.increase muscular effort through pushing or lifting while vocalizing B.use light and gentle vocal-fold contacts C.attempt quick onset of phonation D.use drills with exaggerated contrastive stress on words

Correct Answer: B Option (B) is correct. Having the client use light and gentle vocal-fold contacts will help to reduce tension and thus would be effective in treating hyperadduction of the vocal folds. All of the other answer choices listed would serve to increase tension.

An SLP administers a language test to all kindergarten children in a particular school and finds that 40 percent fall below the tenth percentile of the normative sample. Which of the following is the most reasonable interpretation of this finding? A.Forty percent of the kindergarten children in the school have language disorders. B.The school's kindergarten population differs from the standardization population in the language skills measured. C.The SLP administered the test incorrectly. D.The standardization population is not representative of the nation as a whole.

Correct Answer: B Option (B) is correct. If disparities exist between examinees and the norm group in terms of skills and experiences, the conclusions based on the examinees' test performance may be misleading.

A 90-year-old female patient residing in a long-term care facility is referred to the facility SLP for a swallow evaluation. The patient has advanced dementia, is under hospice care, and is currently on a pureed diet with nectar-thick liquids. During the evaluation, the SLP finds that the patient is showing signs and symptoms of aspiration on all consistencies, including pudding-thick liquids. The patient's cognitive condition makes an instrumental exam inappropriate. The SLP's recommendation is nothing by mouth (NPON P O) except for pleasure. The family and patient have decided against feeding-tube placement. Which of the following is a primary ethical responsibility of the SLP in this situation? A.Asking the family what diet level they would like the patient to consume B.Making a recommendation based on clinical judgment C.Deferring to a doctor or nurse to help with the decision D.Refusing to write diet orders for the patient because of the NPON P O recommendation

Correct Answer: B Option (B) is correct. In this situation the role of the SLP is to keep the patient as safe as possible. Because the family does not want to pursue a feeding tube, the SLP has to choose the diet that is most appropriate for the patient.

Following placement of a tracheostomy tube, a patient recovering from cardiothoracic surgery is successfully weaned from mechanical ventilation. A day later the SLP receives a consult to assess patient candidacy for using a one-way tracheostomy valve. Which of the following observations is the most important contraindication for safe and successful patient tolerance of the one-way valve? A.Oxygen saturation below 95% B.Ability of patient to pass air to the oral cavity while exhaling with the tube cannula occluded by the SLP C.Patient ability to expectorate lung secretions without suctioning D.Patient tolerance of the deflated cuff

Correct Answer: B Option (B) is correct. Inability to pass air to the oral cavity with expiratory occlusion represents an upper-airway obstruction, which is a contraindication for use of a one-way tracheostomy valve

An 81-year-old female presents to a rehabilitation hospital following a severe L MCA stroke. The patient has poststroke right-sided weakness, right neglect, and suspected nonfluent aphasia. Receptive language appears to be mildly impacted. The patient has a complex medical history, including TBI following a car accident two years ago, obesity, diabetes, seizure disorder, congestive heart failure, and hypertension. Socially, the patient has lived alone for the past three years following her husband's death. Since her car accident she mainly watches TV at home and leaves her house only for dialysis. She has meals on wheels and home health aides to assist her with showering. Because of her TBI, her behavior is often variable and erratic. The highest level of education she attained was seventh grade. She stayed home to take care of her children and did not have a career. Which of the following medical factors should the SLP give the most weight when determining the patient's communication prognosis? A.Frequency of seizures B.Severity of the stroke C.Level of obesity D.Severity of diabetes

Correct Answer: B Option (B) is correct. Information about the severity of the patient's stroke is necessary to estimate the patient's chance of recovery.

Question: Six months ago, an SLP evaluated 4-year-old Molly's speech fluency during conversation. At that time, she displayed physically relaxed repetitions of words and phrases (occurring at a frequency of 2 per 100 words), and interjections such as "um" (occurring at a frequency of 1 per 100 words). She did not display any sound prolongations or facial grimaces; she did not produce any pitch rises or phonatory breaks; and she did not appear to avoid any sounds or words. Results from several formal tests suggested that her articulation and language development were within normal limits. Molly reportedly began producing repetitions and interjections at age 24 months, and the frequency of these disfluency types reportedly has remained stable since then. The SLP did not recommend speech-language intervention following the previous evaluation; however, she did provide the parents with information about fluency development, symptoms of stuttering, and general suggestions for how to facilitate children's fluency. A reevaluation is scheduled for next week. Which of the following is most appropriate for the SLP to do if Molly's speech fluency has remained the same since the previous evaluation? A.Recommend that Molly immediately begin fluency therapy, in which the focus is on reducing the frequency of repetitions and interjections in her conversational speech B.Recommend that Molly be released from the SLP's active caseload C.Recommend that Molly be referred for psychological counseling, with a focus on helping Molly improve speech fluency by learning how to manage anxiety more effectively D.Recommend monthly evaluations of Molly's speech fluency until she is five years old

Correct Answer: B Option (B) is correct. Molly's fluency development was within normal limits at the previous evaluation, and, based on the parents' report, it also seemed to be within normal limits at age 2. Further, no other concerns about Molly's communication development were mentioned in the scenario. Thus, it appears that Molly has never stuttered and her communication skills have been and continue to be within normal limits. Therefore, it is unnecessary to reevaluate her speech or enroll her in fluency therapy.

Sensorineural hearing loss resulting from Ménière's disease causes A.a sudden hearing loss without warning B.fluctuating levels of hearing loss C.complete hearing loss at all frequencies D.bilateral progressive hearing loss

Correct Answer: B Option (B) is correct. Ménière's disease is a disorder of the inner ear and causes fluctuating hearing loss

For a patient with potential pharyngeal phase dysphagia and pooling of secretions following open-heart surgery with suspected recurrent laryngeal nerve damage, which of the following instrumental examinations for swallowing provides the most direct view for evaluating the patient based on history and suspected difficulties? A.Upper gastrointestinal (GIG I) series B.Fiberoptic endoscopic evaluation of swallowing (FEESF E E S) C.Videofluoroscopic swallowing study (VFSS) D.Videostroboscopic examination of vocal folds

Correct Answer: B Option (B) is correct. Recurrent laryngeal nerve damage during open-heart surgery would only involve the left vocal fold. This swallowing instrumental examination allows for both assessment for pharyngeal phase dysphagia, by presenting foods and liquids during the assessment procedure, and a direct view of the functioning of both vocal folds, as well as determining the pooling of secretions.

The commonly used chin-down posture was initially developed to eliminate thin-liquid aspiration in people with delayed pharyngeal stage onset after having a stroke, and when its efficacy was investigated, it was found to be 50% effective. For whom is the chin-down posture ineffective at eliminating thin-liquid aspiration? A.People who have cricopharyngeal dysfunction B.People who aspirate residue from the pyriform sinuses C.People who have impaired lingual function D.People who aspirate residue from the valleculae

Correct Answer: B Option (B) is correct. Shanahan et al. (1993) found that 50 percent of people with stroke- and swallowing-related aspiration due to delayed onset of the pharyngeal response continued to aspirate from the pyriform sinuses using the chin-down posture. Moreover, Eisenhuber et al. (2002) found that people with a pyriform sinus residue with a height that was less than 50 percent of the pyriform sinus height aspirated significantly more than those with less or no pyriform sinus residue.

Management of which one of the following examination observations alone will produce the best improvement in the pneumonia risk of an adult patient with chronic dysphagia? A.Poor oral bolus control and containment B.Poor oral hygiene and dependence for oral care C.Delayed onset of the pharyngeal stage of swallowing D.Impaired distension of the upper esophageal sphincter

Correct Answer: B Option (B) is correct. Studies have identified poor oral hygiene (including presence of decayed teeth) and dependency for oral care as significant independent predictors of pneumonia in people with chronic dysphagia.

Which of the following tools is most appropriate to evaluate Alan's functional communication skills? A.Western Aphasia Battery-Revised B.Communication Activities of Daily Living-Second Edition C.Boston Diagnostic Aphasia Examination-Third Edition D.Test of Everyday Attention

Correct Answer: B Option (B) is correct. The CADL-2 is the best tool for evaluating functional communication skills in people with neurological communication disorders. It measures receptive and expressive language, as well as social use of language, gestures, and humor.

Minimal word pairs are typically used in phonological treatment in order to A.stabilize phonological patterns in a system B.build awareness of the phonetic realizations of phonological contrasts C.probe generalization of taught sound patterns to words that have not been used in instruction D.facilitate automatic production of words

Correct Answer: B Option (B) is correct. The erroneous sound is contrasted with the correct sound to encourage the contrasting of the sound production and word meaning or homonym.

A fourth-grade student presents to a school SLP with receptive-expressive language disorder. The student has difficulty with asking "wh-" questions. Specifically, the student tends to ask only "who" and "what" questions. When probed, the student will ask "why" and "when" questions but often uses the terms incorrectly. When writing the annual Individualized Education Program for the student, which of the following goals is most appropriate and measurable? A.The student will ask six related "wh-" questions with 100% accuracy. B.The student will ask six related "wh-" questions with minimal cueing in two out of three tries. C.The student will ask six related "wh-" questions with no cueing and with 100% accuracy. D.The student will ask six related "wh-" questions with no cueing in one out of three tries.

Correct Answer: B Option (B) is correct. The goal is specific, measurable, realistic, and time limited and addresses the student's specific needs.

During an examination of the oral peripheral mechanism of an adult who has had a right hemisphere stroke in the territory of the middle cerebral artery, testing for lingual motor function reveals protrusion of the tongue to the left of midline. Which of the following is true? A.The patient has a lesion affecting the left hypoglossal nucleus. B.The patient exhibits unilateral left lingual weakness. C.Left vocal fold paralysis should be expected because of the site of the lesion. D.The right hypoglossal nerve is damaged.

Correct Answer: B Option (B) is correct. The right side of the tongue is stronger and forces (or pushes) the tongue to the left. The imbalance in strength causes the tongue to deviate from midline to the left upon protrusion, hence left lingual weakness.

An SLP tries to elicit correct production of the target sound /s/forward slash s forward slash. Which of the following words provides the best coarticulatory conditions to meet the SLP's goal? A.Spoon B.Sea C.Sob D.Soup

Correct Answer: B Option (B) is correct. The target sound /s/ and the vowel /i/ that follows it both require lip spreading. The vowel /i/ is high and front.

Which of the following structures is a relay center for sensory information? A.The reticular formation B.The thalamus C.The putamen D.The hippocampus

Correct Answer: B Option (B) is correct. The thalamus is the major relay system for sensory information.

Which of the following best represents an interprofessional treatment model? A.The SLP designs an intervention plan to be implemented by an SLP assistant. B.The SLP and general education teacher plan weekly language activities that they deliver jointly to the classroom. C.The SLP conducts the evaluation and the team designs the treatment program. D.Two district SLPs design language screening tools for the Response to Intervention (RTIR T I) team.

Correct Answer: B Option (B) is correct. This is the only option that involves two different professions working together on the same area or issue.

A 7-month-old infant presents to an SLP for a swallow evaluation prompted by parental concerns related to the child coughing and choking frequently when swallowing thin liquids. The parents have attempted to feed the child small amounts of pureed foods, but the child spits them out. Which of the following developmental milestones is a typically developing 7 month old most likely to demonstrate during a swallow evaluation? A.Showing the ability to manipulate food B.Creating vertical jaw movements C.Using the rooting reflex when food is seen D.Engaging tongue elevation patterns

Correct Answer: B Option (B) is correct. Using jaw movements to initiate chewing begins to emerge around 5 to 7 months, and it would be appropriate for the SLP to assess whether the child has begun to use this skill.

Which of the following strategies is most appropriate for an SLP to try in an effort to minimize the perception of mild nasal emission in a patient? A.Increasing fundamental frequency B.Using light articulatory contacts C.Reducing mouth opening D.Increasing rate of speech

Correct Answer: B Option (B) is correct. Using light articulatory contacts can help decrease perception of nasal emissions.

During a clinical swallow assessment by an SLP, a patient was noted to have the following symptoms. An immediate cough after swallowing thin liquids Prolonged mastication with regular-consistency foods No coughing when fed thickened liquids Postswallow residue Which of the following recommendations is most appropriate for the patient in this situation? A.Neuromuscular electrical stimulation B.Modified barium swallow study C.High-resolution pharyngeal manometry D.Cervical auscultation of swallowing

Correct Answer: B Option (B) is correct. When reasonable suspicion of clinically significant dysphagia is present after a clinical examination and the cause is potentially undefinable without imaging, an imaging study is necessary; a modified barium swallow study is the only imaging study in the list.

Which of the following diagnoses is most appropriate for this patient? A.Cognitive communication deficit B.Other symbolic dysfunction C.Age-related cognitive decline D.Mild cognitive impairment

Correct Answer: B Option (B) is correct. When treating a patient for a cognitive impairment not caused by a stroke, SLPs should use a diagnosis of other symbolic dysfunction.

A 5-year-old patient presents to an SLP with severe apraxia of speech. The SLP has been intensively working with the patient for two years, and the patient has not demonstrated any progress in a month. The SLP recommends a speech-generating device for the patient to use at home and at school, but the insurance company denied the request. Which of the following actions best allows the SLP to advocate for the patient to receive a device? A.Teaching the parent SLP terminology to use when talking to the insurance company B.Writing a letter to the insurance company to refute the basis for the denial C.Asking the parent to buy the device without approval and apply for reimbursement D.Sending a letter to the school's special education teacher asking the teacher to contact the insurance company on the SLP's behalf

Correct Answer: B Option (B) is correct. Writing a letter to the insurance company is within the scope of practice for an SLP to ensure coverage for speech-language therapy.

Place the names of the following theorists with their associated concepts. Piaget Skinner Vygotsky Chomsky

Correct Answer: B, C, D, A Discriminative stimulus is associated with Skinner. Zone of proximal development is associated with Vygotsky. Language generativity is associated with Chomsky. Object permanence is associated with Piaget. Any other pairings would be incorrect.

Place the following aspects of phonological awareness in developmental order starting with the earliest skill to emerge. Listing words that start with the same sound Recognizing words that rhyme Counting syllables in single words Creating words by blending onset and rime

Correct Answer: B, C, D, A Recognizing words that rhyme is first because this occurs between 30 and 36 months of age. Counting syllables in single words is next because this occurs by 4 years of age. Blending words into rime is third because this occurs near 5 years of age. Listing words that start with the same sound is last because this occurs between 6 and 7 years of age. Any other order is incorrect because phonological awareness builds on itself and it is necessary to master each area prior to moving on to the next.

An SLP assesses a 3-year-old child with poor intelligibility. The SLP attempts to determine whether the child has a severe phonological disorder or childhood apraxia of speech (CAS). Which TWO of the following characteristics are primarily indicative of CAS? A.Presenting with decreased strength and muscle coordination B.Displaying inconsistencies in articulation performance C.Making the same errors independent of length of speech utterance D.Speaking with a disrupted rate, rhythm, and stress of speech

Correct Answer: B, D Options (B) and (D) are correct. Children with CAS struggle to make consistent productions of speech sounds. Children with CAS speak with a rate, rhythm, and stress of speech that can be disrupted, and sometimes they can appear to be groping for placement.

Which of the following is an example of a modeled trial? A."This is the opposite of a cat. What is this?" B."What animal is this a picture of?" C."This is a dog. Can you say the word 'dog'?" D."This is a picture of a /da.../.forward slash d a ellipses forward slash"

Correct Answer: C Option (C) is correct. A modeled trial consists of the client's being told the correct answer (This is a dog) and then the client's being asked to imitate the answer.

SLPs often have a responsibility to communicate with the parents of children with severe disabilities. According to mourning theory, when are parents normally most receptive to information and advice provided by professionals regarding their child? A.When the parents are working through their feelings about the child's disabilities B.When the parents fully realize the extent of the child's disabilities and the limitations of treatment and education C.When the parents have acquired greater confidence in their capacity to care for the child and greater motivation to cope with the child's disabilities D.When the parents are making decisions regarding future care and protection of the child

Correct Answer: C Option (C) is correct. According to mourning theory, a grieving person is most receptive to new information about the source of grief after having just entered the recovery stage.

Compensatory swallowing maneuvers and interventions are designed to mitigate a biomechanical impairment during the swallow; however, some of them have unintended consequences that have been documented in scientific literature. Which of the following swallow maneuvers has been found to produce cardiac arrhythmia (irregular heartbeat) in patients with stroke or cardiovascular diseases? A.Mendelsohn maneuver B.Chin-down posture C.Supraglottic swallow maneuver D.Head rotation toward the weak side

Correct Answer: C Option (C) is correct. Chaudhuri et al. (2002) found that 86 percent of people with history of stroke and/or cardiovascular disease developed cardiac arrhythmias while performing the supraglottic or super-supraglottic swallow, including supraventricular tachycardia and premature atrial and/or ventricular contractions

A 23-year-old client recently completed an intensive 3-week-long summer program for stuttering and now exhibits stuttering symptoms on fewer than 3 percent of syllables during both in-clinic conversations with the SLP and beyond-clinic conversations with family members. The SLP is concerned that the client might relapse now that intervention has ended and wishes to enact a plan to help the client maintain fluency gains. Which of the following plans is the most likely to result in maintenance of the fluency gains? A.Recommending that the client participate in an annual intensive review of stuttering management skills B.Recommending that the client practice fluency management skills during monthly telephone calls with the SLP C.Ensuring the client understands the need for continued use of stuttering management techniques and referring to a local SLP for ongoing treatment D.Scheduling the client for reenrollment in the intensive 3-week-long intervention program

Correct Answer: C Option (C) is correct. Continuing to regularly participate in scheduled intervention activities under the direction or supervision of an SLP is essential to maintaining fluency gains. It provides a client with the opportunity to address ongoing fluency challenges or new ones that may arise after completion of the intensive intervention program. This type of continued treatment does not necessarily imply that the client must participate in ongoing weekly or intensive intervention.

Early intentional communication typically emerges in the months leading up to a child's first birthday. Which of the following communicative functions typically emerges first in this period? A.Requesting B.Commenting C.Protesting D.Greeting

Correct Answer: C Option (C) is correct. Early intentional communication develops around 8 to 10 months of age, and the first function that emerges is protesting.

The Family Educational Rights and Privacy Act (FERPA) guarantees parents access to their child's educational records. However, this mandate does not apply to the daily records kept by an SLP working in a school setting if these records A.are not used for treatment-related reimbursement from state or local government B.remain within the school district in which the SLP works C.are kept in the sole possession of the SLP D.are not distributed to anyone outside of the child's school

Correct Answer: C Option (C) is correct. FERPA states that daily notes can be kept in the sole possession of an SLP as long as the purpose is to serve as a "memory jogger" for the creator of the record

Which of the following speech-sampling contexts best assesses hyponasality? A.Producing sentences with oral sonorants B.Counting in numerical order from 60 to 70 C.Saying single words with nasal consonants D.Repeating words with oral-pressure consonants

Correct Answer: C Option (C) is correct. Hyponasality (too little nasal resonance) is most easily detected during the production of nasal consonants.

A 72-year-old right-handed female is evaluated following a left-hemisphere stroke. Findings reveal a moderate Broca's aphasia. Which of the following is the primary goal of intervention for the client? A.To improve the client's pragmatics skills and abilities B.To increase the client's ability to express functional comprehension abilities C.To improve the client's ability to express production of syntactically appropriate sentences D.To enable the client's ability to use a digital augmentative communication system

Correct Answer: C Option (C) is correct. Improvement of expression, particularly syntax, is the most appropriate focus of treatment for Broca's aphasia.

Which of the following is an example of a reversible passive? A.The balloon was broken by the pin. B.The kite was flown by Ben Franklin. C.Jared was bullied by Michael. D.The girl put her doll by her friend.

Correct Answer: C Option (C) is correct. In reversible passive constructions, participants can fit into either role.

Patient is a 77-year-old female with a left hemisphere stroke. Sensorimotor clinical examination revealed right lower facial droop, impaired oral sensation, absence of dentition, need for frequent prompts to engage the patient in evaluation procedures, and aphasia. Bedside swallow evaluation with pureed food and nectar-thick liquids revealed food remnants in right lateral sulcus, drooling from the right oral cavity, and intermittent cough that was not present before the swallow evaluation. Prior medical history includes COPDC O P D, diabetes, and congestive heart failure. Patient does not want an instrumental examination. Based on the data above, which of the following is most likely to lead to a clinically significant dysphagia-related adverse event? A.Impaired oral sensorimotor function B.Prior medical history C.Impaired level of alertness D.Aphasia

Correct Answer: C Option (C) is correct. It is extremely important that the patient is alert and as upright as possible when addressing swallowing and trying out different foods.

Which of the following is characteristic of a mild traumatic brain injury? A.Long-term memory loss B.Onset of frequent seizures C.Memory loss of events immediately preceding or following the trauma incident D.Extensive and permanent neurological impairments

Correct Answer: C Option (C) is correct. Loss of memory of events immediately preceding or following the traumatic incident is a characteristic of a mild traumatic brain injury.

To best facilitate the functional and meaningful use of linguistic forms, a language-intervention program for a child with language impairments should A.focus on comprehension tasks that are just above the child's linguistic level B.stress structured, game-like situations that use imitative tasks C.present and elicit the forms in naturalistic contexts D.stress imitation of the forms to be learned

Correct Answer: C Option (C) is correct. Naturalistic contexts—everyday situations—provide opportunities for the use of functional and meaningful linguistic forms.

Which of the following strategies can the SLP use to most effectively engage the patient in monitoring his progress? A.Recording their sessions for the patient to listen to later B.Reinforcing that the aphonia is not the patient's fault C.Helping the patient recognize his control over his vocal quality D.Asking the patient to keep a journal of why he feels he cannot use his voice

Correct Answer: C Option (C) is correct. Once the patient understands that he is in control and can use his voice, he will be more likely to use his voice consistently.

An 81-year-old female presents to a rehabilitation hospital following a severe L MCA stroke. The patient has poststroke right-sided weakness, right neglect, and suspected nonfluent aphasia. Receptive language appears to be mildly impacted. The patient has a complex medical history, including TBI following a car accident two years ago, obesity, diabetes, seizure disorder, congestive heart failure, and hypertension. Socially, the patient has lived alone for the past three years following her husband's death. Since her car accident she mainly watches TV at home and leaves her house only for dialysis. She has meals on wheels and home health aides to assist her with showering. Because of her TBI, her behavior is often variable and erratic. The highest level of education she attained was seventh grade. She stayed home to take care of her children and did not have a career. Which of the following medical conditions, in combination with her stroke, is likely to have the greatest impact on the patient's communication abilities? A.Seizure disorder B.Diabetes C.TBI D.Obesity

Correct Answer: C Option (C) is correct. Patients with TBIT B I are more likely to have metacognitive problems in addition to the aphasia, thus impacting their ability to improve.

Which of the following conditions is singularly caused by a genetic abnormality? A.Cleft lip and palate B.Specific language impairment C.Prader-Willi syndrome D.Cerebral palsy

Correct Answer: C Option (C) is correct. Prader-Willi syndrome is directly linked to missing genes on chromosome 15.

Which of the following statements best indicates a screening tool with high sensitivity? A.The majority of the children who passed the screening were found to have a communication disorder. B.Many of the children who passed the screening did not have a communication disorder. C.All of the children who failed the screening were found to have a communication disorder. D.Most of the children who failed the screening did not have a communication disorder.

Correct Answer: C Option (C) is correct. Sensitivity is the success rate in identifying all of the children who truly have a disorder.

An SLP is evaluating a preschool child referred for concerns regarding stuttering. In which of the following areas is performance likely to be weaker in a preschooler who stutters than in a typical preschooler? A.Hearing acuity B.Voice C.Language D.Swallowing

Correct Answer: C Option (C) is correct. Some young children who stutter present with concomitant language delay/disorder. School-based SLPs report that among the children to whom they provide stuttering services, a significant percentage are also receiving language services. Some studies with preschoolers and children in early elementary grades who stutter have shown evidence of group-level deficits in various aspects of language functioning. Thus, in a number of studies, a significant subset of children who stutter have shown language skills that are weaker than those of typical children.

Which of the following strategies to treat compensatory articulation errors is most appropriate for a child with hypernasality and glottal stop substitutions following surgery for velopharyngeal insufficiency? A.Using oral motor exercises B.Using a straw to elicit /s/forward slash s forward slash C.Using tactile cues to elicit phonemes D.Using negative practice

Correct Answer: C Option (C) is correct. Tactile cues are used to elicit correct articulation placement.

For a patient's use of consonant cluster reduction, the SLPS L P would most appropriately recommend intervention, including words such as A.knight B.laugh C.stop D.wish

Correct Answer: C Option (C) is correct. The /st/forward slash s t forward slash in "stop" is the only cluster occurring in the word choices.

Which of the following formants typically characterizes a high vowel? A.High-frequency second formant (F2) B.High-frequency first formant (F1) C.Low-frequency first formant (F1) D.Low-frequency second formant (F2)

Correct Answer: C Option (C) is correct. The F1 is inversely associated with tongue height, such that high vowels tend to have low F1 frequencies.

An SLP designs a series of treatment activities for Jake, a 4 year old who presents with severely delayed phonological development. During one of the activities, the SLP asks Jake to say various word pairs in which two phonologically dissimilar target sounds, both of which are absent from the child's phonetic inventory, are contrasted (for example, "chip" and "rip"). Which of the following choices best describes the treatment approach that is being used? A.Multiple-oppositions approach B.Minimal-pairs approach C.Maximal-oppositions approach D.Metaphonological approach

Correct Answer: C Option (C) is correct. The maximal-oppositions approach contrasts two errored sounds differing across place, manner, and voicing to gain the greatest amount of generalization.

The head lift exercise published by Shaker et al. (2002) was designed to produce which of the following changes in swallow biomechanics? A.Increase in laryngeal closure duration B.Strengthening of the base of the tongue C.Increase in diameter and duration of the upper esophageal sphincter (UESU E S) opening D.Prolonged relaxation of the cricopharyngeal portion of the inferior constrictor muscle

Correct Answer: C Option (C) is correct. The results of Shaker et al. (2002) demonstrated a significant (p<.01p left angle bracket period zero one) increase in the opening duration of the UESU E S.

When assessing the ability of a student with cerebral palsy to access an augmentative and alternative communication device, which of the following physical factors must be evaluated first? A.Upper extremity range of motion B.Lower extremity strength C.Trunk stability and control D.Fine motor dexterity

Correct Answer: C Option (C) is correct. Trunk stability will guide the clinician in making an appropriate recommendation for the type of device a client can access as it influences more distal movements.

Which of the following phonological processes is a child expected to suppress by 3 years of age? A.Displaying stopping patterns B.Using vocalization substitutions C.Exhibiting weak syllable deletions D.Having consonant cluster reductions

Correct Answer: C Option (C) is correct. Weak syllable deletion disappears before age 3 in normally developing children.

An 81-year-old female presents to a rehabilitation hospital following a severe L MCA stroke. The patient has poststroke right-sided weakness, right neglect, and suspected nonfluent aphasia. Receptive language appears to be mildly impacted. The patient has a complex medical history, including TBI following a car accident two years ago, obesity, diabetes, seizure disorder, congestive heart failure, and hypertension. Socially, the patient has lived alone for the past three years following her husband's death. Since her car accident she mainly watches TV at home and leaves her house only for dialysis. She has meals on wheels and home health aides to assist her with showering. Because of her TBI, her behavior is often variable and erratic. The highest level of education she attained was seventh grade. She stayed home to take care of her children and did not have a career. Which of the following tasks is most appropriate to include when assessing the patient's phrase length? A.Completing divergent naming tasks B.Maintaining a conversation with a known partner C.Responding to open-ended questions D.Repeating sentences of varying lengths

Correct Answer: C Option (C) is correct. When an SLP assesses phrase length, it is important to task the patient with open-ended questions to assess the patient's ability to spontaneously produce speech.

A clinician is starting treatment with a new client with specific language impairment (SLIS L I). Which of the following is the most appropriate source of information for the SLP to use in developing the treatment plan? A.Advice from a senior colleague in the speech clinic B.An all-day seminar presented by a leading clinician in the area of SLIS L I C.A peer-reviewed article describing a large-scale study comparing several methods of treatment for SLIS L I D.A conference presentation of unpublished cutting-edge research comparing several methods of treatment for SLI

Correct Answer: C Option (C) is correct. When an SLP is looking for information to develop a lesson plan for a client with SLIS L I, the best source of information is a peer-reviewed article that focuses on a large-scale study and compares several treatment methods

Which of the following recommendations to a parent of an infant with cleft lip and palate should an SLP make to best optimize feeding for adequate nutrition and appropriate growth? A.Providing a bottle rather than breast-feeding the infant B.Positioning the infant in a supine position during feeding times C.Allowing the infant to take as much time as needed to feed D.Using a bottle with a modified nipple during feedings

Correct Answer: D Option (D) is correct. A bottle with a modified nipple allows for greater control over the quantity of liquid expressed and the pacing of feeding.

A 3-year-old child was seen at the speech-and-language clinic for a speech assessment to address teacher and parent concerns about speech intelligibility. During the initial interview and observation, the SLP noted that the child exhibited inconsistent errors of vowel and consonant production during repetitive speech tasks, inappropriate prosody, and prolongations of speech sounds. The SLP chooses an assessment to determine the presence of childhood apraxia of speech (CASC A S). Which of the following procedures is most appropriate for this type of assessment? A.Analyzing place, manner, and voicing of all consonant sounds at the word level B.Evaluating fluency of speech during structured and unstructured conversational activities C.Testing stimulability of later-developing speech sounds in multiple phonetic contexts D.Determining speech production in a variety of syllables-to-sentence combinations

Correct Answer: D Option (D) is correct. A hallmark of childhood apraxia of speech is inconsistent consonant and vowel productions in repeated production of words or syllables. One appropriate way to identify areas of need based on these productions is to look at speech production in a progression of tasks from simple to complex.

Which of the following assessment descriptions represents an s/z ratio that is indicative of a vocal pathology? A.1.0 B.1.1 C.1.3 D.1.5

Correct Answer: D Option (D) is correct. An s/zs slash z ratio greater than 1.4 is indicative of possible laryngeal pathology. An s/zs slash z ratio greater than 1 means the client prolonged the /s/forward slash s forward slash phoneme longer than the /z/forward slash z forward slash phoneme. In other words the client was able to produce a longer sound when the vocal folds were not involved, and sound prolongation was less when the vocal folds were involved.

Alan, a 62-year-old, right-handed African American male, sustained a traumatic brain injury mostly affecting his right hemisphere and bi-lateral frontal lobes. Alan was hospitalized for 24 days before being discharged to an inpatient brain injury rehabilitation program. His spouse attends his initial evaluation sessions. The SLP reports impairments in memory, poor awareness of deficits, and some changes in his communication skills. The SLP already has current information about Alan's performance on a standardized language battery suggesting minimal impairments. Therefore, the SLP completes an evaluation of Alan's functional communication skills. The SLP's assessment involves testing his functional use of humor, facial expressions, nonverbal communication strategies, and understanding functional written materials. The SLP hopes to use this information to determine the impact of Alan's impairments on his daily life. The SLP determines that Alan would benefit from an external memory aid, specifically a memory notebook. The SLP wants to be sure that Alan can learn to use the aid but is concerned that because of his memory impairment, he will struggle to retain the basic information about how to use the aid. The SLP discourages Alan from guessing and intervenes with support before Alan can make a mistake when using the device. Alan's performance profile reveals which impairment typical to right hemisphere dysfunction? A.Aprosodia B.Unilateral spatial neglect C.Hyperresponsiveness D.Anosognosia

Correct Answer: D Option (D) is correct. Anosognosia, the reduced awareness of deficits, is common after right hemisphere disorder.

An SLP plans to give Cody, an 8-year-old boy with autism spectrum disorder, tangible reinforcement in conjunction with the use of manual signs during an object labeling task. At first, reinforcement will be presented every time Cody produces a sign correctly. After several sessions, however, reinforcement will be given after every third instance of correct labeling. Which of the following choices best describes the SLP's reinforcement schedule? A.Fixed-ratio schedule followed by variable-ratio schedule B.Fixed-ratio schedule followed by continuous schedule C.Continuous-ratio schedule followed by fixed-interval schedule D.Continuous-ratio schedule followed by fixed-ratio schedule

Correct Answer: D Option (D) is correct. At first the desired behavior is reinforced every time it occurs followed by a fixed ratio of every third time.

Esophageal and tracheoesophageal (TEPT E P) techniques for producing alaryngeal voice are similar in that both A.redirect expiratory airflow from the lungs to the esophagus B.result in a vocal fundamental frequency similar to that of laryngeal phonation C.require the speaker to occlude the stoma with either a thumb or a valve D.depend on adequate vibration of the pharyngoesophageal (P EP E) segment

Correct Answer: D Option (D) is correct. Both esophageal speech and tracheoesophageal speech require vibration of the P EP E segment. Esophageal speech uses air from stomach/lower esophagus to upper esophagus/pharynx, where it vibrates the wall. TEPT E P speech requires the patient to cover the stoma to redirect air and initiate vibration. Fundamental frequency is different than that of laryngeal phonation (e.g.for example, variability and prosody).

Which of the following should an SLP recommend to best help a patient who has advanced to late stages of dementia of the Alzheimer's type? A.Group treatment to improve the patient's conversational intelligibility B.Individual treatment to improve the patient's recall of salient vocabulary words C.Individual treatment to improve the patient's comprehension during social discourse D.Assistance from caregivers to improve the patient's communication skills

Correct Answer: D Option (D) is correct. In the late stages of dementia of the Alzheimer's type the most appropriate intervention is to engage caregivers as facilitators of clients' communication of wants and needs. Restorative treatment is not a goal at this stage.

Two months after undergoing surgery to improve velopharyngeal function, a client continues to exhibit nasal airflow only on the production of /s/ and /sh/forward slash s forward slash and forward slash s h forward slash, and also exhibits glottal stops for several pressure consonants. The most appropriate next action for the SLP to take is to A.request consideration of prosthetic management B.request a nasoendoscopic study C.inform the surgeon that the client is not making satisfactory progress D.provide speech treatment to correct compensatory articulation errors

Correct Answer: D Option (D) is correct. It would be appropriate to provide speech-language services with the objective of reducing and eliminating the speech-production errors evidenced by the patient.

Which of the following instruments is often used to document stages of recovery after traumatic brain injury (TBIT B I) ? A.Glasgow Coma Scale B.Scale of Executive Function C.Sequenced Inventory of Communication Development D.Rancho Los Amigos Scales of Cognitive Function

Correct Answer: D Option (D) is correct. The Rancho Los Amigos Scales of Cognitive Function is the only truly "scaled" instrument developed specifically for rehabilitation documentation following TBIT B I.

Infants with cleft lip and palate are susceptible to middle ear disease because which of the following muscles is commonly impaired? A.The superior constrictor muscle B.The levator veli palatini muscle C.The palatopharyngeus muscle D.The tensor veli palatini muscle

Correct Answer: D Option (D) is correct. The tensor veli palatini muscle when contracted opens the auditory tube, equalizing middle ear pressure. When the tensor is not functioning properly, the auditory tube is not opened, pressure is not equalized, and fluid may accumulate in the middle ear. The tensor veli palatini travels around the hamulus of the sphenoid bone, where it has a fanlike appearance, and becomes the palatine aponeurosis, extending from the hard palate to the free border of the soft palate.

Of the following, which is generally the most appropriate treatment goal for clients who have had a laryngectomy? A.Acceptance of the alaryngeal status B.Production of an esophageal voice C.Use of a voice prosthesis D.Restoration of oral communication

Correct Answer: D Option (D) is correct. The ultimate treatment goal for clients who have had a laryngectomy is the restoration of oral communication, no matter how it is addressed.

Which of the following neuroimaging studies specifies which artery or arteries is/are occluded in a patient with a stroke? A.Electroencephalography B.Magnetoencephalography C.Evoked-response potentials D.CT or MR angiography

Correct Answer: D Option (D) is correct. These studies will give results which specify which artery is occluded.

Place the examples of assessment tasks with the type of attention that is primarily being evaluated. A.Listening to a list of spoken words for a target word B.Focusing on hearing a person speaking while the television is on C.Mentally solving a complex math problem DSorting playing cards by color, then by number, and then by color again

Correct Answer: D, B, A, C Alternating Attention—Sorting playing cards by color, then sorting by number, and then sorting by color again. Sustained Attention—Focusing on hearing a person speaking while the television is on. Selective Attention—Listening to a list of spoken words for a target word over a 3-minute period. Working Memory—Solving a complex math problem in your head. Any other order is incorrect because as Sohlberg and Mateer (2010) lay out the types of attention, their descriptions match the assessment tasks described below. That is, sustained attention involves maintaining attention over a period of time (3 minutes); selective attention involves a distraction (television); working memory involves holding information in your head and manipulating that information (math problem); and alternating attention involves switching between two tasks or two parts of a task (sorting by color and then by number).

Following anoxic encephalopathy, clients are likely to experience the most significant long-term impairments in the area of A. prosody B. resonance C. aphonia D. memory

D Anoxic encephalopathy, or brain damage resulting from oxygen deprivation, typically leads to global impairment that affects memory. It is difficult to reverse the effects of memory loss, so the impairment is usually long-term.

The major objective of auditory training in the treatment of a client with a hearing loss is to A.improve the client's awareness of position and movements of the speech mechanism B.improve the client's kinesthetic and auditory awareness C.increase the client's kinesthetic and proprioceptive discrimination D.teach the client to make discriminations among speech sounds

D Auditory training focuses on the interpretation of auditory input and would thus teach a client to discriminate speech sounds

A child with discourse problems is most likely to need remediation directed at which of the following? A .Morphology B. An initial lexicon C. Gestural communication D. Cohesive devices

D Cohesive devices, such as pre-nominal references, coordinating conjunctions, and conjunctive adverbs, are used to link clausal and sentential elements to form a coherent and unified message.

Research regarding the use of intensive phonemic-awareness treatment for children who have difficulty learning to read has demonstrated that the treatment A. is effective only for children from 4 to 8 years old B. is effective mainly with children who have remediated all phonological process errors C. is effective only when combined with a supplemental literacy program D. might have no direct relationship to improvement in reading abilities

D Intensive phonemic-awareness treatment programs are thought to improve reading by training children to better differentiate and process speech sounds. However, to date, research has not successfully separated the effects of intensive intervention, so no direct relationship has been proved.

Intervention from an SLP for a nursing-home resident who is in a late stage of progressive dementia will most effectively focus on A. conversational intelligibility B. recall of salient vocabulary words C. comprehension of social discourse D. assisting in communication routines

D Intervention from an SLP for a nursing-home resident in a late stage of progressive dementia would most effectively focus on assisting the resident with daily communication functions.

If a child's language exhibits the phonological process of gliding, the child might say [wɛd] for "red." When asked, "Do you mean wed?" the child may respond, "No! [wɛd]!" Such a response demonstrates which of the following? A. Phonological development lags behind semantic development. B. Semantic development lags behind phonological development. C. Linguistic competence lags behind linguistic performance. D. Linguistic performance lags behind linguistic competence.

D Linguistic performance typically lags behind linguistic competence.

Questions 64-67 refer to the following. Michael is a 32-month-old boy who has been receiving early intervention services over the past ten months for delayed speech and expressive-language development. Although his birth was reportedly unremarkable, Michael does have a history of recurrent otitis media with effusion. His parents described him as having been a "well-behaved and quiet baby." When Michael began receiving services, he communicated mainly through gestures and crude vocalizations. An open resting mouth position with slight tongue protrusion was sometimes noted. However, his receptive-language skills were found to be age appropriate and he showed no oral motor deficits during feeding. Michael's expressive-language skills have shown some progress since he began working with the speech-language pathologist, but he remains poorly intelligible. Michael's imitation of tongue, lip, and jaw movements is characterized by inconsistent groping and errors of sequencing not observed in his spontaneous oral movements. Michael has an age-appropriate vocabulary and produces utterances of up to five words. Articulation errors, especially metathesis of phones and syllables, increase as his utterance length increases. Michael's intelligibility is greatest at the single-word level. Automatic speech and highly familiar utterances are much more intelligible than his imitated productions. QUESTION: Based on Michael's case history, the SLP would most appropriately provide activities to A. stabilize muscle tone to improve gross motor support of speech movement B. improve receptive-language ability C. strengthen tongue, lip, and jaw muscles D. increase the accuracy of CV, VC, and CVC syllable sequences

D Michael's case history suggests that his poor intelligibility is tied to his inconsistent sequencing of speech sounds and syllables.

Questions 64-67 refer to the following. Michael is a 32-month-old boy who has been receiving early intervention services over the past ten months for delayed speech and expressive-language development. Although his birth was reportedly unremarkable, Michael does have a history of recurrent otitis media with effusion. His parents described him as having been a "well-behaved and quiet baby." When Michael began receiving services, he communicated mainly through gestures and crude vocalizations. An open resting mouth position with slight tongue protrusion was sometimes noted. However, his receptive-language skills were found to be age appropriate and he showed no oral motor deficits during feeding. Michael's expressive-language skills have shown some progress since he began working with the speech-language pathologist, but he remains poorly intelligible. Michael's imitation of tongue, lip, and jaw movements is characterized by inconsistent groping and errors of sequencing not observed in his spontaneous oral movements. Michael has an age-appropriate vocabulary and produces utterances of up to five words. Articulation errors, especially metathesis of phones and syllables, increase as his utterance length increases. Michael's intelligibility is greatest at the single-word level. Automatic speech and highly familiar utterances are much more intelligible than his imitated productions. QUESTION: As part of ongoing assessment, the SLP's most appropriate action is to have Michael's parents consult with his primary-care provider for referral to A. a special educator B. a neurologist C. a psychologist D. an otolaryngologist

D Michael's history of recurrent middle ear infections, an otolaryngologist would be able to clinically manage otitis media & make provisions to obtain an audiological assessment

Mr. Charles, age 78, has had Alzheimer's disease for the past nine years. A recent speech and language evaluation at his nursing home indicated severe deficits in verbal reasoning, memory, word finding, discourse, pragmatics, phonology, semantics, and syntax. Which of the following should the speech-language pathologist do next? A. Recommend individual treatment to improve his receptive and expressive language skills, beginning with the areas where he is least deficient B. Recommend individual treatment to improve his receptive and expressive language skills, beginning with the areas where he is most deficient C. Initiate group treatment with other adults with language impairments to improve spontaneous conversational speech and pragmatic skills D. Examine his living conditions and, if necessary, educate the nursing staff concerning ways to help him communicate more effectively

D Mr. Charles has Alzheimer's disease with loss of memory and deteriorating language skills. The best that can be done for him is to help his caregivers improve the conditions related to his basic communication needs. Speech-language treatment itself is very unlikely to be effective & thus is not appropriate.

Which of the following is a type of perturbation that can be measured to determine the amount of noise in the voice? A. Changes in the frequency range between F1 and F2 over time B. Changes in the frequency range between F2 and F3 over time C. F3 cycle-to-cycle variations in sound energy over time D. F0 cycle-to-cycle variations in sound energy over time

D Perturbation is a disturbance in the quality of the laryngeal tone, or fundamental frequency, of the voice.

Primary motor innervation to the larynx and velum is provided by which cranial nerve? A. V B. VII C. IX D. X

D Primary innervation to the larynx and velum is provided by cranial nerve X, the vagus nerve. The other answer choices identify cranial nerves that are not primarily involved in motor innervation to the larynx and velum.

An SLP has a consultation with a self-referred adult who has a fluency disorder. The client had been enrolled in treatment programs with the clinician three times before and had reached from 75 to 90 percent fluency before dropping out of treatment for various reasons. Thirty percent of the client's syllables are spoken disfluently. The client also exhibits signs of depression and anxiety. Which of the following is the most appropriate action for the SLP to take? A.Encouraging the client to re-enroll for remedial services B.Encouraging the client to take responsibility for maintaining fluency by using techniques learned in the previous treatment sessions C.Recommending that the client schedule a neurological evaluation D.Referring the client for psychological counseling

D Psychological counseling is most appropriate for a client who appears interested in improving speech but has not remained motivated long enough to complete the several treatment programs started. The client also does not exhibit maintenance of benefits from prior treatment. Thus, psychological counseling should precede any further remedial efforts.

Which of the following is the most important acoustic cue that distinguishes between an unreleased final /p/ and an unreleased final /b/, as in "cap" versus "cab"? A.Locus frequency of burst B.Voice onset time C.Vocal fundamental frequency D.Duration of the preceding vowel

D Research shows that vowel duration influences a listener's perception of voicing. Vowels that precede unreleased voiced stop consonants are as much as 1.5 times as long as vowels that precede voiceless stops.

Which of the following actions will most effectively control the problem of overreferral in school screening programs that use impedance/immittance measurements? A. Obtaining the measurements in a professional sound-insulated room B. Including 500 Hz in the audiometric screening procedure C. Retesting immediately those who did not pass the first screening D. Waiting three to five weeks to retest those who did not pass the first screening

D Some children may have a temporary problem due to a cold or ear infection (very common in children), which may resolve in a few weeks.

In the treatment of voice disorders, the chewing technique is used to do which of the following? A. Improve control of loudness B. Increase pitch range during voice production C. Increase air supply during voice production D. Reduce tension in the laryngeal area

D The chewing technique is used to reduce muscular tension in the laryngeal area

A 3-year-old child presents for an evaluation of communication skills. When the SLP says "Sit in your seat," the child responds by saying [tɪ ɪ ti]. When the SLP asks the child to "put the big block in the box," the child responds by saying [bɪ bɑ ɪ bɑ]. Based on the responses, the child's primary problem with communication is most likely which of the following? A .Oral-motor weakness B. Poor auditory discrimination C. A fluency disorder D. A receptive and/or expressive language impairment

D The child is imitating what the SLP is saying but not adding new info to the conversation. This may be due to difficulty comprehending what is said (receptive lang.) and difficulty with producing sounds & words (expressive lang.).

A 42-year-old client with upper-and lower-extremity weakness and a diagnosis of amyotrophic lateral sclerosis is referred for a speech-language evaluation. The evaluation reveals a progressive severe dysarthria that is characterized by imprecise articulation secondary to bilateral facial and lingual weakness, atrophy, and fasciculations; mild-to-moderate hypernasality and weak pressure consonants with associated nasal emission during speech; and strained, harsh, groaning voice quality with occasional inhalatory stridor. Speech intelligibility is poor. Which of the following will most effectively improve this client's ability to communicate? A. Teflon injection into one or both vocal cords B. Palatal-lift prosthesis C. Amplification device D. Augmentative communication system

D The client has an advanced stage of ALS with consequent progressive deterioration of communication abilities. An AAC system is the best option for improving or maintaining communication for this client.

A 4-month-old-infant who has a low birth weight but passed a neonatal hearing screening was evaluated for development of communication skills. The speech-language pathologist found that the infant followed moving objects visually, showed interest in mouthing and banging objects, and began sucking in anticipation of eating, but failed to localize to environmental sounds. On the report to the infant's primary care physician, the most appropriate recommendation by the speech-language pathologist is A.consideration of auditory amplification B.hearing-loss counseling for the parents C.careful parent monitoring of the child's speech-language development D.evaluation of auditory function by an audiologist

D The infant is showing normal development except for problems in localizing environmental sounds. This symptom is indicative of a possible auditory problem, making it appropriate for the infant to be referred to an audiologist for evaluation of auditory function.

For a test of expressive morphology and syntax for speakers of African American Vernacular English (AAVE), the test item that would be considered LEAST biased against such speakers would be one requiring A. use of the auxiliary verb "be" in the present progressive tense B. use of the past-tense ending "-ed" C. use of sentences with multiple negation D. agreement of personal pronouns with their antecedents in gender and number

D The morphological feature listed is one in which AAVE does not differ from Standard American English (SAE).

An SLP is planning treatment for a 5-year-old child with multiple speech-production errors. The most effective strategy the clinician can use to treat the child is to A. arrange error sounds by developmental pattern and correct them sound by sound B. start with sounds the child can make and use them as bridges to error sounds C. teach sounds in isolation, then use nonsense syllables, and then build to words D. delineate phonological processes in operation and address them through minimal-contrast pairs

D The most effective procedure for the child would be for the SLP to define the phonological processes in operation and address them through minimal-contrast pairs.

A prospective client is described as a man in his forties who is under chronic stress. He uses his voice extensively in daily life has a hard-driving personality, and exhibits glottal fry. The client has the classic profile of a person at high risk for A.spastic dysphonia B.acute laryngitis C.vocal nodules D.contact ulcers

D The symptoms exhibited by this patient represent a classic profile of a person who has contact ulcers. Contact ulcers are most common in males in their forties.

Halliday described seven functions of communicative intent that develop between 9 and 18 months of age. Which of the following is an example of the heuristic function? A. "More" (said by a child with an empty juice glass). B. "I want cookie." C. "Grandpa white car." D. "Why doggy bark?"

D. "Why doggy bark?"

Sensitivity to sound of the normal ear of a young adult is limited to A. 30 Hz to 30,000 Hz. B. 5 Hz to 5,000 Hz. C. 7 Hz to 70,000 Hz. D. 20 Hz to 20,000 Hz.

D. 20 Hz to 20,000 Hz.

Although differences of opinion exist, some clinicians diagnose stuttering on the basis of the frequency of disfluencies when all forms of disfluencies are counted in a speech sample. If you accept this practice, you would diagnose stuttering if the disfluency rate in a speech sample were A. 10% or more. B. 3.5% or more. C. 15% or more. D. 5% or more.

D. 5% or more.

___________ states that all children and youth with disabilities from ages 3 to 21 years are guaranteed free and appropriate public education in the "least restrictive environment," including special education and related services. A. P.L. 142-204 B. P.L. 29-204 C. P.L. 49-142 D. P.L. 94-142

D. P.L. 94-142

Which of the following statements is true? A. The innermost intercostal (T2-T11) elevates ribs 1 through 11. B. The external intercostal (T2-T11) pulls the rib cage down. C. The pectoralis minor (C4-T1) decreases the transverse dimension of the rib cage. D. The serratus anterior elevates ribs 1 through 9.

D. The serratus anterior elevates ribs 1 through 9.

Which of the following muscles helps adduct the vocal folds? A. Hyoglossus B. Cricothyroid C. Posterior cricoarytenoids D. Transverse arytenoids

D. Transverse arytenoids

The impedance meter can measure a simple reflex response of the muscles attached to the stapes bone. This reflex is called the A. stapes reflex. B. tympanic reflex. C. basilar membrane reflex. D. acoustic reflex.

D. acoustic reflex.

A group of clinicians working with voice disorders wish to conduct research in a hospital setting. Many of their clients are hoarse because they work in noisy factories where they shout a great deal during the work week. The clinicians devise a rating scale to evaluate the hoarseness of these clients. The scale looks like this: Almost no hoarseness Slight hoarseness Moderate hoarseness Great amount of hoarseness This type of scale is called A. a ratio scale. B. a nominal scale. C. an interval scale. D. an ordinal scale.

D. an ordinal scale.

A clinician has been asked to evaluate a patient, Sydney T., who is 65 years old. He was rushed to the emergency room because of trouble with speaking, numbness and loss of movement in his face, vision problems, and confusion understanding simple statements. After completing a medical test, it was determined that Sydney had suffered a cerebrovascular accident (CVA). According to the medical chart, he "has trouble swallowing and suffers from speech and language problems secondary to the CVA." The dietitian is concerned that Sydney has lost weight and is not eating all his meals. The report indicates that "the patient seems depressed and uninterested in eating." The nursing staff indicates that he eats minimally during breakfast, lunch, and dinner. Reportedly, the patient seems to be unable to produce intelligible speech, and the certified nursing assistants are unable to understand him. Sydney's wife indicated that prior to his hospitalization he was an outgoing person and coached high school football. Question When the clinician evaluates Sydney's language skills, she finds that he has difficulty naming objects. This is an example of A. telegraphic speech. B. jargon. C. paraphasia. D. anomia.

D. anomia.

A clinician at a trauma center receives a referral of a 19-year-old college student, Peter, who was involved in a motorcycle accident. He recently received his motorcycle license. His parents stated, "He is a carefree young man who has always believed he was invincible." The medical chart indicates that Peter was riding his bike at high speeds and was not wearing a helmet when he crashed into a stationary delivery truck. The paramedic report indicates that he was thrown from his bike and went head first into the delivery truck. He was unconscious when he entered the hospital and was rushed to the emergency room. He suffered several fractures. His right leg is in a cast, and he has injuries to several parts of his body. The medical chart also indicates that he has a traumatic brain injury (TBI) and suffers many symptoms of TBI. Emergency surgery was performed to repair hematomas that were a result of the accident. Question Peter's parents are very concerned about his recovery and request an explanation about the type of head injury that he sustained as a result of the accident. The attending physician informs the parents that Peter has a condition known as A. Kaposi sarcoma. B. glioma. C. neuroblastoma. D. closed-head injury.

D. closed-head injury.

When a surgeon closes a cleft of the soft palate first and a cleft of the hard palate later, that is known as A. secondary surgery. B. palatal surgery. C. pharyngeal flap surgery. D. delayed hard palate closure.

D. delayed hard palate closure.

A 72-year-old patient was diagnosed with apraxia of speech post CVA. The clinician instructed the patient to "watch and listen to me" and then "say it with me." The aim of the treatment approach was to use simultaneous practice to bring to consciousness the look and sound of the target speech output. This is an example of A. metrical pacing. B. metronomic pacing. C. articulatory cuing. D. integral practice.

D. integral practice.

The source-filter theory of speech production states that A. the larynx filters the sound before the sound travels through the vocal tract. B. the resonating cavities of speech are the source of speech production. C. the oral cavity is the source of all English sounds. D. the laryngeally produced sound is modified by the resonating cavities.

D. the laryngeally produced sound is modified by the resonating cavities.

A 9-year-old child was seen for an evaluation because of a major dysfunction in his gastrointestinal tract. The child had short bowel syndrome, resulting in the removal of a major part of the intestines. The gastroenterologist most likely would recommend A. a jejunostomy tube (J-tube). B. a duodenal tube (duo-tube). C. a gastrojejunal tube (GJ-tube). D. total parenteral nutrition (TPN).

D. total parenteral nutrition (TPN).

What Dementia chorea, irregular spasmoid involunatry movement gait disturbances slow memvemnt excessive complaining schizophrenic like behaviors delusions

Demenia associated with huntington's disease

What dementia fatigue, sleep distrubances memory/reasoning impiarments cerebellar ataxia

Dementia due to Creutzfeldt-Jakob

What dementia subtle memory problems in early stages self neglect, avoidance of routine tasks disorientation agressive naming problems

Dementia of Alzheimer's Type

What dementia slow voluntary movement(bradykinesia) tremors muscle ridgity mask like face dysarthric speech reduced speech volume

Dementia with Parkinson's Disease

in terms of feeding with a cuff

Do not feed when the cuff is inflated.

What is called when a cliniciian expands a child's telegraphic or incomplete utternace into a more grammatically or complete utterance doggy bark yes the doggy is barking

Expansion

A 15-year-old high school sophomore with th/s substitution comes to you for therapy. She is frustrated because she wants to act in high school plays but has been told she cannot do that because of her "speech problem." She is highly motivated to produce /s/ correctly, and you begin seeing her for therapy. If you are using Van Riper's approach, what will you do?

Focus on phonetic placement, auditory discrimination, and drill-like practice at increasingly complex motor levels until accurate /s/ production is automatized.

Sig symbols

Ideographic or pictographic symbols based on ASL and often use in conjuction with ASL

How are some forms of dementia reversible?

If caused by a toxic reaction to medication especially in their early stages

Where are Broca's and Wernicke's area supplied by

Middle Cerebral Artery

Which aphasia is 1. limited spontaneous speech 2.automatic involuntary communication 3. rare variety of non fluent aphasia 4. damage spares and isolates broca's, wernickes, and arcuate fasciculus

Mixed transcortital aphasia

A clinician in a private practice is approached by the parents of Cole, a 5-year-old boy. They want to place Cole in kindergarten in the fall, but they share, "We know there's something wrong with him—we're just not sure what." According to Cole's parents, he is a "sweet, lovable boy who will go to anybody. He likes to sing a lot, too." Because the parents live in a rural area, health-care access has been limited. After seeing Cole for the first time, the clinician refers his parents to a neurologist because she suspects that Cole has a syndrome. Cole is small for his age and has an elfin-like appearance characterized by a small chin, turned-up nose, puffiness around the eyes, a long upper lip, and a wide mouth. Cole's teeth are small and widely spaced. The clinician knows that she will probably end up seeing Cole for intervention if his parents are able to bring him on a weekly basis. The clinician will probably be working on which of the following goals?

Overall expressive and receptive language, because children with this syndrome generally have IQs between 50 and 70, although some have good language skill

While completing an Augmentative and Alternative Communication (AAC) assessment, the clinician noted that the client was proficient in unaided communication. Which of the following would be an example of unaided communication?

Sign language

What involves deviaions in the shape/dimensions of mandible and maxilla?

Skeletal malocclusion

sound waves are disturbances in adjacent molecules within a medium

Sound is defined as vibrations or disturbances of molecules in a medium

What technique will narrow or close the vallecula space

Tilting head back

An 84-year-old woman is having difficulties swallowing. She had a stroke 2 months ago and is now being seen as an outpatient. She complains that "food keeps getting stuck in her throat." Her husband mentions that when she swallows a large bolus, she coughs often. He mentions that his wife is very frustrated and cannot eat steaks and other solid foods." He also mentions that she gets very embarrassed when they go out to restaurants to eat dinner because she can only order soups, apple sauce, and other soft foods, as she is afraid of coughing if she eats solid foods. Question The clinician would like to determine where the bolus is lodged, so that she can plan an effective treatment strategy. She visited the radiology department and discussed evaluation options with the radiologist. Which of the following methods would the radiologist most likely suggest to view the bolus in the pharynx?

Videofluoroscopy

A screening test for hearing that uses a vibrating tuning fork (which is placed on the middle of the forehead) to detect unilateral conductive hearing loss and unilateral sensorineural hearing loss is called the

Weber Test

WHAT aphasia results in deficits related to the ability to recognize the adequacy of one's verbal production.

Wernicke's

What aphasia 1.incessant, effortless produced flowing speech with normal or abormal fluenecy 2.rapid rate of speech with normal prody and good articulation 3. intact grammatical structures 4.severe word finding 5. poor auditory comprehension 6. lesion in posterior portion of superior temporal gyrus in left hemisphere 7. impaired repetition

Wernicke's

In terms of culturally and linguistically diverse backgrounds, the development of formal tests have grown out of a framework that is...

Western, Literate, and middle class

In treating the communication deficits of a young adult with traumatic brain injury, you would do which of the following

Withhold attention from irrelevant and inappropriate responses.

Nominal Scale

a category is presented that do not have a numerical relationship to the other

As a clinician in a medically based private practice, you receive a referral of 23-year-old Allison, a college cheerleader. Allison has been a cheerleader since her freshman year at Freeport College; she is now a senior. She works part-time as a telemarketer, and, according to her boyfriend, she "is glued to her cell phone." She also sings in the college chorus. She has been hoarse for several years and tells you during the case history, "I've ignored the way I sound—it's just me. I haven't felt like I've needed to change anything." However, she shares that lately she has been feeling a lot of pain and the hoarseness is substantially worse. She says, "Sometimes when I talk, it's almost like there's a 'double voice.'" She tells you that she is worried because she will graduate from college in 3 months and will be looking for a job. She is worried that employers will not want to hire someone who "sounds like a frog." You immediately refer her to an otolaryngologist for a thorough examination of her vocal folds. You then proceed to do your own instrumental and perceptual evaluation. You come up with a number of findings, including the fact that Allison has increased laryngeal airway resistance, a maximum phonation time of 6 seconds, and dysphonia. You think that she is a probable candidate for phonosurgery but will wait for the otolaryngologist's diagnosis and recommendations. Question You receive a phone call and a report from the otolaryngologist that Allison will indeed need phonosurgery for the presence of bilateral vocal fold polyps (the right polyp is larger than the left). The otolaryngologist wants you to obtain quantitative measurements of Allison's voice before phonosurgery; he wants to use these baseline measures as a comparison with measures taken after phonosurgery to evaluate whether the phonosurgery was successful. To obtain these quantitative measurements, you will probably use

a sound spectrograph.

Stuttering diagnosed on basis of

a total disfluency rate that exceeds an objective criterion (e.g., 5% of words spoken)

Auditory Sequencing is when

ability to identify the order in which auditory stimuli occur

Auditory attention

ability to ignore irrelevant acoustic stimuli and focus on important information

Auditory Memory

ability to mentally sort speech stimuli or remember what one has heard

What impedance meter measures a simple reflex response of the muscles attached to the stapes bone

acoustic reflex

impedance is the

acoustical, mechincal, electrical resistance to motion or sound transmission

Thyroid cartilage

adams apple largest of all laryngeal cartilage

Transverse arytenoids do what to vf

adduct vocal folds

what model says a stuttering persons hesitations and repetitions indicate a conflict between a desire to approach speaking and equally strong desire to avoid them

approach-avoidance model

Treatment for apraxia of speech appropriately emphasizes

auditory-visual stimulation, oral-motor repetition, and phonetic placement.

What is it called when the external ear canal is completely closed?

aural atresia

Broca's area is often caused by damage in

brodmann's areas 44 and 45

two or more sounds of different frequencies are called

complex tones

The epithelium, the superficial layer of the lamina propria and much of the intermediate layer of the lamina propria vibrate as a "cover" that is made up of the remainder of the intermediate layer, the deep later and the TA muscle is called what

cover body theory of phonation

agent + action is shown though the utterance of "doggie ball."

doggie ball."

A 45-year-old man was referred to a gastrointestinal physician for an evaluation because of respiratory distress and difficulties swallowing. The physician conducted a barium esophagram study and noted an oblique filling defect in the posterior esophagus. On completing her evaluation, the attending cardiologist also mentioned to the patient that she noticed an aortic arch anomaly (an aberrant right subclavian artery—the right subclavian artery arose from the left side of the aortic arch and was externally compressing the esophagus). The cardiologist mentioned to the patient that this vascular anomaly in the thorax, particularly in the aortic arch, was compressing the trachea and esophagus and resulted in the patient experiencing significant respiratory distress and feeding difficulties. This condition is called

dysphagia lusoria

what procedure is used to study the pattern of electrical activity of the vocal folds and view muscle activity pattern?

electromyography

What does the serratus anterior do

elevate ribs 1-9

Concrete operations stage

employs logical causality

What is it when the clinician comments on the childs utterances and adds new and relevant information play ball yes, you are playing with a big red ball

extension

Aural ateresia is when

external ear canal is completely closed

What is an infection of the outer ear canal called?

external otitis

What is a frequency with a region with concentrated acoustic energy

formant frequency

What results are obtained from a visi pitch

frequency range, optimal pitch, and habitual pitch

Formant frequency

frequency region with concentrated acoustic energy

Natural frequency

frequency with which a source of sound normally vibrates

Flaccid dysarthria and dysphagia

frequently co exist

What is the lowest intensity of a sound that will stimulate the auditory system

hearing level

in spite of their significant disadvantages, standardized tests are used by most clinicians in assessing clients because such tests

help qualify children for clinical services in public schools.

what is an iconic symbol that indicates the word house and have a picture of a house

hieroglyphic pictures

Videokymography

high speed imagine method to visualize vocal fold vibration dynamics

Stress

important in differentiating noun and verb paris that have identical sounds

A clinician who concentrated on syntax during therapy with children with language delays and did not believe in external reinforcement would probably subscribe to the

nativist theory

pink or white wart like growth that can be found anywhere in airway that makes persons voice hoarse, breathy, and low

papilloma

You are asked to develop a management plan for a patient with right hemisphere syndrome. Your treatment targets would include

pragmatic language impairments.

Where is primary motor cortex in the frontal lobe located

precentral gyrus

The velopharyngeal problem could be assisted by

prosthetic or surgical intervention.

When sounds waves move from one medium to another, motion causes bending of the sound wave due to change in it speed of propagation

refraction

In people who stutter who have laryngeal dysfunction may have

slight delayed voice onset time

Athenoid CP

slow writhing involunary movements d damage in indirect motor pathways, basal ganglia

What theory is the structure of human language may have arisen from language's social communicative function in human relations?

social interactionist

Indirect laryngoscopy

specialist uses a bright light source and small, round 21-25 mm mirror angled on a long slender handle to lift the velum and press gently against her patient's posterior pharyngeal wall maneuvers the mirror to view the laryngeal structures during quiet respiration while patient produces eeee

What is a measure of validity denoting the extent which scores deviate from the mean/average score

standard deviation

An approach that uses emotion orientated therapy by playing audio recordings of relatives of the patient hoping to decreased agitation and improve well being of patient is called ALZHEMIZERS

stimulated presence therapy

Manner of articulation examples

stop, fricative, affricates, nasals

What technique includes inhale and hold breath tightly by bearing down and swallow while holding breath and bear down

super-supraglottic swallow

A 32-year-old patient was involved in a motor vehicle accident. He is a ventilator-dependent patient with a tracheostomy tube in place. Because the ventilator controls the respiratory cycle, the patient cannot lengthen exhalations; however, the physician recommends trial therapy for tolerance for swallowing. In preparing for feeding and swallowing, which of the following is recommended?

t is preferable to present food to the patient at the beginning of the exhalation phase of the respiratory cycle.

The two muscles in the middle ear that dampen the vibrations of the tympanic membrane and the ossicular chain are the

tensor tympani and stapedius muscle.

You are working in a neonatal intensive care unit (NICU) in a hospital. You often evaluate the infants there by using Oller's stages as a guide. One of Oller's stages of infant vocalization, in which the infant continues to use adult-like syllables in CV (consonant-vowel) sequences but uses a variety of consonants and vowels in single vocalizations, is known as the

variegated or nonreduplicated babbling stage.

the volume of air that a patient can exhale after maximal inhalation

vital capacity

You are evaluating the language skills of Pascal, who has come to the United States with his family from Mexico. As you are gathering a language sample, you hear many utterances that reflect the transfer of Spanish to English. Which of the following would be a typical utterance for a child from a Spanish-speaking home who is learning English in elementary school? A. "This balloon is more big." B. "I don't not have no more balloons." C. "The big house be red." D. "The girl's book done be gone."

A. "This balloon is more big."

A semi-vowel that can be categorized as a voiced bilabial glide that is + anterior and + continuant is the A. /w/. B. /j/. C. /l/. D. /h/.

A. /w/.

ou move to a new elementary school and begin seeing children on the caseload. Johnny, a 7-year-old child, is receiving intervention to "increase semantic skills." Five goals are listed on his IEP. Which one of these goals is inappropriate? A. Increase use of appropriate discourse skills. B. Increase numbers and types of words he uses in the classroom. C. Increase knowledge of antonyms. D. Decrease use of overextensions of words.

A. Increase use of appropriate discourse skills.

When a person reads aloud a brief printed passage, the frequency of stuttering may decrease from the first to subsequent readings. Select the statement that is true of this phenomenon. A. It is known as the adaptation effect. B. Reduction in stuttering frequency is highest on the 10th reading. C. The effect transfers from one reading passage to another. D. The magnitude of the effect increases with increasing intervals between the oral readings.

A. It is known as the adaptation effect.

While working in a hospital setting, you are asked to evaluate a 70-year-old patient who has had a brainstem stroke. The medical records indicate that the patient has difficulty swallowing. When you conduct test swallow trials, you observe anterior tongue movements, food residue in the anterior and lateral sulcus, premature swallow, and reduced range of tongue elevation. Which of the following is the most likely diagnosis you would make of this patient? A. Predominantly a disorder of the oral phase B. Predominantly a disorder of the pharyngeal phase C. Predominantly a problem of delayed or absent swallowing reflex D. Predominantly a problem of a tracheoesophageal fistula

A. Predominantly a disorder of the oral phase

Select the true statement. A. T-units contain an independent clause and one or more subordinate clauses. B. T-units are types of linguistic units. C. C-units may never contain incomplete sentences produced in response to questions. D. C-units do not contain one or more independent clauses.

A. T-units contain an independent clause and one or more subordinate clauses.

You give a new test of cognition to patients on your hospital caseload who have been diagnosed with Alzheimer's disease. You find a vast spread in the scores of the patients, depending upon how advanced their Alzheimer's disease is. You want to determine the range of scores of your patients. What can you calculate to do this? A. The difference between the highest and the lowest scores of the patients to whom the test has been administered B. The lowest and highest 25% of the scores C. The middle 50% of the scores D. The middle 50% of the scores divided by 2

A. The difference between the highest and the lowest scores of the patients to whom the test has been administered

Select the statement that applies to ethnographic studies. A. They are mostly descriptive. B. They are suitable for evaluating treatment effects. C. They are well established in speech-language pathology. D. They are not appropriate for studying patterns of cultures.

A. They are mostly descriptive.

Consonants and vowels have certain distinct characteristics. Of the following statements, which one accurately distinguishes the two classes of speech sounds? A. Vowels form the nucleus of syllables, whereas consonants release and arrest syllables. B. Consonants are syllabic, whereas vowels cannot stand alone to form syllables. C. The vocal tract is constricted for vowel productions, whereas it is open for consonantal productions. D. Consonants may be described in terms of distinctive features, whereas vowels cannot be so described.

A. Vowels form the nucleus of syllables, whereas consonants release and arrest syllables.

Bloodstein suggested that stuttering is A. a response of tension and speech fragmentation. B. an avoidance behavior. C. due to parental diagnosis. D. due to genetic factors.

A. a response of tension and speech fragmentation.

A researcher is describing the speech of a group of children who clutter. She finds that the faster the children speak, the less intelligible they are. The researcher obtains a Pearson r correlational relationship of −.89. This shows that there is ___________________ between rate of speech and intelligibility. A. a strong negative correlational (or inverse) relationship B. a positive correlational relationship C. a moderate cause-and-effect relationship D. virtually no correlational relationship

A. a strong negative correlational (or inverse) relationship

A clinician who works in a skilled nursing facility has been referred a patient with swallowing disorders. The referring physician states that the patient has a weak cricopharyngeus, causing difficulties in passing the bolus through the cricopharyngeus muscle and past the 7th cervical vertebra. Most likely this patient has A. disorders of the esophageal phase of swallow. B. difficulties in propelling the bolus through the pharynx and into the P-E segment. C. difficulty in forming and holding the bolus, accompanied by slippage of food into the lateral sulcus. D. food residue in the vallecula, on top of the airway, and in the pyriform sinuses.

A. disorders of the esophageal phase of swallow.

Acoustical, mechanical, or electrical resistance to motion or sound transmission is called A. impedance. B. admittance. C. immittance. D. velocity.

A. impedance.

A clinician measured the disfluency rates of a client from a speech sample. She then asked another clinician to measure disfluencies using the same method she had used. The first clinician calculated a reliability index based on her measure and that of the second clinician. This index is a measure of A. interobserver agreement (or interobserver reliability). B. intraobserver agreement. C. test-retest reliability. D. split-half reliability.

A. interobserver agreement (or interobserver reliability).

A patient was referred for therapy to an outpatient rehabilitation facility because of a diagnosis of apraxia of speech post CVA. The clinician decided to use a shaping technique that focused on orofacial and articulatory postures with specific instructions about how to change current speech and non-speech movements to achieve the target sounds. This is an example of A. phonetic derivation. B. phonemic awareness. C. rate control. D. integral practice.

A. phonetic derivation.

The primary motor cortex in the frontal lobe is located on the A. precentral gyrus. B. homunculus. C. supramarginal gyrus. D. angular gyrus.

A. precentral gyrus.

A clinician is teaching a patient a technique for dysphagia that includes having the patient manually lift the larynx to improve swallowing functioning, because of reduced laryngeal elevation and cricopharyngeus opening. The clinician teaches the patient to take liquid in the mouth and hold it while simultaneously placing the index finger and the thumb around the thyroid notch. The clinician then instructs the patient to hold the larynx at the highest point of laryngeal elevation and swallow, continuing to hold the thyroid notch for a few seconds during and after the swallow before releasing that hold. This technique is called A. the Mendelssohn maneuver. B. an effortful swallow. C. a supraglottic swallow. D. a super-supraglottic swallow.

A. the Mendelssohn maneuver.

You have been asked to supervise a student clinician who has never worked in an early intervention setting before. The clinician has assessed many elementary-age children but has no experience assessing the language skills of infants and toddlers. As you instruct this student, which of the following would you not recommend to her in testing infants and toddlers? A. Assess attention and physiological states such as crying, alertness, and eye opening. B. Avoid working with a multidisciplinary team because that may only confuse and distract the child. C. See if the child has difficulty establishing eye contact, mutual gaze, and joint reference. D. Assess how the caregiver interacts with the child.

B. Avoid working with a multidisciplinary team because that may only confuse and distract the child.

A high school teacher has referred Ashley to you for assessment. Ashley is 17 years old, and she possibly has a language impairment. When you assess Ashley, which specific problems will you look for as evidence of a language impairment? A. Sentences that are average in length but contain omissions of bound morphemes B. Difficulty using cohesion devices (e.g., therefore, for example) C. Normal ability to maintain a conversation, but some use of non sequiturs D. Normal ability to use figurative language, but difficulty with concrete language

B. Difficulty using cohesion devices (e.g., therefore, for example)

You are asked to see a 6-year-old boy, Gabe, for potential therapy because he is very hoarse and has been hoarse for approximately 7 months. He is an active, happy first grader who loves sports and is engaged in various types of sports (e.g., soccer, baseball) year-round. Reportedly, he frequently screams at games. At the school, there is one 15-minute recess in the morning and a 30-minute recess after lunch. You observe Gabe on the playground at recess several times over a period of 2 weeks and see that he loves to run, play, and yell loudly with his friends. His parents have given you a letter from the ENT that definitively states that Gabe has vocal nodules. After an evaluation, what is the first thing you would do? A. Send Gabe to a counselor to probe into any possible psychological or emotional factors that are contributing to his vocally abusive behavior. B. Focus on identification and reduction of vocally abusive behavior such as yelling and screaming; use computer games and prizes to help motivate Gabe to use better vocal habits. C. Monitor Gabe's vocal status by seeing him once every 3 months for the next year to observe whether his hoarseness gets better or worse. D. Prescribe 3 or 4 weeks of almost total voice rest, telling Gabe and his parents that he can speak only when he absolutely has to—no yelling at recess or when he plays sports.

B. Focus on identification and reduction of vocally abusive behavior such as yelling and screaming; use computer games and prizes to help motivate Gabe to use better vocal habits.

You have completed the case history and assessment of a 72-year-old man and have found, among other things, uninhibited and inappropriate social behavior, excessive eating, depression, impaired judgment, and dominant language problems, with better-preserved memory and orientation. The neurologist's report suggests dense intracellular formation in the neuronal cytoplasm. Your most likely diagnosis would be which of the following? A. Dementia of the Alzheimer's type B. Frontotemporal dementia associated with Pick's disease C. Dementia associated with Parkinson's disease D. Dementia due to Creutzfeldt-Jakob disease

B. Frontotemporal dementia associated with Pick's disease

Both the American Speech-Language-Hearing Association (ASHA) and state governments regulate the practice of speech-language pathology. Clinicians often have to meet different requirements for different professional settings. Among the following choices, which statement is incorrect? A. In many states, it is not essential to get the ASHA certification to work as a clinician in public schools. B. In most states, both the state licensure and ASHA certification are required to work as an SLP in public schools. C. Although widely recognized by employers, the ASHA certifications do not have the legal authority of state licensures. D. A state education department's credential is not the same as the same state's licensure.

B. In most states, both the state licensure and ASHA certification are required to work as an SLP in public schools.

elect the correct statement. A. It is not possible to have Broca's aphasia without a specific injury to Broca's area. B. Injury to Broca's area is not essential to have Broca's aphasia. C. Auditory comprehension is typically worse than speech production in Broca's aphasia. D. Syntactic skills are severely impaired in anomic aphasia.

B. Injury to Broca's area is not essential to have Broca's aphasia.

Which one of the following statements is false regarding providing rehabilitative services to adult culturally and linguistically diverse (CLD) patients with neurological impairments? A. Some families may be offended by the rehabilitation team's encouragement of the patient's independence. B. Most standardized aphasia tests are appropriate for use with these patients because the tests have been carefully normed using samples that include CLD individuals. C. Clinicians need to remember that some elderly CLD patients have little money and possibly no health insurance. D. It is important to assess premorbid educational levels and vocational attainments of patients because these affect assessment and intervention.

B. Most standardized aphasia tests are appropriate for use with these patients because the tests have been carefully normed using samples that include CLD individuals.

Jamal is an 8-year-old boy who transfers to your school district. The report from the previous speech-language pathologist is missing from his cumulative file, but you see a note that says, "Jamal has childhood apraxia of speech (CAS)." Which one of the following characteristics will you not expect Jamal to present with? A. Repetition of sounds and syllables B. Rapid speech that becomes faster as Jamal gets more deeply involved in a conversation C. Unusual errors of articulation, such as metathesis (e.g., axe/ask) D. Deviations in prosody

B. Rapid speech that becomes faster as Jamal gets more deeply involved in a conversation

During an informal conversation with an adolescent who has come to you for assessment, you notice the following problems: difficulty in using figurative language, difficulty in using words with multiple meanings, and difficulty using synonyms appropriately. What kinds of language skills do you need to especially target during your assessment? A. Pragmatic language skills B. Semantic language skills C. Syntactic structures D. Morphologic features

B. Semantic language skills

A speech-language pathologist is administering an indirect treatment procedure to the patient. Which is an example of indirect treatment for dysphagia? A. The Mendelssohn maneuver B. Thermal-tactile stimulation C. Supraglottic swallow D. Super-supraglottic swallow

B. Thermal-tactile stimulation

Select the statement that is incorrect about group designs. A. They are effective in establishing internal validity. B. They usually have results similar to those of single-subject experiments. C. They can help establish cause-effect relationships. D. Their requirement of randomization may be difficult to meet.

B. They usually have results similar to those of single-subject experiments.

A child who regularly says, "He the small one," instead of "He is the smallest one," or "She is tall than her" instead of "She is taller than her" has specific problems with A. adjectives. B. comparatives and superlatives. C. irregular past-tense forms. D. word retrieval.

B. comparatives and superlatives.

In standardizing a test of language skills in children, the investigator asked two experts to judge each item on the test to make sure that all items were relevant to children's language skills. This is a method of establishing the A. concurrent validity. B. content validity. C. construct validity. D. predictive validity.

B. content validity.

ohn has cerebral palsy that has resulted in a motor speech disorder caused by central nervous system damage. This damage has caused him to have weakness and incoordination of the muscles of speech. His speech is classified as A. paraphasic. B. dysarthric. C. apraxic. D. aphasic.

B. dysarthric.

A factor that may positively influence pediatric feeding for infants is A. nonnutritive sucking. B. nutritive sucking. C. choanal atresia. D. stenosis.

B. nutritive sucking.

Impaired facial recognition is more common in patients with A. anterior right hemisphere damage. B. posterior right hemisphere damage. C. left temporal lobe damage. D. damage to the perisylvian region.

B. posterior right hemisphere damage.

You are treating an 88-year-old woman with dementia. You introduce a treatment program to improve her quality of life. You begin your therapy by discussing past experiences and events in her life. You also use photographs of the patient's family members as well as music that she likes. You tailor the therapy specifically for the client by using objects and items that her son has provided to you. This treatment method is an example of A. reality orientation (RO). B. reminiscence therapy (RT). C. simulated presence therapy (SPT). D. cognitive stimulation therapy (CST).

B. reminiscence therapy (RT).

Rosalia is a third-grade Mexican American Spanish-speaking 8-year-old girl who is in the process of learning English. Her parents emigrated from Mexico 2 years ago; thus, Rosalia was exposed first to Spanish at home and to English in first grade, when she was 6 years old. The classroom teacher shares with you that she thinks Rosalia may have an articulation disorder, but the teacher is not sure. The teacher provides you with some examples of things that Rosalia has said in the past 2 or 3 weeks. As you look at these examples, which one of the following would not be a typical predictable production based on Spanish influence? A. Omission of /h/ in word-initial position (e.g., -elp/help) B. w/r substitutions (e.g., wing/ring) C. t/th substitutions in word-initial positions (e.g., tin/thin) D. Devoicing of final consonants (e.g., beece/bees)

B. w/r substitutions (e.g., wing/ring)

Which area of the brain connects Broca's area with Wernicke's area? A. Corpus callosum B. Choroid plexus C. Arcuate fasciculus D. Corona radiata

C. Arcuate fasciculus

A patient with amyotrophic lateral sclerosis (ALS; Lou Gehrig's disease) has profound motoric impairments and limited hand mobility. He is not able to use a manual switching device. The medical team decides that he can use a type of augmentative/alternative communication (AAC) that uses bio-electrical signals such as muscle-action potentials to activate and display messages on a computer monitor. This type of AAC is called: A. Picture Exchange Communication system B. Gestural-assisted AAC C. Neuro-assisted AAC D. Eye-blink encoding

C. Neuro-assisted AAC

Which model contends that a child who is unable to cope with the expectations of fluent speech production may begin to stutter? A. The diagnosogenic model B. The expectancy deconfirmation model C. The demands and capacities model D. The approach-avoidance model

C. The demands and capacities model

When working with interpreters to serve bilingual children and their families, all of the following are appropriate except A. Interpreters must be able to relate to members of the cultural group. B. Interpreters must have excellent bilingual communication skills. C. The speech-language pathologist should look at the interpreter and not the client when communicating. D. Interpreters must possess good oral and written proficiency in both English and the primary language.

C. The speech-language pathologist should look at the interpreter and not the client when communicating.

After suffering from a severe stroke, Robert has difficulty with auditory comprehension but speaks fluently, though he does not make much sense. Which area of Robert's brain was affected by the stroke? A. The frontal lobe B. The occipital lobe C. Wernicke's area D. The angular gyrus

C. Wernicke's area

A clinician is teaching a patient a technique for dysphagia that includes having the patient take a deep breath, hold the breath, swallow, cough on exhalation, swallow again before breathing, and then breathe again. This technique is called A. the Mendelssohn maneuver. B. an effortful swallow. C. a supraglottic swallow. D. a super-supraglottic swallow.

C. a supraglottic swallow.

A patient was receiving irradiation to the oral and pharyngeal areas and was experiencing xerostomia, as well as weight loss and an increase in dental caries. During the evaluation, the clinician recommended that the patient A. swallow once prior to taking a spoonful of food in the mouth. B. swallow twice prior to taking a spoonful of food in the mouth. C. be given synthetic saliva just prior to eating. D. use a chin tuck procedure before swallowing.

C. be given synthetic saliva just prior to eating.

A patient comes to you complaining of a sore throat and hoarseness. After talking with the patient, you discover that he also often experiences heartburn and acid indigestion. You consult with the on-call physician, who mentions that this patient's gastric contents are spontaneously emptying into his esophagus. The physician's diagnosis of the problem is A. hyperkeratosis. B. laryngomalacia. C. gastroesophageal reflux. D. leukoplakia.

C. gastroesophageal reflux.

A clinician measured the number of misarticulations in a child's speech sample in December 2015. She went on winter break and came back 2 weeks later, in January 2016, rested and refreshed. She decided to gather another speech sample from the child and remeasure the number of misarticulations. She did this to establish A. construct validity. B. predictive validity. C. intraobserver reliability. D. interobserver reliability.

C. intraobserver reliability.

A concerned mother brings her 3-week-old child to you because he is having difficulties breathing. You refer the mother and her child to a physician. Upon laryngeal examination, the physician notices that a membrane has grown across the anterior portion of the glottis. The physician informs the mother that this problem may be either congenital or acquired and that surgery is required to remove the membrane. According to the physician, the diagnosis would be hyperkeratosis. B. leukoplakia. C. laryngeal web. D. papilloma.

C. laryngeal web.

A clinician is starting a private practice that focuses primarily on accent training for adult clients who speak English as a foreign language (EFL). Which one of the following is NOT an important consideration in the provision of accent training services? A. have one or two unfamiliar listeners listen to clients' speech samples and determine the percent of unintelligible words B. slowing down a client's rate of speech is often effective in making an immediate improvement in intelligibility C. most EFL clients cannot benefit from the use of the Visi-Pitch because the use of this type of technology is confusing D. many EFL clients will appreciate doing activities involving reading and discussing information about their countries of origin

C. most EFL clients cannot benefit from the use of the Visi-Pitch because the use of this type of technology is confusing

You are working in a skilled nursing facility with elderly patients and their families. Many family members are distressed by the problems experienced by these patients secondary to stroke and dementia, and you find yourself spending time counseling with the family members. You mainly use the ______ approach, wherein the family members freely express their emotions while you listen and respond to both the content and the feeling behind the words that they say. A. clinician-centered B. directive clinician-centered C. nondirective client-centered D. nondirective clinician-centered

C. nondirective client-centered

While evaluating the efficacy of a treatment procedure for aphasia, an investigator made sure that the clients in the experimental and control groups did not receive any other form of therapy during the course of the experimental study. This was done to A. increase the amount of improvement in the clients. B. eliminate scheduling conflicts. C. rule out the influence of confounding variables. D. ensure that the Hawthorne effect was not operating.

C. rule out the influence of confounding variables.

A 5-year-old child with cerebral palsy was referred to a pediatric clinic for an evaluation for excessive drooling. The speech-language pathologist noted oromotor dysfunction and a lack of voluntary control of the orofacial musculature. The condition of excessive drooling is called A. nonnutritive sucking. B. stenosis of the anterior faucial pillars. C. sialorrea. D. isolated cricopharyngeal dysfunction.

C. sialorrea.

The softest level of hearing at which a person can understand 50% of the words presented is known as A. pure-tone threshold. B. speech discrimination threshold. C. speech reception threshold. D. word discrimination threshold.

C. speech reception threshold.

A kindergarten teacher refers Julie, a 4-year-old girl, to you for a speech and language screening. During an oral facial examination, you discover that the surface tissue of Julie's soft and hard palates has fused together, but her underlying muscle and bone have not fused. You also note that she has a bifid uvula. On palpation of the soft palate, you discover that Julie has a A. cleft of the alveolar process. B. postmucous cleft. C. submucous cleft. D. rare orofacial cleft.

C. submucous cleft.

The concrete operations stage of cognitive development, defined by Piaget, occurs at what age? A. 2-7 years B. 5-7 years C. 4-8 years D. 7-11 years

D. 7-11 years

Which of the following is not a semantic relation expressed by two-word utterances? A. Notice (e.g., "hi" + noun) B. Attribute + entity (e.g., "brown horse") C. Agent-action (e.g., "kitty meow") D. Affirmation (e.g., "yes cereal")

D. Affirmation (e.g., "yes cereal")

Sara has arterial damage that causes her to have cognitive deficits such as impaired judgment, problems concentrating, and difficulties with reasoning. According to the surgeon, damage to the affected artery can also cause a person to have paralysis of the feet and legs. Damage to which artery produces these effects? A. Internal carotid B. External carotid C. Middle cerebral D. Anterior cerebral

D. Anterior cerebral

Peter has suffered a cerebrovascular accident (CVA); the neurologist reports lesions in the third convolution of the left cerebral hemisphere. Based on this, you conclude that the damaged area is A. the occipital lobe. B. the basal ganglia. C. Wernicke's area. D. Broca's area.

D. Broca's area.

What are the most commonly described suprasegmentals or prosodic features that affect speech production? A. Labials, bilabials, linguadentals, and glottals B. Nasals, stops, fricatives, liquids, and glides C. Consonants, vowels, voiced sounds, and voiceless sounds D. Length of vowels, stress, rate, pitch, volume, and juncture

D. Length of vowels, stress, rate, pitch, volume, and juncture

You are evaluating a 5-year-old boy whose mother has a history of alcohol abuse while she was carrying her son. In your assessment, you would look for specific speech and language problems; in addition, you would look for which of the following positive or negative signs? A. Normal motor and intellectual development, normal play activities, and normal facial and skull features B. Language problems, but no speech problems, coupled with good eye contact and generally compliant behavior C. Lack of gestures, good eye contact, and lack of attachment to new people D. Low birth weight and length, behavior problems, and possible swallowing difficulties

D. Low birth weight and length, behavior problems, and possible swallowing difficulties

You are working with a 9-year-old girl who has been diagnosed with high-functioning autism. She has an excellent vocabulary, as well as good morphological and syntactic skills. However, her pragmatic skills are reportedly delayed, and her parents wish her to gain "better social skills." Which one of the following would not be an appropriate treatment goal for her? A. Initiate and maintain conversation in small groups B. Sustain a topic through a number of conversational turns C. Announce topic shifts D. Make sure conversations with peers focus on her interests and hobbies so that she can maintain control of the topic

D. Make sure conversations with peers focus on her interests and hobbies so that she can maintain control of the topic

Allophones do which of the following: A. Change word meanings B. Are not variations of phonemes C. Are not perceived as the same D. May vary from production to production

D. May vary from production to production

The Health Insurance and Portability and Accountability Act (HIPAA) passed by the U.S. Congress mandates certain rules that all health professionals must follow. Of the following statements regarding HIPAA, select the one that is correct. A. Consumers who think their privacy may have been violated by a medical provider can complain only to the provider in question. B. The law (HIPAA) does not provide for civil or criminal penalties for facilities that misuse patient information. C. HIPAA regulations do not cover electronically transmitted medical records of patients. D. Medical information about a patient may be shared by other professionals who serve the same patient, but it cannot be released for purposes unrelated to the care of the patient.

D. Medical information about a patient may be shared by other professionals who serve the same patient, but it cannot be released for purposes unrelated to the care of the patient.

A classroom teacher refers a 10-year-old African American male student to you because she is concerned about his intelligibility. This teacher is anxious to avoid the mistake of mislabeling the student as having a "speech disorder" if he is merely manifesting characteristics of AAE. When you screen the boy, you find that he makes the following substitutions: d/m, f/n, and m/n. A. Provide therapy for the student because, even though these are typical patterns for speakers of AAE, the boy needs to learn Standard American English articulation patterns. B. Do nothing, knowing that boys mature more slowly than girls do. C. Do nothing, realizing that this is typical for speakers of African American English. D. Provide intervention for the student, because this is a sign of an articulatory-phonological disorder involving substitutions of nasals with other sounds.

D. Provide intervention for the student, because this is a sign of an articulatory-phonological disorder involving substitutions of nasals with other sounds.

The public elementary school where you work has asked you to give an in-service to their teachers on appropriate referrals of bilingual children with possible communication disorders. Which of the following would be appropriate to tell these teachers? A. It is legal to place these children into special education to work on improving their English skills. B. A child who has typical skills and development in the primary language has an underlying language-learning disability if he has low English test scores. C. A bilingual child who has a true communication disorder can never be served in the general education classroom; this child must always receive pullout special education services. D. Some bilingual children attain low scores when tested in their first language due to language loss in that language.

D. Some bilingual children attain low scores when tested in their first language due to language loss in that language.

You work in a school district where increasing numbers of parents are coming to you with issues experienced by their internationally adopted children. Which one of the following would you not expect to be an issue experienced by these children? A. The younger the children are at the time of adoption, the greater their progress in language development is after living in the United States for some time. B. Some of the children show delays in their first or primary languages as well as English. C. Children who show language delays when they first arrive in the United States are likely to continue to show delays in language development over time. D. They maintain their first language skills and become fully bilingual, speaking both their first language and English fluently.

D. They maintain their first language skills and become fully bilingual, speaking both their first language and English fluently.

To evaluate the effects of a phonological treatment procedure, an investigator started by establishing the baselines of target phoneme productions, then offered treatment to all children who were base rated, withdrew treatment for a period of time, and finally offered the treatment again. This is an example of A. a pretest-posttest control group design. B. a multiple-baseline design. C. a case study. D. a ABAB design.

D. a ABAB design.

Beth has central nervous system damage with no weakness or paralysis of her facial muscles; however, her central nervous system damage makes it difficult for her to program the precise movements necessary for smoothly articulated speech. Her motor programming disorder is A. dysarthria. B. aphasia. C. anomia. D. apraxia.

D. apraxia.

Senbo is a third-grade student who speaks Afrikaans. Her teacher refers her for assessment because she is having academic difficulty. To evaluate Senbo's language skills, you use dynamic assessment, an alternative approach in which the clinician A. collects various kinds of work samples from the child being evaluated. B. compares a child's performance with the established norms. C. seeks to sample the child's speech and language in naturalistic settings. D. evaluates her over time in a test-teach-retest format

D. evaluates her over time in a test-teach-retest format

An infant presents with bilateral choanal atresia with accompanying respiratory distress. The speech-language pathologist in consultation with the pediatrician should immediately recommend A. pureed foods. B. thin liquids. C. thick liquids. D. immediate oral airway placement and gavage feedings.

D. immediate oral airway placement and gavage feedings.

A medical chart indicates that a patient's true vocal folds adduct instead of abduct during inhalation. The medical information also indicates that at times the patient's vocal folds remain closed throughout the respiratory cycle. According to the speech-language pathologist, the patient has A. unilateral vocal fold paralysis. B. bilateral vocal fold paralysis. C. spasmodic dysphonia. D. paradoxical vocal fold motion.

D. paradoxical vocal fold motion.

A child is referred to you by his preschool teacher. This child, Jaden, is 4 years 5 months old and has transferred from out of state. In his previous state, Jaden was reportedly assessed by a speech-language pathologist who recommended that he receive intervention before kindergarten. According to the report from the previous clinician, he uses the phonological processes of gliding, consonant-cluster reduction, stopping, reduplication, and final-consonant deletion. Your assessment confirms the presence of these phonological processes. You would begin treatment by addressing A. final-consonant deletion. B. gliding. C. consonant-cluster reduction. D. reduplication.

D. reduplication.

You are conducting therapy with a kindergartener with a speech sound disorder and language impairment. He has deficient phonological awareness skills, and you are targeting these skills in therapy along with other treatment targets. Which of the following are you working on as part of phonological awareness treatment? A. auditory discrimination skills B. metaphon skills C. awareness of print and print conventions D. rhyming, phoneme isolation, sound blending

D. rhyming, phoneme isolation, sound blending

You are seeing a 12-year-old girl at the local junior high school. Her scores on the Peabody Picture Vocabulary Test-Fourth Edition and the Expressive Vocabulary Test-Second Edition were both 2 years below age level. She has many friends and is well liked by her peers. Academically, she struggles with the vocabulary in her textbooks. Therapy should focus primarily on remediation of A. pragmatic skills. B. discourse. C. morphologic skills. D. semantic skills.

D. semantic skills.

The back-and-forth movement of particles when that movement is symmetrical and periodic is called A. an aperiodic wave. B. a sinusoidal wave. C. rarefaction. D. simple harmonic motion.

D. simple harmonic motion.

You refer a patient to an audiologist because you suspect that the patient might have a hearing loss. You ask the audiologist to inform you about the patient's threshold of hearing for selected frequencies. In response to your request, the audiologist will A. tell you the intensity at which tones are faintly heard at least 75% of the time they are presented in a pure-tone hearing test. B. inform you of the intensity level at which tones are faintly heard at least 25% of the time in a speech reception threshold test. C. inform you of the intensity level at which tones are faintly heard at least 50% of the time in a masked speech reception threshold test. D. tell you that the threshold of hearing is the quietest sound a human can detect.

D. tell you that the threshold of hearing is the quietest sound a human can detect.

Research has generally shown that children who stutter have parents A. who hold extremely high standards of fluency. B. who have high ambitions for their children. C. who have unique and aberrant personalities. D. who are similar to parents with children who do not stutter.

D. who are similar to parents with children who do not stutter.


Kaugnay na mga set ng pag-aaral

Practice Assessment for Exam AZ-900

View Set

OB Chapter 2 - Personality and Values

View Set

Geometría Analitica- (Linea Recta,Parábola,Circunferencia,Elipse,Hipérbola)

View Set

Business AS level unit 4 -Operations management

View Set

LIFE INSURANCE, POLICY PROVISIONS, OPTIONS AND RIDERS

View Set

Unit 3 - Campaigns, Elections, Interest Groups and Mass Media

View Set

Wordly Wise 3000® Level 7, Lesson 7

View Set